SlideShare a Scribd company logo
1 of 161
Download to read offline
DIKTAT 
PEMBINAAN OLIMPIADE MATEMATIKA 
TAHUN PELAJARAN 2010-2011 
DISUSUN OLEH : 
EDDY HERMANTO, ST 
SMA Negeri 5 Bengkulu 
Jalan Cendana Nomor 20 
Bengkulu
SINGKATAN 
AHSME : American High School Math Exam 
AIME : American Invitational Mathematics Examination 
APMO : Asian Pasific Mathematical Olympiad 
ARML : American Regions Mathematics League 
COMC : Canadian Open Mathematics Challenge 
Hongkong PSC : Hongkong Preliminary Selection Contest 
India RMO : India Regional Mathematical Olympiad 
MATNC : Mu Alpha Theta National Convention 
ME VXNY : Mathematical Excalibur Volume X Nomor Y 
NHAC : Niels Henrik Abel Contest 
OSK : Olimpiade Sains Indonesia SMA/MA Tingkat Kabupaten/Kota 
OSK SMP/MTs : Olimpiade Sains Indonesia SMP/MTs Tingkat Kabupaten/Kota 
OSN : Olimpiade Sains Indonesia SMA/MA Tingkat Nasional 
OSN SMP/MTs : Olimpiade Sains Indonesia SMP/MTs Tingkat Nasional 
OSP : Olimpiade Sains Indonesia SMA/MA Tingkat Provinsi 
OSP SMP/MTs : Olimpiade Sains Indonesia SMP/MTs Tingkat Provinsi 
QAMT : Queensland Association of Mathematics Teacher 
USAMTS : USA Mathematical Talent Search 
ii
KATA PENGANTAR 
Alhamdulillah Penulis ucapkan kepada Allah, SWT karena dengan karunia-Nya Penulis dapat menyelesaikan penulisan diktat ini. Diktat ini Penulis tulis dalam rangka mempermudah tugas dalam mempersiapkan siswa-siswa SMA menghadapi olimpiade matematika pada tahap-tahap awal. 
Menurut pengamatan Penulis, masih ada jurang pemisah yang cukup jauh antara siswa- siswa dari pulau Jawa dengan dari luar pulau Jawa. Masih sangat banyak siswa-siswa di luar pulau Jawa yang belum memahami persoalan-persoalan dasar di bidang Olimpiade Matematika sehingga mengalami kesulitan besar ketika akan menghadapi OSN. Buku ini berusaha menjawab tentang persoalan-persoalan mendasar di bidang Olimpiade Matematika tersebut. Para guru pembina olimpiade diharapkan dalam membina siswa-siswa juga memberikan soal-soal pada tingkatan OSN pada kegiatan umpan balik yang dapat dilaksanakan setelah guru menyelesaikan pembinaan pada setiap babnya. 
Penulis menyarankan sebelum menyampiakan isi materi buku ini, Pembina Olimpiade di sekolah dapat menyampaikan materi pendahuluan tentang pembuktian langsung maupun tidak langsung serta Strategi Penyelesaian Masalah (Problem Solving Strategies) yang dapat dipelajari dari beberapa buku yang ada seperti Langkah Awal Menuju ke Olimpiade Matematika karya Wono Setya Budhi, Problem-Solving Strategies karya Arthur Engel maupun Problem-Solving Through Problems karya Loren C. Larson. 
Ucapan terima kasih kepada semua pihak yang telah membantu dalam penyelesaian diktat ini, khususnya kepada isteri tercinta Penulis, Rosya Hastaryta, S. Si, yang telah memberi dukungan yang besar kepada Penulis serta juga telah melahirkan puteri pertama kami, Kayyisah Hajidah, pada tanggal 2 Desember 2009. 
Penulis merasa bahwa diktat ini masih jauh dari sempurna. Untuk itu Penulis mengharapkan saran dan kritik dari Pembaca yang budiman sebagai bahan perbaikan diktat ini. 
Akhir kata semoga buku ini dapat bermanfaat yang sebesar-besarnya bagi Pembaca sekalian. 
Bengkulu, September 2010 
EDDY HERMANTO, ST 
Email : eddyhbkl@yahoo.com 
iii
DAFTAR ISI 
HALAMAN JUDUL …………………………………………………………………… i 
SINGKATAN …………………………………………………………………… ii 
KATA PENGANTAR …………………………………………………………………… iii 
DAFTAR ISI …………………………………………………………………… iv 
BAB I ALJABAR 
1. Pemfaktoran dan Penguraian …………………… 1 
2. Barisan dan Deret …………………… 5 
3. Fungsi …………………… 15 
4. Suku Banyak …………………… 19 
5. Persamaan …………………… 25 
6. Sistem Persamaan …………………… 39 
7. Ketaksamaan …………………… 44 
BAB II TEORI BILANGAN 
1. Sifat-Sifat Penjumlahan Dan Perkalian Dua Bilangan …………………… 52 
2. Sifat-sifat Keterbagian …………………… 53 
3. Uji Habis dibagi …………………… 56 
4. Faktor Persekutuan Terbesar (FPB) Dan Persekutuan Terkecil (KPK) …………………… 57 
5. Banyaknya Faktor Positif …………………… 60 
6. Kekongruenan …………………… 61 
7. Bilangan Bulat, Rasional dan Prima …………………… 64 
8. Bilangan Kuadrat Sempurna …………………… 68 
9. Fungsi Tangga dan Ceiling …………………… 69 
BAB III GEOMETRI 
1. Trigonometri …………………… 73 
2. Garis …………………… 77 
3. Segitiga …………………… 79 
4. Segi Empat …………………… 93 
5. Segi-n Beraturan …………………… 98 
6 Lingkaran …………………… 99 
BAB IV KOMBINATORIK 
1. Kaidah Pencacahan Dan Penjabaran Binom Newton …………………… 105 
2. Kejadian dan Peluang Suatu Kejadian, Pengambilan Contoh …………………… 130 
Dengan dan Tanpa Pengembalian 
3. Prinsip Inklusi Eksklusi, Peluang Kejadian Majemuk …………………… 139 
4. Pigeon Hole Principle (Prinsip Lubang Merpati) …………………… 146 
KUNCI JAWABAN LATIHAN …………………… 151 iv
Pembinaan Olimpiade Matematika 
Eddy Hermanto, ST Aljabar 
1 
BAB I 
ALJABAR 
1. PEMFAKTORAN DAN PENGURAIAN 
Beberapa bentuk pemfaktoran maupun penguraian yang harus diketahui adalah : 
(i) x2 − y2 = (x + y)(x − y) 
(ii) x3 − y3 = (x − y)(x2 + xy + y2) 
(iii) x3 + y3 = (x + y)(x2 − xy + y2) 
(iv) x3 + y3 + z3 − 3xyz = (x + y + z)(x2 + y2 + z2 − xy − xz − yz) 
(v) (x + y)(x − y)2 = x3 − x2y − xy2 + y3 
(vi) (an − bn) = (a − b)(an-1 + an-2b + an-3b2 + ⋅⋅⋅ + abn-2 + bn-1) dengan n ∈ bilangan asli 
(vii) (an + bn) = (a + b)(an-1 − an-2b + an-3b2 − ⋅⋅⋅ − abn-2 + bn-1) dengan n ∈ bilangan ganjil 
(viii) (x + 1)(y + 1)(z + 1) = xyz + xy + xz + yz + x + y + z + 1 
(ix) x4 + 4y4 = (x2 + 2y2 + 2xy)(x2 + 2y2 − 2xy) 
(x) (x + y)2 = x2 + 2xy + y2 
(xi) (x + y + z)2 = x2 + y2 + z2 + 2xy + 2xz + 2yz 
(xii) (x − y)2 = x2 − 2xy + y2 
(xiii) (x + y)3 = x3 + y3 + 3xy(x + y) 
(xiv) (x − y)3 = x3 − y3 − 3xy(x − y) 
(xv) (x + y)4 = x4 + 4x3y + 6x2y2 + 4xy3 + y3 
(xvi) (x − y)4 = x4 − 4x3y + 6x2y2 − 4xy3 + y3 
Penguraian bentuk (x + y)n untuk n > 4 dapat menggunakan binomial Newton yang akan diterangkan dalam bagian lain. 
Berdasarkan bentuk (vi) dan (vii) didapat fakta bahwa (a − b) membagi (an − bn) untuk n asli dan (a + b) membagi (an + bn) untuk n ganjil yang terkadang digunakan untuk menyelesaikan soal pada teori bilangan. 
Contoh 1 : 
(OSK 2004 SMP/MTs) Nilai dari LL=−2249505050 
Solusi : 
Perhatikan bahwa a2 − b2 = (a + b)(a − b) maka ()( ) 49505050495050504950505022−+=− ()()1000000100100004950505022==− 000.14950505022=− 
Contoh 2 : 
(OSK 2005 SMP/MTs) Salah satu faktor dari 173 − 53 adalah ⋅⋅⋅⋅⋅⋅⋅ 
A. 5 B. 17 C. 13 D. 273 E. 399 
Solusi : 
Perhatikan bahwa a3 − b3 = (a − b)(a2 + ab + b2) maka 
173 − 53 = (17 − 5)(172 + 17 ⋅ 5 + 52) 
173 − 53 = 12 ⋅ 399 (Jawaban : E)
Pembinaan Olimpiade Matematika 
Eddy Hermanto, ST Aljabar 
2 
Contoh 3 : 
Jika a2 + b2 = 6ab maka tentukan nilai dari baba− + untuk a, b ≠ 0. 
Solusi : 
Misalkan x = baba− + maka 
x2 = ()2baba− + 
x2 = abbaabba222222−+ ++ 
Karena a2 + b2 = 6ab maka x2 = abab48 = 2 
Jadi, baba− + = 2 
Contoh 4 : 
Hitunglah 32+ − 32−. 
Solusi : 
Misalkan X = 32+ − 32− X2 = (2 + 3) + (2 − 3) − 2322− 
X2 = 4 − 2 
X2 = 2 
Maka 32+ − 32− = 2 
Contoh 5 : 
(AIME 1989) Nilai dari 313029281⋅⋅⋅+ adalah ⋅⋅⋅⋅⋅⋅ 
Solusi : 
Mengingat bahwa 1 + (n − 1)(n)(n + 1)(n + 2) = (n(n + 1) − 1)2 maka 313029281⋅⋅⋅+ = 29 ⋅ 30 − 1 313029281⋅⋅⋅+ = 869. 
Contoh 6 : 
Tentukan nilai x yang memenuhi persamaan 32+x − x−7 = 1 
Solusi : 32+x − x−7 = 1 
Karena bilangan dalam akar harus tak negatif maka penyelesaian persamaan tersebut harus memenuhi syarat −23 ≤ x ≤ 7 32+x = 1 + x−7 
2x + 3 = 1 + 7 − x + 2x−7 
3x − 5 = 2x−7
Pembinaan Olimpiade Matematika 
Eddy Hermanto, ST Aljabar 
3 
Kuadratkan kedua ruas dengan syarat 3x − 5 ≥ 0 sebab x−7 ≥ 0 
9x2 − 30x + 25 = 4(7 − x) 
9x2 − 26x − 3 = 0 
(9x + 1)(x − 3) = 0 
x = 3 atau x = −91 (tidak memenuhi syarat x ≥ 35) 
Untuk x = 3 maka 32+x − x−7 = 3 − 2 = 1. 
Jadi, nilai x yang memenuhi adalah x = 3. 
Contoh 7 : 
Jika a2 = 7b + 1945 dan b2 = 7a + 1945 dengan a dan b adalah bilangan real berbeda, maka nilai dari ab adalah ⋅⋅⋅⋅⋅ 
Solusi : 
a2 = 7b + 1945 dan b2 = 7a + 1945 
a2 − b2 = 7(b − a) 
Karena a ≠ b maka a + b = −7 
a2 + b2 = 7(b + a) + 3890 
(a + b)2 − 2ab = 7(a + b) + 3890 
49 − 2ab = −49 + 3890 
ab = −1896 
Contoh 8 : 
a, b, c dan d adalah bilangan real tak nol yang memenuhi 
a2 + b2 = 1 
c2 + d2 = 1 
ac + bd = 0 
Buktikan bahwa ab + cd = 0. 
Solusi : 
Karena ac + bd = 0 maka ba = −cd ⋅⋅⋅⋅⋅⋅⋅⋅⋅⋅⋅⋅⋅⋅⋅⋅⋅⋅⋅⋅⋅⋅⋅⋅⋅⋅⋅ (1) 
Misalkan ba = −cd = k maka 
a = bk dan d = −ck 
a2 + b2 = 1 
b2(k2 + 1) = 1 
k2 + 1 = 21b ⋅⋅⋅⋅⋅⋅⋅⋅⋅⋅⋅⋅⋅⋅⋅⋅⋅⋅ (2) 
c2 + d2 = 1 
c2(k2 + 1) = 1 
Subtitusikan persamaan (2). 
b2 = c2 
ab + cd = bk ⋅ b + c (−ck) 
ab + cd = b2k − c2k 
ab + cd = (b2 − c2)k 
Karena b2 = c2 maka ab + cd = (b2 − c2)k = 0 
Jadi, terbukti bahwa ab + cd = 0.
Pembinaan Olimpiade Matematika 
Eddy Hermanto, ST Aljabar 
4 
LATIHAN 1 : 
1. Jika baba2243− + = 5 maka tentukan nilai dari abba222+ 
2. (AIME 1986) Tentukan nilai dari ()()()()765765765765++−+−−+++. 
3. Jika 183=+++yxyx dan 152=−−−yxyx, maka x⋅y = ⋅⋅⋅⋅⋅ 
4. Tentukan nilai X yang memenuhi ()()⎟⎠⎞ ⎜⎝⎛ −++⎟⎠⎞ ⎜⎝⎛ +−=53535353X 
5. Jika diketahui bahwa 4820142+−yy + 1520142−−yy = 9, maka tentukan nilai dari 4820142+−yy − 1520142−−yy. 
6. (OSP 2006) Himpunan semua x yang memenuhi (x − 1)3 + (x − 2)2 = 1 adalah ⋅⋅⋅⋅⋅ 
7. (Canadian MO 1992) Selesaikan persamaan x2 + ()221+xx = 3. 
8. (OSP 2007) Tentukan semua bilangan real x yang memenuhi x4 − 4x3 + 5x2 − 4x + 1 = 0 
9. (AIME 1983) w dan z adalah bilangan kompleks yang memenuhi w2 + z2 = 7 dan w3 + z3 = 10. Apakah nilai terbesar yang mungkin dari w + z ? 
10. (Baltic Way 1999) Tentukan semua bilangan real a, b, c dan d yang memenuhi sistem persamaan berikut : 
abc + ab + bc + ca + a + b + c = 1 
bcd + bc + cd + db + b + c + d = 9 
cda + cd + da + ac + c + d + a = 9 
dab + da + ab + bd + d + b + a = 9 
11. (AIME 2000) Tentukan tepat kedua akar real persamaan 2000x6 + 100x5 + 10x3 + x − 2 = 0. 
12. (AIME 1987) Tentukan nilai dari ()()()()() ()()()()( ) 32452324403242832416324432458324463243432422324104444444444+++++ +++++. 
13. (Baltic Way 1993 Mathematical Team Contest) Tentukan semua bilangan bulat n yang memenuhi nn−−+−+46252254625225 adalah bilangan bulat 
14. (Canadian MO 1998) Tentukan penyelesaian x real yang memenuhi persamaan : 
x = xx1− + x1 1− 
15. (AIME 1990) Bilangan real a, b, x dan y memenuhi ax + by = 3, ax2 + by2 = 7, ax3 + by3 = 16 dan ax4 + by4 = 42. Tentukan nilai dari ax5 + by5. 
16. (OSN 2003 SMP/MTs) Diketahui a + b + c = 0. Tunjukkan bahwa a3 + b3 + c3 = 3abc
Pembinaan Olimpiade Matematika 
Eddy Hermanto, ST Aljabar 
5 
2. BARISAN DAN DERET 
1, 2, 3, 4, 5, ⋅⋅⋅ dikatakan sebagai barisan karena mempunyai suatu pola tertentu dengan rumus suku ke- n adalah n. 
1 + 2 + 3 + 4 + ⋅⋅⋅⋅ disebut sebagai deret. 
Ada beberapa barisan dan deret yang akan dibahas. 
A. Barisan dan Deret Aritmatika 
1. Pengertian, rumus suku ke-n dan rumus Jumlah n suku pertama 
Barisan aritmatika adalah barisan yang setiap dua suku berurutan memiliki selisih yang konstan. a, a + b, a + 2b, a + 3b ⋅⋅⋅ adalah barisan aritmatika dengan suku pertama = a dan beda = b. 
Suku ke-n, Un, dirumuskan dengan : 
Un = a + (n − 1)b 
Jumlah n bilangan pertama, Sn, dirumuskan dengan 
Sn = 2n(2a + (n − 1)b) = 2n(a + Un) 
Contoh 9 : 
Diketahui barisan 2, 5, 8, 11, ⋅⋅⋅. Tentukan suku ke-10 dan jumlah 4 suku pertama. 
Solusi : 
2, 5, 8, 11, ⋅⋅⋅ adalah barisan aritmatika dengan suku pertama 2 dan beda 3. 
Suku ke-10, U10 = 2 + (10 − 1) ⋅ 3 = 29 
Jumlah 4 suku pertama = ()3)14(2224 ⋅−+⋅⋅ = 26 
Contoh 10 : 
Sebuah barisan jumlah n buah suku pertama dirumuskan dengan Sn = 3n2 − 15n, maka U3 = ⋅⋅⋅⋅ 
Solusi : 
Perhatikan bahwa jika kita mengurangkan jumlah n suku pertama, Sn dengan jumlah n − 1 suku pertama, Sn-1, maka akan didapatkan suku ke-n, Un. 
Jadi, Un = Sn − Sn-1. 
Un = (3n2 − 15n) − (3(n − 1)2 − 15(n − 1)) 
Un = 3n2 − 15n − 3n2 + 6n − 3 + 15n − 15 = 6n − 18 
Maka U3 = 6(3) − 18 = 0 
Cara lain adalah dengan langsung menghitung U3 = S3 − S2. 
Contoh 11 : 
Nilai dariΣ= =+ nkk1)32(LL 
Solusi : 
Jika nilai k dari 1 sampai n dijalankan didapat 
(Σ= + nkk132 ) = 5 + 7 + 9 + ⋅⋅⋅ + (2n + 3) merupakan deret aritmatika dengan a = 5 serta b = 2. 
Sn = ()()()21522−+nn 
Jadi, = n(Σ= + nkk132 ) 2 + 4n
Pembinaan Olimpiade Matematika 
Eddy Hermanto, ST Aljabar 
6 
Contoh 12 : 
(OSK 2006) Pada sebuah barisan aritmatika, nilai suku ke-25 tiga kali nilai suku ke-5. Suku yang bernilai dua kali nilai suku pertama adalah suku ke ⋅⋅⋅⋅⋅⋅ 
Solusi : 
u25 = 3(u5), maka a + 24b = 3(a + 4b) sehingga a = 6b 
un = a + (n − 1)b = 2u1 = 2a 
6b + (n − 1)b = 2(6b) 
n = 7 
Suku tersebut adalah suku ke-7 
Contoh 13 : 
Sisi-sisi sebuah segitiga siku-siku membentuk barisan aritmatika. Jika sisi hipotenusa sama dengan 20, maka keliling segitiga tersebut adalah ⋅⋅⋅⋅ 
Solusi : 
Karena sisi terpanjang segitiga sama dengan 20 dan membentuk barisan aritmatika maka sisi-sisi segitiga tersebut dapat dimisalkan dengan 20, 20 − x dan 20 − 2x dengan x adalah bilangan positif. 
Karena ketiga sisi membentuk segitiga siku-siku maka 
(20 − 2x)2 + (20 − x)2 = 202 
400 − 80x + 4x2 + 400 − 40x + x2 = 400 
5x2 − 120x + 400 = 0 
(x − 4)(x − 20) = 0 
x = 4 atau x = 20 
Jika x = 20 maka sisi-sisi segitiga tersebut adalah 20, 0 dan −20 yang tidak mungkin merupakan sisi-sisi segitiga. 
Jika x = 4 maka sisi-sisi segitiga tersebut adalah 20, 16 dan 12 yang membentuk sisi-sisi segitiga siku-siku. 
Jadi, keliling segitiga tersebut = 20 + 16 + 12 = 48. 
2. Suku Tengah 
Misalkan Ut menyatakan suku tengah dari suatu barisan aritmatika maka : 21nUUtU+= 
dengan n merupakan bilangan ganjil 
Contoh 14 : 
Diketahui 3, ⋅⋅⋅, 13, 15, ⋅⋅⋅ adalah barisan aritmatika. Tentukan suku tengah barisan tersebut. 
Solusi : 
3, ⋅⋅⋅, 13, 15 adalah barisan aritmatika. Maka U1 = a = 3 dan Un = 15. 
Maka suku tengah, Ut = 21(3 + 15) = 9 
3. Sisipan 
Misalkan setiap dua bilangan berurutan pada barisan aritmatika disisipi k buah bilangan namun tetap membentuk barisan aritmatika. Maka beda barisan tersebut akan memiliki perubahan dengan suku pertama tetap. 
Misalkan bB = beda barisan yang baru dan bL = beda barisan yang lama. Hubungan keduanya adalah 
bB = 1+kbL
Pembinaan Olimpiade Matematika 
Eddy Hermanto, ST Aljabar 
7 
Contoh 15 : 
Pada setiap dua bilangan berurutan dari barisan 2, 12, 22, 32, 42. ⋅⋅⋅⋅ disisipi sebanyak 4 bilangan. Tentukan suku ke-100 dari barisan yang baru. 
Solusi : 
Beda barisan yang baru adalah bB = 14101++=kbL = 2 
Suku pertama, a = 2. 
U100 = a + 99bB = 2 + 99 ⋅ 2 = 200 
Suku ke-100 = 200. 
Jadi, suku ke-100 barisan tersebut adalah 200. 
4. Barisan Aritmatika Bertingkat 
Misalkan ada barisan u1, u2, u3, ⋅⋅⋅, un bukanlah merupakan barisan aitmatika sebab un − un-1 tidak konstan. Tetapi apabila diambil D1(n) = un − un-1 lalu D2(n) = D1(n) − D1(n − 1) dan seterusnya sampai pada suatu saat Dk(n) − Dk(n − 1) bernilai konstan. Maka kita dapat mengambil kesimpulan bahwa rumus jumlah n suku pertama, Sn, barisan tersebut merupakan polinomial pangkat n. 
Contoh 16 : 
Diketahui barisan 1, 3, 6, 10, 15, 21, ⋅⋅⋅. Tentukan rumus jumlah n suku pertama, Sn. 
Solusi : 
Kalau diperhatikan, barisan 1, 3, 6, 10, 15, 21, ⋅⋅⋅ bukanlah barisan aritmatika. Tetapi rumus suku ke-n barisan tersebut ternyata merupakan rumus jumlah n suku pertama dari barisan 1, 2, 3, ⋅⋅⋅, n yang merupakan barisan aritmatika. 
Maka kita dapat menyelesaikan soal tersebut dengan menganggapnya merupakan barisan aritmatika bertingkat. 
n 
S(n) 
D1(n) = S(n) – S(n − 1) 
D2(n) = D1(n) − D1(n − 1) 
D3(n) = D2(n) − D2(n − 1) 
1 
1 
2 
4 
3 
3 
10 
6 
3 
4 
20 
10 
4 
1 
5 
35 
15 
5 
1 
Karena D3(n) konstan maka dapat diambil kesimpulan bahwa rumus Sn merupakan polinomial pangkat 3. 
Misalkan S(n) = an3 + bn2 + cn + d. 
n 
S(n) 
D1(n) = S(n) – S(n − 1) 
D2(n) = D1(n) − D1(n − 1) 
D3(n) = D2(n) − D2(n − 1) 
1 
a+b+c+d 
2 
8a+4b+2c+d 
7a+3b+c 
3 
27a+9b+3c+d 
19a+5b+c 
12a+2b 
4 
64a+16b+4c+d 
37a+7b+c 
18a+2b 
6a 
5 
125a+25b+5c+d 
61a+9b+c 
24a+2b 
6a 
Dari kedua tabel didapat bahwa : 
6a = 1 ⋅⋅⋅⋅⋅⋅⋅⋅⋅⋅⋅⋅⋅⋅⋅⋅⋅⋅⋅⋅⋅⋅⋅⋅⋅⋅⋅⋅⋅⋅⋅ (1) 
12a + 2b = 3 ⋅⋅⋅⋅⋅⋅⋅⋅⋅⋅⋅⋅⋅⋅⋅⋅⋅⋅⋅⋅⋅⋅⋅⋅⋅⋅⋅⋅⋅⋅⋅⋅⋅⋅⋅⋅ (2) 
7a + 3b + c = 3 ⋅⋅⋅⋅⋅⋅⋅⋅⋅⋅⋅⋅⋅⋅⋅⋅⋅⋅⋅⋅⋅⋅⋅⋅⋅⋅⋅⋅⋅⋅⋅⋅⋅⋅⋅⋅⋅⋅⋅⋅ (3) 
a + b + c + d = 1 ⋅⋅⋅⋅⋅⋅⋅⋅⋅⋅⋅⋅⋅⋅⋅⋅⋅⋅⋅⋅⋅⋅⋅⋅⋅⋅⋅⋅⋅⋅⋅⋅⋅⋅⋅⋅⋅⋅⋅⋅⋅⋅⋅⋅ (4) 61=a Dari pers (1) didapat
Pembinaan Olimpiade Matematika 
Eddy Hermanto, ST Aljabar 
8 
b2122 3 = = − Dari pers (2) didapat Dari pers (3) didapat ()()31697182161373==−−=−−c Dari pers (4) didapat 0162316312161==−−−=−−−d Maka rumus jumlah n 31 n = ()() 621++nnn 61n3 + 21nsuku pertama, S(n) = 2 + 
B. arisan dan Deret Geometri 
e-n dan rumus Jumlah n suku pertama 
iliki perbandingan yang 
Un = a ⋅ rn-1 
uska 
B 
1. Pengertian, rumus suku kBarisan geometri adalah 
barisan yang setiap dua suku berurutan mem2 
konstan. Misalkan a, ar, ar, ⋅⋅⋅ adalah barisan geometri dengan suku pertama = a dan rasio = r maka : Suku ke-n, Un, dirumuskan dengan : 
Jumlah n bilangan pertama, Sn, dirum 
n dengan 
Sn = ( 
) 11− −ra 
r 
ontoh 17 : 
risan 2, 6, 18, 54, ⋅⋅⋅ . Tentukan suku ke-5 dan jumlah 4 suku pertama barisan 
⋅⋅⋅ adalah contoh barisan geometri dengan suku pertama 2 dan rasio 3. 
5-1 
n 
C 
Diketahui batersebut. 
Solusi : 
2, 6, 18, 54, 
Suku ke-5, U5 = 2 ⋅ 3 = 162 
Jumlah 4 suku pertama = () 13132− ⋅4− = 80 
ontoh 18 : 
geometri diketahui U8 = 36 dan S7 = 52, maka S8 = ⋅⋅⋅⋅⋅ 
7 = 52 
tika maupun geometri berlaku Sn − Sn−1 = Un. 
ontoh 19 : 
n membentuk deret geometri dengan jumlah 65. Jika suku ke-3 dikurangi 20 
bilangan-bilangan tersebut adalah a, ar dan ar2. Maka 
2 
C 
Pada barisan 
Solusi : 
U 
8 = 36 dan S 
Pada barisan aritma 
S8 − S7 = U8S = 52 + 36 = 88. 
8 
C 
Tiga bilanga 
terbentuklah deret aritmatika, maka rasio barisan tersebut adalah ⋅⋅⋅⋅ Solusi : 
Misalkan 
a, ar dan ar − 20 membentuk barisan aritmatika sehingga 2 
2ar = a + ar − 202 ⋅⋅⋅⋅⋅⋅⋅⋅⋅⋅⋅⋅⋅⋅⋅⋅⋅ (1) 
2ar + 20 = a + ar 2⋅⋅ 
a + ar + ar = 65 ⋅⋅⋅⋅⋅⋅⋅⋅⋅⋅⋅⋅⋅⋅⋅⋅ (2) 2ar + 20 + ar = 65 
ar = 15 
⋅⋅⋅⋅⋅⋅⋅⋅⋅⋅⋅⋅⋅⋅⋅⋅⋅⋅⋅⋅⋅ (3)
Pembinaan Olimpiade Matematika 
Eddy Hermanto, ST Aljabar 
9 
Subtitusikan persamaan (3) ke (1) didapat 
0 = a + 15r ⋅⋅⋅⋅⋅⋅⋅⋅⋅⋅⋅⋅⋅⋅⋅⋅⋅⋅ (4) 
5 
Subtitusikan persamaan (4) ke (3). 
(50 − 15r)r = 15 
10r − 3r2 = 3 
(3r − 1)(r − 3) = 0 
r = 
31 atau r = 3 
Jika r = 3 maka k1 etiga bilangan tersebut adalah 45, 15 dan 5 yang membentuk barisan geometri 
− dan 45, 15, 15 yang membentuk barisan aritmatika. Jika r = 3 maka ketiga bilangan tersebut adalah 5, 15 dan 45 yang membentuk barisan geometri dan 5, 15, 25 yang membentuk barisan aritmatika. 
Jadi, rasio barisan tersebut adalah 31 atau 3. 
2. 
isalkan Ut menyatakan suku tengah dari suatu barisan geometri maka : 
2 
iketahui 2, 6, 18, 54, 162, ⋅⋅⋅⋅ adalah barisan geometri. Tentukan suku tengah dari barisan 
, 6, 18, 54, 162 adalah barisan geometri. Maka U1 = a = 2 dan Un = 162. 
Suku Tengah 
M 
nt1 dengan n merupakan bilangan ganjil 
UUU⋅= 
Contoh 20 : 
D 
tersebut. 
Solusi : 
2 
Maka suk 
u tengah, 181622=⋅=tU 
3. 
Misalkan setiap dua bilangan berurutan pada barisan geometri disisipi k buah bilangan namun 
embentuk barisan geometri. Maka rasio barisan tersebut akan memiliki perubahan 
Sisipan 
tetap m 
dengan suku pertama tetap. Misalkan rB = rasio barisan yang baru dan rL = rasio barisan yang lama. Hubungan keduanya adalah 
1+=kLBrr 
Contoh 21 : 
ada setiap dua bilangan berurutan dari barisan 2, 32, 512, 8192, ⋅⋅⋅⋅ disisipi sebanyak 3 
tukan suku ke-7 dari barisan yang baru. 
ng baru, 
P 
bilangan. Ten 
Solusi : 
R 
asio ya 21641===+kLBrr . 
a, a = 2. 
128 
4. tak hingga 
Suku pertamU7 = ar6 = (2)(26) = 
Suku ke-7 = 128. 
Barisan geometri 
D 
ari persamaan Sn = 
() 1−r jika n a rn −1 
Æ ∞ maka S∞ = ra−1 dengan syarat −1 < r < 1. 
an rumus jumlah dari suatu barisan tak hingga dengan suatu syarat Rumus tersebut merupaktertentu.
Pembinaan Olimpiade Matematika 
Eddy Hermanto, ST Aljabar 
10 
Contoh 22 : 
entukan nilai dari 2 + 1 + 
T21 + 41 + ⋅⋅⋅ 
ak hingga dengan a = 2 dan r = ½ 
+ 1 + ½ + ¼ + ⋅⋅⋅ = S∞ = 
Solusi : Persoalan di atas termasuk barisan geometri t 
2ra−1 . 
2 + 1 + ½ + ¼ + ⋅⋅⋅ = 2112− = 4. Maka nilai dari 2 + 1 + 21+ 41 + ⋅⋅⋅ = 4. 
mlah suatu deret geometri tak hingga adalah 6 dan jumlah dari suku-suku yang bernomor 
anjil adalah 4. Suku ke-6 barisan tersebut adalah ⋅⋅⋅⋅ 
sebut adalah a dan rasio r. 
+ ar + ar2 + ⋅⋅⋅ = 
Contoh 23 : 
Ju 
g 
Solusi : Misalkan suku pertama barisan geometri tak hingga ter 
ara−1 = 6 ⋅⋅⋅⋅⋅⋅⋅⋅⋅⋅⋅⋅⋅⋅⋅⋅⋅⋅⋅⋅⋅⋅ (1) 
dengan suku 
⋅⋅⋅ 
Suku-suku yang ganjil adalah a, ar2, ar4, ⋅⋅⋅ yang membentuk barisan tak hinggao r2pertama a dan rasi. 246 
a + ar + ar + ar + = 21r− = 4 ⋅⋅⋅ (2) Subtitusikan persamaan a⋅⋅⋅⋅⋅⋅⋅⋅⋅⋅⋅⋅⋅⋅⋅⋅⋅⋅⋅ 
(1) ke ()()rra+−111 = 4 sehingga 
r+11 = 32 3 = 2 + 2r 
r = 2 
1 
maan di atas ke persamaan (1) didapat 
a ku ke-6 = U = ar5 = 
Subtitusikan persaa = 3 Mak su 
632 
3 . 
C. Bari Bentuk Tak Hingga 
Suatu barisan tidak harus masuk ke dalam salah satu dari dua bentuk di atas. Sebagai contoh adalah 
arisan yang berbentuk 1, 1, 2, 3, 5, 8, 13, 21, ⋅⋅⋅ yang merupakan penjumlahan dari dua bilangan 
ditanyakan memerlukan pengetahuan terhadap 
san dan Deret Lainnya serta 
b 
sebelumnya. Untuk menyelesaikan persoalan yang 
pola dari barisan tersebut. Beberapa contoh rumus deret lainnya : 12 + 22 + 32 + ⋅⋅⋅ + n2 = ()( 
) 6121+nn n+ 
13 + 23 + 33 + ⋅⋅⋅ + n3 = ()()21+nn 
2 
Berikut ini dibahas bentuk-bentuk tak hingga yang dapat diselesaikan dengan memisalkan dengan 
itunglah nilai dari 
suatu variabel. 
Contoh 24 : 
HL2222
Pembinaan Olimpiade Matematika 
Eddy Hermanto, ST Aljabar 
11 
Solusi : 
Misalkan L2222 = X. 
Kuadratkan kedua ruas. Pada ruas kiri akan didapatkan bentuk tak hingga semula. Maka 
2X = X2 
X(X − 2) = 0 
Karena L2222 tidak mungkin sama dengan 0 maka L2222 = 2. 
Jadi, L2222 = 
: 
itunglah bentuk tak hingga berikut, 
2 Contoh 25 
H M323232+ + + . 
Solusi : 
isalkan 
MM323232+ + + = X maka 
X3 2 + = X 
X2 − 2X − 3 = 0 
(X − 3)(X + 1) = 0 
Karena M32 
32+ + 32+ tidak mungkin sama dengan −1 maka 
M323232+ + + = 3. 
Jadi, 
M323232+ + + = 3 
D. ip Teleskopik 
rinsip teleskopik banyak digunakan untuk menyederhanakan suatu deret. Ada dua bentuk umum 
dan perkalian sebagai berikut : 
PrinsP 
y 
ang dikenal, yaitu penjumlahan 
a. ()()()()()()111134231211aaaaaaaaaaaaaannnnniii−=−+−++−+−+−=−++− = +ΣL nb. 111134231211aaaaaaaaaaaaaannnnnniii++ −= +=⋅⋅⋅⋅⋅=ΠL 
Contoh 26 : 
()− ( − )( − )L( − )( − )( + )( + )( + )L( + )( + ) =L 2006 
1 
2004 
1 
6 
1 
4 
1 
2 
1 
2005 
1 
2003 
1 
7 1 
5 
1 
3 
1 1 1 1 1 1 1 1 1 1 
1
Pembinaan Olimpiade Matematika 
Eddy Hermanto, ST Aljabar 
12 
Solusi : 
isal S = 
M ()()()()()()()()()()20061200416141212005120031 71 51311111111111+++++−−−−−LL 
S 
= 20062007674523 200520047653 
2 ⋅ 4 ⋅ ⋅L⋅ ⋅ ⋅ ⋅ ⋅L⋅ 
S = 20062007200420052005200467764554232 3 ⋅ ⋅ ⋅ ⋅L⋅ ⋅ ⋅ ⋅⋅ S = 20062007 Cont27 oh : 
entukan nila 
T 
i 211⋅ 
+ 321⋅ 
+ 431⋅ 
+ 541⋅ 
+ ⋅⋅⋅ + 200620051⋅ 
. 
prinsip ik. 
Solusi : Soal di atas merupakan contoh penerapan teleskop2 
1⋅1 
= 2111 − ; 321⋅ 
= 3121− ; 431⋅ 
= 4131− ; ⋅⋅⋅⋅⋅⋅⋅⋅⋅⋅ ; 200620051⋅ 
= 2006120051− 
21⋅ 
1 + 32⋅1 
+ 43⋅ 1 
+ 54⋅ 1 
+ ⋅⋅⋅ + 20062005⋅ 1 
= ()()()()()20061200515141433221 
1 − 1 + 1 − 1 + 1 − 1 + − +L+ − 
211⋅ 
+ 321⋅ 
+ 43⋅ + 1541⋅ 
⋅⋅⋅ + +2002005⋅ 6 = 1 − 1 2006 1 
Jadi, 211⋅ + 321⋅ + 431⋅ + 541⋅ + ⋅⋅⋅ + 2005⋅ 20 06 
1 = 20062005 
anadian MO 1997) Buktikan bahwa 
Contoh 28 : 
(C19991 < 1998199765 43 21⋅⋅⋅⋅L < 441 . 
Solusi : Misal P = 1998199765 43 21⋅⋅⋅⋅L dan Q = 19991998765432 
⋅ ⋅ ⋅ ⋅L 
= 
PQ 1998199765 43 
2 
1 ⋅ ⋅ ⋅L⋅ 
⋅19991998765432 ⋅⋅⋅⋅L = 19991 Jelas bahwa P < Q 
hingga P2 < P2 < PQ se19991 < 2441. Maka P < 441 
P > 1999753⋅⋅⋅⋅L = 3 5 1997 1 19991 Maka didapat 19991 < P < 44 
1 
1999 
1 < 1998 
1997 
6 5 
4 3 
2 
1 ⋅ ⋅ ⋅L⋅ < 44 
1 (terbukti) 
LATIHAN 2 : 
1. ebuah deret aritmatika terdiri dari n suku (ganjil). Jumlah semua sukunya 260, besar suku 
ngahnya 20, serta beda deret tersebut a U6 = ⋅⋅⋅⋅ 
S 
te 
dalah 3. Maka 
2. Perhatikan barisan bilangan 500, 465, 430, 395, ⋅⋅⋅. Suku negatifnya yang pertama adalah ⋅⋅⋅⋅⋅⋅ 11711 
()Σ=+()LL=+Σ 
3. Diketahui 
=17724iik dan Σ 
= 
=114iik, maka nilai 
= 
deret aritmatika berlaku u2 + u5 + Maka S10 = ⋅⋅⋅⋅⋅ 
5. (OSK 2009) Jika xk+1 = xk + 
824iik 4. Pada suatu u6 + u9 = 40.21 untuk k = 1, 2, ⋅⋅⋅ dan x1 = 1 maka x1 + x2 + ⋅⋅⋅ + x400 = ⋅⋅⋅⋅⋅⋅⋅
Pembinaan Olimpiade Matematika 
Eddy Hermanto, ST Aljabar 
13 
32006 dan 2006 6. (OSP 2006) Hasil penjumlahan semua bilangan bulat di antara adalah ⋅⋅ 
. (OSK 2006) Diketahui a + (a + 1) + (a + 2) + ⋅⋅⋅ + 50 = 1139. Jika a bilangan positif, maka a = ⋅⋅⋅⋅⋅ 
1 2 3 98 n+1 n n = 1, ⋅⋅⋅, 97 dan mempunyai 
0 1 2 n 
11. dan x − 1 adalah tiga suku pertama deret geometri tak hingga, maka jumlah 
78. (AIME 1984) Barisan a, a, a, ⋅⋅⋅, a memenuhi a = a + 1 untuk 2, 3, 
jumlah sama dengan 137. Tentukan nilai dari a2 + a4 + a6 + ⋅⋅⋅ + a98. 
9. Misalkan un adalah suku ke-n dari suatu barisan aritmatika. Jika uk = t dan ut = k maka tentukan nilai dari suku ke-(k + t). 
10.(OSK 2004) Agar bilangan 2 + 2 + 2 + ⋅⋅⋅ + 2 sedekat mungkin kepada 2004, haruslah n = ⋅ Jika 9 − 7x, 5x − 6 
suku-sukunya adalah ⋅⋅⋅ 
12. Pada suatu deret tak hingga, suku-suku yang bernomor ganjil berjumlah 9/4 sedangkan suku-suku yang bernomor genap berjumlah 3/4 , maka suku pertamanya adalah ⋅⋅⋅⋅⋅ 
13. Batas-batas nilai a supaya deret geometri tak berhingga dengan suku pertama a konvergen dengan jumlah 2 adalah ⋅⋅⋅⋅⋅ 
14. (OSP 2006) Afkar memilih suku-suku barisan geometri takhingga 1, 21, 41, 81 , ⋅⋅⋅ untuk membuat barisan geometri tak71 
hingga baru yang jumlahnya . Tiga suku pertama pilihan Afkar adalah ⋅⋅⋅⋅⋅ 
15. Tentukan jumlah dari LL+−+−+−494278749432 4 
16. Tiga buah bilangan merupakan barisan aritmatika. Bila suku tengahnya dikurangi 5, maka terbentuk 
dengan rasio sama dengan 2. umlah barisan aritmatika itu adalah ⋅⋅⋅⋅⋅ ⋅⋅ 
18. gan 
a dengan 1, maka nilai a1 
20. agian Pertama) Diketahui 0 < a < b < c < d adalah bilangan bulat yang memenuhi a, b, c 
0 
suatu barisan geometri J 
17. Tentukan rumus jumlah n suku pertama dari barisan 4, 10, 20, 35, 56, ⋅⋅⋅ (AIME 1992) Misalkan A adalah barisan a1, a2, a3, ⋅⋅⋅ dengan a19 = a92 = 0 dan ΔA didenisikan den 
barisan a2 − a1, a3 − a2, a4 − a3, ⋅⋅⋅. Jika semua suku-suku barisan Δ(ΔA) samadalah ⋅⋅⋅⋅⋅⋅ 
19. (MATNC 2001) Tentukan jumlah 100 bilangan asli pertama yang bukan bilangan kuadrat sempurna. (AIME 2003 B 
membentuk barisan aritmatika sedangkan b, c, d membentuk barisan geometri. Jika d − a = 3maka tentukan nilai dari a + b + c + d. 
21. (OSK 2009) Bilangan bulat positif terkecil n dengan n > 2009 sehingga nn3333321++++ L 
22. (AIME 1985) Barisan bilangan bulat a1, a2, a3, ⋅⋅⋅ memenuhi a = an+1 − an untuk n > 0. Jumlah 1492 
985 bilangan pertama adalah 1492. Tentukan jumlah 
merupakan bilangan bulat adalah ⋅⋅⋅⋅⋅⋅⋅⋅⋅ n+2 
bilangan pertama adalah 1985 dan jumlah 12001 bilangan pertama.
Pembinaan Olimpiade Matematika 
Eddy Hermanto, ST Aljabar 
14 
23. Nilai x yang memenuhi persamaan : ...444.....+++=xxxxxx 
adalah ⋅⋅⋅⋅⋅⋅⋅ 
24. Hitunglah nilai dari 
M43 
43+ 
3 4 
5 
+ 
+ 
25. (OSK 2006/AIME 1990) Barisan 2, 3, 5, 6, 7, 10, 11, ⋅⋅⋅ terdiri dari semua bilangan asli yang bukan 
kuadrat atau pangkat tiga bilangan bulat. Suku ke-250 barisan tersebut adalah ⋅⋅⋅⋅⋅ 
26. HSME 1996) Barisan 1, 2, 1, 2, 2, 1, 2, 2, 2, 1, 2, 2, 2, 2, 1, 2, 2, 2, 2, 2, 1, 2, ⋅⋅⋅ memiliki blok 
27. Misalkan f adalah adalah fungsi yang memenuhi f(n) = f(n − 1) + 
6− 
(A 
angka 1 yang berisi n buah angka 2 pada blok ke-n. Tentukan jumlah 1234 bilangan pertama. 
2007Jika f(0) = 1945 maka tentukan f(2007). 
n untuk setiap n bilangan asli. 
28. (NHAC 1997-1998 Second Round) Tentukan nilai dari 199819971996432321xxxxxx 
1 1 + 1 ++ L 
29. 
. (OSK 2003) Berapakah hasil perkalian ()()()()()22222432111 2003 
1 
2002 
1 1 1 1− 1 1− −−−L 
30. Tentukan jumlah dari : 100991431321211++++++++L 
x2, x3, ⋅⋅⋅ memenuhi 
31. (AIME 2002) Barisan x1, kkkx+=21 . Jika terdapat bilangan berurutan sehingga 
xm + xm+1 + ⋅⋅⋅ + xn = 291 , maka tentukan semua pasangan (m, n) yang memenuhi. 
uhi a1 = 
a = a −1 − a + a − a . tentukan nilai dari a + a + a . 
3. (AIME 1998) Barisan 1000, n, 1000 − n, n − (1000 − n), (1000 − n) − (n − (1000 − n), ⋅⋅⋅ dengan n 
arisan tersebut negatif. Jadi, untuk 
n = 100 maka barisan tersebut memiliki panjang 3. Tentukan n sehingga panjang barisan tersebut 
34. 
32. (AIME 2001) Barisan a1, a2, a3, a4, ⋅⋅⋅ memen211, a2 = 375, a3 = 420 dan a4 = 523 serta nnn−2n−3n−4531753975 
3 
bilangan bulat berakhir ketika bilangan negatif pertama muncul. Sebagai contoh untuk n = 100 maka barisan tersebut adalah 1000, 100, 900, −800. Suku ke-4 b 
maksimal. (USAMTS 1999-2000 Round 4) Tentukan nilai dari S = 222223222000111++ + 111++ + ⋅⋅⋅ + 111++ 
1 2 1999 
angan 
1133+ − kk , k = 2, 3, 4, ⋅⋅⋅, 100 lebih dari 32 
35. (Baltic Way 1992) Buktikan bahwa hasil kali 99 bil .
Pembinaan Olimpiade Matematika 
Eddy Hermanto, ST Aljabar 
15 
3. 
Misalkan diketahui fungsi y = f(x) = 
FUNGSI A. Pengertian xx231− + . 
k mencari nilai dari f(2) maka cukup mengganti x di ruas kanan dengan 2. 
Untu 
Jadi, f(2) = 
() ()22312− + = −3 
lam kelas adalah fungsi kuadrat, yaitu fungsi yang berbentuk y = 
meny n y maksimum adalah xp = 
Salah satu fungsi yang dibahas di da 
f(x) = ax2 + bx + c Nilai x yang ebabkaab2− 
Nilai y maksimum = ymaks = a(xp)2 + bxp + c atau ymaks = () aacb442−− Terkadang suatu fungsi tidak hanya memiliki satu variabel, tetapi dapat lebih dari satu variabel. 
f(1, 2) cukup mengganti x = 1 dan y = 2 
= 
bulat positif ke bilangan bulat positif dan 
idefinisikan dengan : f(ab) = b⋅f(a) + a⋅f(b). Jika f(10) = 19 ; f(12) = 52 dan f(15) = 26. Tentukan 
ilai dari f(8). 
) = 8f(15) + 15f(8) = 8 ⋅ 26 + 15f(8) = 208 + 15f(8) ⋅⋅⋅⋅⋅⋅⋅⋅⋅⋅⋅ (2) 
48 = 208 + 15f(8) 
) = 36 
isalkan f(x) adalah fungsi yang memenuhi 
) untuk setiap bilangan real x dan y maka f(x + y) = x + f(y) dan 
) f(0) = 2 
998) adalah ⋅⋅⋅⋅⋅ 
f(y) dan f(0) = 2 
x) = f(x + 0) = x + f(0) 
1998 + 1998) = −1998 + f(1998) 
i yang tidak terdefinisi untuk x = 0 dengan f(x) + 2f( 
Sebagai contoh adalah f(x,y) = xy + x2y + y3. Untuk mencaridari persamaan tersebut didapat f(1, 2) = 2 + 2 + 8 12. 
Contoh 29 : Misal f adalah suatu fungsi yang memetakan dari bilangan 
d 
n 
Solusi : f(120) = f(10 ⋅ 12) = 12f(10) + 10f(12) = 12 ⋅ 19 + 10 ⋅ 52 = 748 ⋅⋅⋅⋅⋅⋅⋅⋅⋅⋅⋅⋅ (1) f(120) = f(8 ⋅ 15 
7 
Jadi, f(8 
Contoh 30 : 
(AHSME 1998) M 
(a 
(b 
Nilai dari f(1 
Solusi : 
f(x + y) = x + 
Alternatif 1 : 
f( 
f(x) = x + 2 
Maka f(1998) = 2000 
Alternatif 2 : 
2 = f(0) = f(− 
Maka f(1998) = 2000 
Contoh 31 : 
Jika f(x) adalah fungsx1 
) = 3x. Tentukan f(x). 
Solusi : f(x) + 2f(x1 ) = 3x ⋅⋅⋅⋅⋅⋅⋅⋅⋅⋅⋅⋅⋅⋅⋅⋅⋅⋅⋅ (1) 
Jika y = x1 
maka
Pembinaan Olimpiade Matematika 
Eddy Hermanto, ST Aljabar 
16 
f(y1) + 2f(y) = y3 
sehingga f(x1 ) + 2f(x) = x3 
2f(x1 ) + 4f(x) = x6 
⋅⋅⋅⋅⋅⋅⋅⋅⋅⋅⋅⋅⋅⋅⋅⋅⋅⋅⋅⋅⋅⋅⋅⋅⋅ (2) 
r a n (2) dengan (1) didapat Kuangkan persmaa3f(x) = x − 3x 6 
Jadi, f(x) = xx22− 
B. osis 
ungsi komposisi merupakan gabungan lebih dari satu fungsi. 
isalkan diketahui fungsi f(x) dan g(x). Jika ingin mencari pemetaan suatu nilai terhadap fungsi f(x) 
jutkan terhadap fungsi g(x), maka akan digunakan fungsi komposisi. 
) ditulis sebagai (g(x) o f(x)). 
− 3x. Tentukan pemetaan x = 2 oleh fungsi f(x) dilanjutkan g(x). 
(11) = 7 − 3(11) = −26 
etaan x = 2 oleh fungsi f(x) dilanjutkan oleh g(x) menghasilkan nilai −26. 
gan memanfaatkan definisi fungsi komposisi. 
x + 5) = 7 − 3(3x + 5) = −8 − 9x 
−26. 
iketahui f(x) = x + 7 dan (fog)(x) = 5x + 3. Tentukan g(x). 
x + 3 = g(x) + 7 
− 4 
ika g(x − 5) = 7x + 3 maka tentukan g(x). 
ada alternatif 1 ini kita ubah variabel x pada ruas kanan ke dalam (x − 5). 
+ 3 = 7(x − 5) + 38 
+ 38 
y sehingga x = y + 5. 
aka g(y) = 7(y + 5) + 3 
Fungsi Kompi 
F 
M 
y ang hasilnya dilan 
Pemetaan terhadap fungsi f(x) yang dilanjutkan oleh fungsi g(x 
Didefinisikan (g(x)of(x)) = g(f(x)). 
Contoh 32 : 
Diketahui f(x) = 3x + 5 dan g(x) = 7 
Solusi : 
f(2) = 3(2) + 5 = 11 
g 
Jadi, pem 
Cara lain adalah den 
(g(x)of(x)) = g(f(x)) = g(3 
Untuk x = 2 maka nilai g(f(2)) = −8 − 9(2) = −26. 
Jadi, pemetaan x = 2 oleh fungsi f(x) dilanjutkan oleh g(x) adalah 
Contoh 33 : 
D 
Solusi : 
f(g(x)) = g(x) + 7 
5 
g(x) = 5x 
Contoh 34 : 
J 
Solusi : 
Alternatif 1 : 
P 
g(x − 5) = 7x 
Maka g(x) = 7x 
Alternatif 2 : 
Kita misalkan x − 5 = 
M 
g(y) = 7y + 38 
Maka g(x) = 7x + 38
Pembinaan Olimpiade Matematika 
Eddy Hermanto, ST Aljabar 
17 
Contoh 35 : 
Diketahui (gof)(x) = 2x2 + 5x − 5 dan f(x) = x − 1. Maka g(x) = ⋅⋅⋅⋅⋅⋅ 
(f(x)) = 2x2 + 5x − 5 
2 + 5x − 5 
− 2 + 5x − 5 
(x − 1) = 2(x − 1)2 + 9x − 7 
1)2 + 9(x − 1) + 2 
= y + 1 
(y) = 2(y + 1)2 + 5(y + 1) − 5 
+ 2 
iketahui (gof)(x) = 4x2 + 4x dan g(x) = x2 − 1 dan berlaku f(x) > 1 untuk x > 
Solusi : 
(gof)(x) = 2x2 + 5x − 5 
g 
g(x − 1) = 2x 
Alternatif 1 : 
g(x − 1) = 2(x − 1)2 + 4x 
g 
g(x − 1) = 2(x − 
Maka g(x) = 2x2 + 9x + 2 
Alternatif 2 : 
Misalkan y = x − 1 maka x 
g 
g(y) = 2y2 + 9y 
Maka g(x) = 2x2 + 9x + 2 
Contoh 36 : 
D 
−21 , maka f(x − 2) 
2 + 4x 
(x)) − 1 = 4x2 + 4x 
4x2 + 4x + 1 = (2x + 1)2 
uk x > 
adalah ⋅⋅⋅⋅ Solusi : g(f(x)) = 4x2 
(f 
(f(x))2 = 
Karena f(x) > 1 unt 
−21 maka 
= 2x − 3 
C. x) 
erdasarkan fungsi y = f(x) = 
f(x) = 2x + 1 f(x − 2) = 2(x − 2) + 1 f(x − 2) = 2x − 3 
Jadi, f(x − 2) 
Fungsi Invers dari y = f( 
Bxx231− + dari keterangan sebelumnya jika diketahui nilai x kita dengan 
agaimana caranya bila yang diketahui adalah nilai y dan kita diminta 
mudah mencari nilai y. Bmencari nilai x untuk nilai y tersebut ? Persoalan ini dapat diselesaikan apabila kita bisa mendapatkan fungsi inversnya yaitu x = f(y). Contoh 37 : Tentukan invers dari fungsi y = f(x) = 3x − 8 
Solusi : 
y = 3x − 8 
x 
= 3 
y+8 
gsi inversnya adalah f-1(x) = 
Didapat fun28+x
Pembinaan Olimpiade Matematika 
Eddy Hermanto, ST Aljabar 
18 
Contoh 38 : 
Tentukan invers dari fungsi y = f(x) = xx231− + . 
Solusi : Dari y = xx231− 
+ seh ga 
(2y + 1) = 3y − 1 
didapat 3y − 2yx = x + 1 ing 
x 
x = 12+y-113−y 
ersnya adalah f (x) = 
Didapat fungsi inv1213 
+ 
− 
x 
D. mp isi. 
isalkan f (x) dan g (x) berturut-turut menyatakan fungsi invers dari f(x) dan g(x). Maka 
o g)−1(x) = (g−1 o f−1)(x) 
= 
x Hubungan fungsi invers dengan fungsi koos−1−1 
M 
(f 
(g o f)−1(x) = (f−1 o g−1)(x) 
Contoh 39 : 
Jika f(x) = 5x + 3 dan g(x) xx− + 532 maka tentukan (f o g)−1(x). 
omposisi akan didapat 
= 
Solusi : Alternatif 1 : asarkan keterangan dalam pembahasan mengenai fungsi k 
Berd 
(f o g)(x) xx− + 5307 . 
ri (f o g)(x) tersebut adalah 
Maka invers da 
(f o g)−1(x) = 7305+ 
− 
ipelajari sebelumnya akan didapat 
(x) = 
xx 
Alternatif 2 : Berdasarkan apa yang d−1 
f53−x dan g− (x) = 1235+ − xx sehingga 
(g−1 o f−1)(x) = 
7+x−1 
5x−30 
LATIHAN 3 : 
. Jika f(x) = − + 3, maka f(x2) + (f(x))2 − 2f(x) = ⋅⋅⋅⋅⋅⋅⋅ 
2. Diketahui g(x) = x + 1 dan (gof)(x) = 3x2 + ⋅⋅⋅⋅⋅⋅⋅ 
. (OSK 2007) Misalkan f(x) = 2x - 1, dan g(x) = 
Jadi, didapat (f o g)(x) = (g 
−1−1 
o f)(x). 
1 
x 
4. Maka f(x) = 
3x . Jika f(g(x)) = 3, maka x = ⋅⋅⋅⋅⋅⋅⋅⋅⋅ 
4. iketahui (fog)(x) = 5x. Jika g(x) = 
D 151−x , maka f(x) = ⋅⋅⋅⋅⋅⋅⋅⋅ 
5. ungsi g(x) = x2 + 2x + 5 dan (f(g(x)) = 3x2 + 6x − 8, maka f(x) = ⋅⋅⋅⋅⋅⋅⋅⋅ 
6. ika f(x) = 2x + 1 ; g(x) = 5x + 3 dan h(x) = 7x, maka (fogoh)(x) = ⋅⋅⋅⋅ 
F 2 
J 
7. itentukan 
Dxaxxf − 
+ 
2 =1)(. Jika f−1(4) = 1, maka f(3) = ⋅⋅⋅⋅⋅⋅⋅
Pembinaan Olimpiade Matematika 
Eddy Hermanto, ST Aljabar 
19 
8. ika 
J 11)(+ −=xxxf dan , maka 12)(1−=−xxg()()=−xfg1o ⋅⋅⋅⋅⋅⋅ 
= 1 dan (fog)(x) x2 + = 9. Jika f(x) 2 dan berlaku ≥ 0 u 2 4 5 
− 
− + 
x 
x x g(x) ntuk x > 2, maka g(x − 3) = ⋅⋅⋅⋅⋅ 
10. SK 2003) Misalkan f suatu fungsi yang memenuhi 
(O()()xxfx21 f x 
1 =− + 
untuk setiap bilangan real x ≠ 0. Berapakah nilai f(2) ? 
11. (AHSME 1996) Sebuah fungsi f : Z Æ Z dan memenuhi 
n + 3 jika n ganjil f(n) = 2n jika n genap 
Misalka k ada f(f(f(k))) = 27. Tentukan penjumlahan digit-digit dari k. 
12. (OSP 20 4) Misalkan s ang memenuhi f(x) f(y) − f(xy) = x + y, untuk setiap bilangan 
14. n f adalah fungsi untuk semua bilangan bulat x dan y yang 
i dari f(3) sama dengan ⋅⋅⋅⋅⋅⋅ 
n bilangan ganjil dan 
lah0 febuah fungsi y 
bulat x dan y. Berapakah nilai f(2004) ? 
13. (OSK 2006) Jika f(xy) = f(x + y) dan f(7) = 7, maka f(49) = ⋅⋅⋅⋅ NHAC 1998-1999 Second Round) Misalka 
( 
memenuhi f(x + y) = f(x) + f(y) + 6xy + 1 dan f(−x) = f(x). Nila 
15. (OSP 2009) Suatu fungsi f : Z Æ Q mempunyai sifat ())(1)(11xfxfxf− +=+ untuk setiap x ∈ Z. Jika f(2) = 2, maka nilai fungsi f(2009) adalah ⋅⋅⋅⋅⋅ 16. (AIME 1988) Misalkan f(n) adalah kuadrat dari angka n. Misalkan juga f jumlah angka-snya. 
Tentukan nilai dari f (11). 
4. UK 
. Pengertian Suku Banyak 
Perhatikan bentuk-bentuk aljabar berikut : 
2 + 7 
u bar di atas disebut juga dengan suku banyak atau polinom dalam peubah 
erajat suatu sukubanyak dalam peubah x adalah pangkat tertinggi dari 
banyak tersebut. 
B. 
sebut. Pembagian f(x) oleh p(x) dapat ditulis sebagai berikut : 
f(x) = p(x) ⋅ g(x) + s(x) 
2(n) didefiniskan sebagai f(f(n)), f3(n) sebagai f(f(f(n))) dan seteru1998 
U BANYAK 
SA 
(i) x − 3x3 
(ii) 4x + 6x − 2x 432 
(iii) 2x − 7x + 8x + x − 5 54x3 − 8x2 + 3x − 4 
(iv)−2x + x + 7 
Bentk-bentuk alja 
(variabel) x. Yang dimaksud d 
peubah x yang termuat dalam suku 
Suku banyak pada (i) memiliki derajat 2 sedangkan suku banyak pada (ii), (iii) dan (iv) berturut- turut berderajat 3, 4 dan 5. 
Pembagian Suku Banyak Sebagaimana pembagian dalam bilangan, pembagian suku banyak pun memiliki kemiripan dengan pembagian pada bilangan ter 
dengan f(x) adalah suku banyak yang akan dibagi
Pembinaan Olimpiade Matematika 
Eddy Hermanto, ST Aljabar 
20 
p(x) adalah pembagi 
ah hasil bagi 
embagian bilangan, persyaratan s(x) adalah bahwa pangkat tertinggi (derajat) 
dari p(x). 
banyak pun mengikuti dalam bilangan. 
4x − 2 
arena f(x) berderajat 4 maka q(x) akan berderajat 2 sehingga q(x) = ax + bx + c 
fisen x4 dari f(x) sama dengan 4 maka koefisien x2 dari q(x) juga 4 sehingga a = 4. 
x2. Kurangkan 4x4 + 3x3 − 2x2 + x − 7 dengan 
maka koefisien x 
2) ⋅ (4x2 − 13x + 58) − 257x + 109. 
109. 
dapat dilakukan dengan cara horner. 
entukan hasil bagi dan sisanya jika f(x) = x + 2x + 3x − 5 dengan x − 2 
g(x) adal 
s(x) adalah sisa pembagian 
Sebagaimana dalam p 
dari s(x) harus kurang 
Cara pembagian dalam suku 
Contoh 40 : 
Tentukan hasil bagi dan sisanya jika 4x4 + 3x3 − 2x2 + x − 7 dibagi x2 + 
Solusi : 
f(x) = 4x4 + 3x3 − 2x2 + x − 7 = (x2 + 4x − 2) ⋅ q(x) + s(x) 2 
K 
Kare koe 
Kalikan 4x2 dengan (x2 + 4x − 2) didapat 4x4 + 16x3 − 8 
4x4 + 16x3 − 8x2 didapat −13x3 + 6x2 + x − 7. Karena koefisien x3 sama dengan −13 
dari q(x) sama dengan −13 sehingga b = −13. 
Kalikan −13x dengan (x2 + 4x − 2) didapat −13x3 − 52x2 + 26x. Kurangkan −13x3 + 6x2 + x − 7 dengan −13x4 − 52x2 + 26x didapat 58x2 − 25x − 7. Karena koefisien x2 sama dengan 58 maka konstanta dari q(x) sama dengan 58 sehingga c = 58. 
Kalikan 58 dengan (x2 + 4x − 2) didapat 58x2 + 232x − 116. Kurangkan 58x2 − 25x − 7 dengan 58x2 + 232x − 116 didapat −257x + 109. Jadi, 4x4 + 3x3 − 2x2 + x − 7 = (x2 + 4x − 
Maka pembagi dan sisa jika suku banyak 4x4 + 3x3 − 2x2 + x − 7 dibagi x2 + 4x − 2 berturut-turut adalah 4x2 − 13x + 58 dan − 257x + 
Contoh 40 merupakan pembagian suku banyak dengan cara pembagian bersusun. Apabila pembaginya linier maka pembagian juga 
Contoh 41 : 32 
T 
Solusi : 2282 
5321− 
2 
1 4 11 + 
Maka pembagian f(x) = x3 + 2x2 + 3x − 5 oleh x − 2 akan menghasilkan x2 + 4x + 11 dengan sisa 17. 
C. 
ari penjelasan sebelumnya telah kita dapatkan bahwa 
f(x) = p(x) ⋅ g(x) + s(x) 
x) = x − k maka akan didapat f(x) = (x − k) ⋅ g(x) + s 
Jadi, didapat suatu teorema bahw )dibagi oleh x − k maka sisanya 
orema sisa atau dalil sisa. 
Leb ah 
Teorema Sisa 
D 
Jika diambil p( 
Jika diambil x = k maka didapat f(k) = s 
a jika suku banyak f(x 
adalah f(k). 
Teorema di atas dikenal dengan nama te 
ih lanjut dengan cara yang sama didapat bahwa jika f(x) dibagi (ax + b) maka sisanya adal 
f()b. 
a − 
17
Pembinaan Olimpiade Matematika 
Eddy Hermanto, ST Aljabar 
21 
Contoh 42 : Tkan entu sisanya jika f(x) = x4 − 6x3 − 6x2 + 8x + 6 dibagi x − 2 
lah f(2). 
jika f(x) = x4 − 6x3 − 6x2 + 8x + 6 dibagi x − 2 adalah −34. 
D. 
tnya akan dipelajari pengertian faktor dalam suku 
anyak. Pengertian faktor dalam suku banyak dapat dinyatakan dalam bentuk teorema faktor 
(1) 
pakan akar-akar persamaan f(x) = 0. 
a ada paling banyak n buah akar real 
a 
3 2 
ari f(x). Terbukti. 
±24. Kita akan mencoba akar-akar 
jika ada. 
Solusi : 
Dengan teorema sisa akan didapat sisa jika f(x) dibagi x − 2 adaisa = f(2) = 2 
S 
4 − 6 ⋅ 23 − 6 ⋅ 22 + 8 ⋅ 2 + 6 = −34. 
Jadi, sisa 
Coba kerjakan soal di atas dengan cara Horner. 
Teorema Faktor Setelah mempelajari teorema sisa, maka selanju 
b 
berikut : 
Misalkan f(x) adalah suku banyak. (x − k) merupakan faktor dari f(x) jika dan hanya jika f(k) = 0 Perhatikan bahwa pernyataan di atas merupakan biimplikasi. Sehingga pernyataan di atas memiliki arti 
: Jika (x − k) merupakan faktor dari f(x) maka f(k) = 0 
(2) Jika f(k) = 0 maka (x − k) merupakan faktor dari f(x) 
Pada contoh di atas memiliki arti juga bahwa k adalah meru 
Jikaf(x) merupakan suku banyak dalam derajat n makersmaan f(x) = 0. 
p 
Contoh 43 : Tunjukkan bahwa (x + 2) merupakan faktor dari f(x) = x4 + 3x3 + 4x2 + 8x + 8. 
Solusi : 4 
f(−2) = (−2) + 3(−2) + 4(−2) + 8(−2) + 8 = 0 arena f(−2) = 0 maka sesuai teorema faktor maka (x + 2) merupakan faktor d 
K 
Contoh 44 : entukan semua faktor linier dari x 
T 
4 − 2x3 − 13x + 14x + 24 = 0 
Solusi : Kita akan me 
ncoba menyelesaikannya dengan horner. aktor bulat dari 24 adalah ±1, ±2, ±3, ±4, ±6, ±8, ±12 dan 
F 
t ersebut 
014111−− 
24141321−− 
1 −1 −14 0 + 
Karena sisanya tidak sama dengan 0 maka (x − 1) bukan faktor. 
242602−− 
2 
4141321−− 
2 
1 0 −13 −12 + 
Karena sisanya sama dengan 0 maka (x − 2) adalah faktor linier. 
4 3 − x = (x − 3 
x − 2x 13x + 14 + 24 2)(x − 13x 12) 
24 
0 
−
Pembinaan Olimpiade Matematika 
Eddy Hermanto, ST Aljabar 
22 
1211 
121301−− 
1− − 
1 −1 −12 + 
Karena sisanya sama dengan 0 maka x + 1) adalah faktor linier. 
x4 − 2x3 − 13x + 14x + 24 = (x − 2)(x + 1)(x2 − x − 12) 
1 14x + )(x + 3) 
aka faktor-faktor linier dari x4 − 2x3 − 13x + 14x + 24 adalah (x + 1), (x − 2), (x + 3) dan (x − 4). 
E. 
: 
1, x2, x3, ⋅⋅⋅, xn, (dengan kata lain x1, x2, x3, ⋅⋅⋅, xn adalah akar-akar p(x) = 0) maka hubungan-t 
berlaku : 
( 
x4 − 2x3 − 3x + 24 = (x − 2)(x +1)(x − 4 
M 
Teorema Vieta 
Jika 0112211)(axaxaxaxaxpnnnnnn+++++=− − − −L adalah polinomial dengan pembuat nol 
x 
0 
hubungan berikunnaannxxxx 
x1+ 1 
2 3 1 
+ + + + = − − L − 
nna annjijixxxxxxxxxxxx2142323121−=++++++=− <Σ LL nnaannnkjikjixxxxxxxxx x x x x x x x x x 3 
1 2 3 1 3 4 2 3 4 2 4 5 2 1 
= + + + + + + = − − − − 
< < 
Σ L L 
M 
()naannnxxxxx011321−=−L 
Contoh 45 : 
Persamaan kuadrat x2 + 5x − = 0 memiliki akar-akar x1 dan x2. Tentukan nilai x1 
3 + x2 
3. 
olusi : 
n x2 + 5x − 7 = 0 didapat nilai A = 1, B = 5 dan C = −7 
7 
S 
Dari persamaa 
x1 + x2 = −AB = −5 
x1x2 = AC = −7 
x − 3x1x2(x1 + x2) 
x + x = (− 3(−7)(−5) = −125 − 105 
3 . 
banyak x − 5x − 16x + 41x − 15 = 0 memiliki akar-akar a, b, c dan d. Maka nilai 
ari 
. a2 + b2 + c2 + d2 
x13 + x23 = (x1 + x2)3 − 3x12x2 − 3x1x22 3x13 + x23 = (1 + x2) 
1 
333− 
25) x13 + x2= −230 
Contoh 46 : 432 
Persamaan suku 
d 
a 
b. a1 + b1 + 
cadalah ⋅⋅⋅⋅⋅⋅ 
1 
+ 
+ bc + bd + cd = −16 
Solusi : a + b + c d =5 
ab + ac + ad
Pembinaan Olimpiade Matematika 
Eddy Hermanto, ST Aljabar 
23 
abc + abd + acd + bcd = −41 
. a2 + b2 + c2 + d2 = (a + b + c + d)2 − 2(ab + ac + ad + bc + bd + cd) = 52 − 2(−16) = 57 
abcd = −15 
a 
b 
. a + 1 
b + 1 
c + 1 
d = 1 
abcd 
abc+abd +acd +bcd = 1541− − = 1541 
Contoh 47 : (OSP 2005/Canadian Mα, β O 1996) Jika dan γ adalah akar-akar x3 − x − 1 = 0 maka tentukan nilai 
αα − + 
1 
1 + ββ − + 11 + γγ − + 11 . 
Dengan melihat Ax + Bx + Cx + D = 0 dan x x 1 = 0 didapat A = 1, B = 0, C = 1 dan D = 1. 
Solusi : 323−−−− γβα++ = AB− = 0 
βγαγαβ++ = AC = 11− = −1 αβγ = AD = − 1)1(− = 1 − 
αα − + 11 + ββ − + 11 + γγ = − 
+ 
1 
1 ()()( ) ( )( )( ) ( )( )( ) 
( α )( β )( γ ) 
γ β α γ γ α β 
− − − 
+ + − − + + − − 
1 1 1 
1 1 1 1 1 1 
= 
βα−−+111 
()() ()()αβγβγαγαβγβααβγβγαγαβγβα −+++++− +++−++− 133 
= 
()()() ()()()110113103−−+− +−−− = −7 
LATIHAN 4 : 
1. ika f(x) dibagi dengan (x − 2) sisanya 24, sedangkan jika dibagi dengan (x + 5) sisanya 10. Jika f(x) 
ibagi dengan x2 + 3x − 10 sisanya adalah ⋅⋅⋅⋅⋅⋅ 
2. Jika v(x) dibagi x2 − x dan x2 + x berturut-turut akan bersisa 5x + 1 dan 3x + 1, maka bila v(x) dibagi 
. (OSP 2006) Jika (x − 1)2 membagi ax4 + bx3 + 1, maka ab = ⋅⋅⋅⋅ 
4. adalah bilangan-bilangan bulat dan x2 − x − 1 merupakan faktor dari ax3 + 
bx + 1, maka b = ⋅⋅⋅⋅⋅⋅⋅⋅⋅⋅ 
5. HSME 1999) Tentukan banyaknya titik potong maksimal dari dua grafik y = p(x) dan y = q(x) 
ersebut adalah 1. 
alah ⋅⋅⋅⋅⋅⋅ 
J 
d 
x2 − 1 sisanya adalah ⋅⋅⋅⋅ 
3 
(OSK 2008) Jika a dan b 2 
(A 
dengan p(x) dan q(x) keduanya adalah suku banyak berderajat empat dan memenuhi koefisien x4 dari kedua suku banyak t 
6. Suku banyak f(x) dibagi (x + 1) sisanya −2 dan dibagi (x − 3) sisanya 7. Sedangkan suku banyak g(x) jika dibagi (x + 1) akan bersisa 3 dan jika dibagi (x − 3) akan bersisa 2. Diketahui h(x) = f(x) ⋅ g(x). Jika h(x) dibagi x2 − 2x − 3, maka sisanya ad
Pembinaan Olimpiade Matematika 
Eddy Hermanto, ST Aljabar 
24 
7. (OSP 2009) Misalkan p(x) = x2 − 6 dan A = {x ∈ R⏐p(p(x)) = x}. Nilai maksimal dari {⏐x⏐ : x ∈ A} adalah ⋅⋅⋅⋅⋅⋅ 
8. Jika persamaan (3x2 − x + 1)3 dijabarkan dalam suku-sukunya maka akan menjadi persamaan 
a4 + a3 + a2 + a1 + a0 
b) a6 − a5 + a4 − a3 + a2 − a1 + a0 
9. + x − 2 mempunyai tiga pembuat nol yaitu a, b, dan c. Nilai dari 
10. Tentukan semua nilai m sehingga persamaan x4 − (3m + 2)x2 + m2 = 0 memiliki 4 akar real yang 
1. (OSP 2008) Misalkan a, b, c, d bilangan rasional. Jika diketahui persamaan x4 + ax3 + bx2 + cx + d = 0 
polinomial a6x6 + a5x5 + a4x4 + a3x3 + a2x2 + a1x + a0. Tentukan nilai dari : a) a6 + a5 + 
c) a6 + a5 + a4 + a3 + a2 + a1 d) a6 + a4 + a2 + a0 
(OSK 2010) Polinom P(x) = x3 − x2 
a3 + b3 + c3 adalah ⋅⋅⋅⋅⋅⋅⋅⋅ 
membentuk barisan aritmatika. 
12 dan mempunyai 4 akar real, dua di antaranya adalah 2008. Nilai dari a + b + c + d adalah ⋅⋅ 12. (OSP 2010) Persamaan kuadrat x − px − 2p = 0 mempunyai dua akar real α dan β. Jika α + β= 16, maka hasil tambah semua nilai p yang memenuhi adalah ⋅⋅⋅⋅⋅⋅ 
2 3 3 
13. IME 1996) Akar-akar x3 + 3x2 + 4x − 11 = 0 adalah a, b dan c. Persamaan pangkat tiga dengan akar- 
14. (OSK 2003) Misalkan bahwa f(x) = x + ax + bx + cx + dx + c dan bahwa f(1) = f(2) = f(3) = f(4) = 
15. (AIME 1993) Misalkan po(x) = x + 313x − 77x − 8 dan pn(x) = pn−1(x − n). Tentukan koefisien x dari 
16. (OSP 2009) Misalkan a, b, c adalah akar-akar polinom x3 − 8x2 + 4x − 2. Jika f(x) = x3 + px2 + qx + r 
17. (NAHC 1995-1996 Second Round) Misalkan p(x) = x6 + ax5 + bx4 + cx3 + dx2 + ex + f adalah polinomial 
18. (OSP 2010) Diberikan polinomial P(x) = x + ax + bx + cx + d dengan a, b, c, dan d konstanta. Jika 
(A 
akar a + b, a + c dan b + c adalah x3 + rx2 + sx + t = 0. Tentukan nilai t. 5432 
f(5). Berapakah nilai a ? 32 
p20(x). 
adalah polinom dengan akar-akar a + b − c, b + c − a, c + a − b maka f(1) = ⋅⋅⋅⋅⋅⋅ 
yang memenuhi p(1) = 1, p(2) = 2, p(3) = 3, p(4) = 4, p(5) = 5 dan p(6) = 6. Nilai dari p(7) adalah ⋅⋅⋅⋅⋅ 432 
P(1) = 10, P(2) = 20, dan P(3) = 30, maka nilai ()() 10812−+PP = ⋅⋅⋅⋅⋅⋅⋅⋅⋅⋅ 
(AIME 2003 Bagian Kedua) Akar-akar persamaa 19. n x4 − x3 − x2 − 1 = 0 adalah a, b, c dan d. Tentukan 
nilai dari p(a) + p(b) + p(c) + p(d) jika p(x) = x6 − x5 − . 
20. anadian MO 1970) Diberikan polinomial f(x) = xn + a xn-1 + a2xn-2 + ⋅⋅⋅ + an-1x + an dengan koefisien 
uhi f(k) = 8. 
x3 − x2 − x 
(C 
1a1, a2, ⋅⋅⋅, an semuanya bulat dan ada 4 bilangan bulat berbeda a, b, c dan d yang memenuhi f(a) = f(b) = f(c) = f(d) = 5. Tunjukkan bahwa tidak ada bilangan bulat k yang memen
Pembinaan Olimpiade Matematika 
Eddy Hermanto, ST Aljabar 
25 
5. ERSAMAAN 
da beberapa persamaan yang akan dibahas, yaitu : 
. Persamaan Kuadrat 
rsamaan kuadrat adalah Ax2 + Bx + C = 0. 
Misalkan x dan x2 adalah nilai x yang memenuhi persamaan kuadrat di atas. Nilai x1 dan x2 
gan akar-akar. Maka berlaku. 
2) 
ilai x yang memenuhi dapat dicari dengan cara kuadrat sempurna, 
dengan menggunakan rumus x1,2 = 
P 
A 
A 
Bentuk pe 
1) Pengertian akar 
1dikenal juga den 
Ax12 + Bx1 + C = 0 
Ax22 + Bx2 + C = 0 
Menentukan nilai akar-akar persamaan kuadrat 
Untuk mencari n 
m 
emfaktorkan maupunAACBB242−±− sebagaimana yang telah 
an. 
eda. 
rta x dan x keduanya real. 
3) 
dituliskan ke dalam bentuk 
isalkan terdapat persamaan kuadrat Ax2 + Bx + C = 0 yang memiliki akar-akar x1 dan x2. Maka 
x adalah sebagai berikut. 
didapatkan dari pelajaran di kelas. Persamaan B2 − 4AC dikenal dengan nama diskriminan. Nilai diskriminan ini menentukan jenis- jenis akar (nilai x1 dan x2). Ada tiga kemungkinan nilai diskrimin• Jika B2 − 4AC > 0 maka x1 dan x2 keduanya real dan berb• Jika B2 − 4AC = 0 maka x1 = x2 se 
12• Jika B2 − 4AC < 0 maka x1 dan x2 keduanya tidak real. 
Hubungan kedua akar 
Persamaan kuadrat yang memiliki akar-akar x1 dan x2 dapat 
persamaan x2 − (x1 + x2)x + x1x2 = 0. 
M 
hubungan antara x1 dan 
2x1 + x2 = AB− 
x1 ⋅ x2 = AC 
4) an rsamaan kuadrat baru. 
persamaan kuadrat Ax2 + Bx + C = 0 memiliki akar-akar x1 dan x2. Ada beberapa cara 
n enentukan persamaan kuadrat yang memiliki akar-akar x3 dan x4 dan memiliki 
ubungan tertentu dengan x1 dan x2. 
(x + x )x + x x = 0. 
x + x4 dan x3x4 ke dalam bentuk x1 + x2 dan x1x2 maka 
ehingga akan didapat persamaan 
b. 
-akar x3 dan x4. 
5) M 
MenentukpeMisalkanjika ingim 
h 
a. Membawa ke dalam persamaan x2 − 
3434Misalkan terdapat persamaan kuadrat Ax2 + Bx + C = 0 yang memiliki akar-akar x1 dan x2. Dari keterangan sebelumnya akan didapatkan nilai dari x1 + x2 dan x1x2. Jika dapat ditentukan nilai dari 
3berarti nilai dari x3 + x4 dan x3x4 dapat ditentukan s 
kuadrat yang memiliki akar-akar x3 dan x4 yaitu x2 − (x3 + x4)x + x3x4 = 0. Melakukan subtitusi setelah menghilangkan indeks 
Jika dari hubungan x3 dan x4 yang memiliki hubungan tertentu dengan x1 dan x2 kita hilangkan indeksnya lalu kita subtitusikan ke persamaan semula dan mendapatkan persamaan kuadrat baru. Maka persamaan kudarat tersebut memiliki akar 
enentukan nilai suatu bilangan yang berbentuk abba2++ dan abba2−+ ka ba+ dan ba− keduanya dikuadratkan akan didapat Ji 
()abbaba22++=+ ()abbaba22 − = + −
Pembinaan Olimpiade Matematika 
Eddy Hermanto, ST Aljabar 
26 
Sehingga dapat ditentukan nilai dari a+ b+2ab dan abba2−+ , yaitu 
baabba+=++2 baabba−=−+2 dengan syarat a ≥ b. 
Contoh 48 : 
Jika salah satu akar x2 + (a + 1)x + (3a + 2) = 0 adalah 5, maka akar lainnya adalah ⋅⋅⋅⋅ 
Solusi : 
esuai pengertian akar maka akan didapat 
5 + (3a + 2) = 0 
1)x − (3(−4) + 2) = 0 
an x2 = −2 
-akar persamaan cx2 + bx + a = 0, maka 
S 
52 + (a + 1) ⋅ 
a = −4 ersamaan kuadrat tersebut adalah 
P 
x2 + (−4 + 
x2 − 3x − 10 = 0 
(x − 5)(x + 2) = 0 
x1 = 5 d 
Jadi, akar lainnya adalah −2. 
Contoh 49 : 
Jika x1 dan x2 akar222111xx+ = ⋅⋅⋅⋅⋅⋅ 
olusi : 
S 
x1 + x2 = −cb x1x2 = ca 
2221xx 
1 + 1 = 
()2212221xxxx+ 2211xx+= () 
1 2 
()221212212xxxxxx−+ 222111+ x x = 222aacb− 
Contoh 50 : 
Persamaan kuadrat yang akar-akarnya 3 lebihnya dari akar-akar persamaan kuadrat x2 + 5x − 24 = 0 
adalah ⋅ 
olusi : 
an x2 adalah akar-akar persamaan kuadrat x2 + 5x − 24 = 0. 
1 2 an x1x2 = −24 
(x1 + x2) + 6 = 1 
⋅⋅⋅⋅⋅⋅ 
S 
Misalkan x1 d 
Maksud soal adalah menentukan persamaan kuadrat yang akar-akarnya x3 = x1 + 3 dan x4 = x2 + 3. Alternatif 1 : + x = −5 d 
x 
x3 + x4 = 
x3 ⋅ x4 = (x1 + 3)(x2 + 3) = x1x2 + 3(x1 + x2) + 9 = −24 − 15 + 9 = −30 
Persamaan kuadrat baru adalah x2 − (x3 + x4)x + x3x4 = 0. n kuadrat yang diminta adalah x2 − x − 30 = 0 
Jadi, persamaa
Pembinaan Olimpiade Matematika 
Eddy Hermanto, ST Aljabar 
27 
Alternatif 2 : 
Misalkan y3 = x1 + 3 dan y4 = x2 + 3 
Jika indeks dihilangkan akan didapat y = x + 3. Subtitusikan x = y − 3 ke persamaan semula. 
− y − 30 = 0 
2 − y − 30 = 0 merupakan persamaan kuadrat yang akar-akarnya x1 + 3 dan x2 + 3. 
n kuadrat yang diminta adalah x2 − x − 30 = 0 
(y − 3)2 + 5(y − 3) − 24 = 0 2 
y 
y 
Jadi, persamaa 
Contoh 51 : 
LL=+348 
olusi : 
S262261228348⋅++=+=+ . Memperhatikan baabba+=++2 , maka 
26348+=+ 
Contoh 52 : 
(AIME 1983) Tentukan hasil kali semua akar-akar real 4518230182+xx + = x2 + x + . 
olusi : 
isalkan y = x2 + 18x + 30 dan kuadratkan kedua ruas persamaan semula, maka 
) = 0 
aka nilai y yang memenuhi hanya y = 10. 
adi, terdapat dua akar yang tidak real. 
+ 30 = 10 sehingga x2 + 18x + 20 = 0 
l yang berbeda sehingga 
S 
M 
y 2 = 4(y + 15) (y − 10)(y + 6Karena akar dari suatu bilangan tak mungkin negatif m 
J 
x2 + 18x 
Jadi, hasil kali kedua akar persamaan x2 + 18x + 20 = 0 sama dengan 20. 
Contoh 53 : 
(OSK 2002) Misalkan a dan b bilangan reaabbaba1010+ ++ = 2 
T 
entukan nilai ba. 
Solusi : 
Karena abbaba1010+ 
+ + = 2 maka bab101+ = 2 b 
a 
a +10 + 
isa 
M 
l xba= , maka 
xx+101x + 10 = + 19x 
x= + − 
2 − 10x2 
(5x − 4) ) = 0 
x = 1 atau x 
210(x − 1 
=54 Jadi, karena a ≠ b, maka x ≠ 1. 
Jadi, 54=ba
Pembinaan Olimpiade Matematika 
Eddy Hermanto, ST Aljabar 
28 
LATIHAN 5.A 
r-akarnya dua lebih besar dari akar-akar x2 + px + 1 = 0 tapi tiga 
bih ke il dari akar-akar persamaan 2x2 − 3x + q = 0 adalah ⋅⋅⋅⋅ 
. Jika p = 
1. Persamaan kuadrat yang aka 
lec 
22231xxx− + maka batas-batas p supaya x r ⋅⋅⋅ 
2 rsamaan kuadrat yang akar-akarnya 
eal adalah 
3. Jika kedua akar persamaan kuadrat x2 − px + p = 0 bernilai real positif, maka batas-batas nilai p yang memenuhi adalah ⋅⋅⋅⋅ 4. Jika x1 dan x akar-akar persamaan x + 2x + 4 = 0, maka pe 
211 dan 
1 − 
x 1 
1 
x2 − 
entukan 
adalah ⋅⋅⋅⋅⋅ 
5. (OSK 2005) Misalkan a dan b adalah bilangan real taknol yang memenuhi 9a2 − 12ab + 4b2 = 0. 
T ba . 
6. (AIME 1990) Tentukan nilai dari ()()2/32/34365243652−−+ . 
7. (AIME 1990) Tentukan penyelesaian positif 6910211222 −10 −29 −10 −45 − − =+ x x x x x x 
8. RML 1999 Individual) Jika a dan b adalah akar-akar persamaan kuadrat 11x2 − 4x − 2 = 0. 
(1 + a + a + )(1 + b + b + ) 
tuk x2 + bx + c = 0. Berapa banyakkah persamaan 
demikian yang memiliki akar-akar real jika koefisien b dan c hanya boleh dipilih dari himpunan 
10. an bulat sehingga 
. 
(Ahitunglah nilai dari : 2 ⋅⋅⋅2 ⋅⋅⋅ 9. (OSP 2002) Tinjau persamaan yang berben 
{1,2,3,4,5,6} ? 
(OSK 2010) Jika a dan b bilang200922010+ merupakan solusi kuadrat 
x2 + ax + b = 0 maka nilai a + b adalah ⋅⋅⋅⋅⋅ (OSK 2010) Diketahui bahwa ada tepat 1 bilangan asli n sehingga n 11. 
rna. Bilangan asli n tersebut adalah ⋅⋅⋅⋅⋅⋅⋅ 
ah x1 dan x2 sedan 
dan v. Jika u + v = −uv maka nilai dari x1 
3x2 + x1x2 
3 
adalah ⋅⋅⋅⋅⋅⋅⋅⋅ 
13. )x + 2a + 4b = 0. Jika α = 2β maka nilai 
dari a + b = ⋅⋅⋅⋅⋅⋅⋅⋅ 
14. 
2 + n + 2010 merupakan uadrat sempu 
k12. Akar-akar persamaan kuadrat x2 + 6x + c = 0 adalgkan akar-akar persamaan kuadrat x2 + (x12 + x22)x + 4 = 0 adalah u 
α dan β adalah akar-akar persamaan kuadrat x2 − 3(a − 12 
Jika suku pertama deret geometri tak hingga adalah 2 dan rasionya adalah r = αm = m β untuk ilai m > 0 d 
n an α, β akar-akar x2 − (3m + 2) + (4m + 12) = 0, maka jumlah deret geometri tak 
15. + 1)x + p = 0 dengan akar-akar x1 dan x2 serta 3x2 − (q − 1)x − 1 = 0 dengan 
kar-akar x3 dan x4. Jika x1x3 = 1 dan x2x4 = 1, maka nilai dari p − 2q + 13 = ⋅⋅⋅⋅⋅⋅⋅⋅⋅⋅ 
hingga tersebut adalah ⋅⋅⋅ iketahui x 
Da 
2 − (2p
Pembinaan Olimpiade Matematika 
Eddy Hermanto, ST Aljabar 
29 
16. Jika a ≠ b dan jika persamaan-persamaan x2 + ax + bc = 0 dan x2 + bx + ac = 0 mempunyai tepat sebuah akar persekutuan, tunjukkan bahwa akar-akar yang lain dari kedua persamaan tersebut memenuhi persamaan x2 + cx + ab = 0. 
α β 2 δ 
18. (AIME 1991) Misalkan k adalah penjumlahan semua nilai ilai x yang 
17. isalkan dan adalah akar-akar persamaan x + px + 1 = 0 sedangkan γ dan adalah akar-akar persamaan x 
M 
2 + qx + 1 = 0. Buktikan bahwa (α − γ)(β − γ)(α + δ)(β + δ) = q2 − p2. 
mutlak dari nilai-nmemenuhi 2. xx91199119+ += . Tentukan nilai dari k 
9119911919+ + + 91 
19. 1, c1, b2 dan c2 adalah bilangan real yang 1 c2). Tunjukkan 
bahwa sedikitnya satu dari dua persamaan x2 + b1x + c1 = 0 dan x2 + b2x + c2 = 0 memiliki akar-akar 
real. 
20. Diberikan a, b, c ∈ bilangan real serta a dan 4a + 3b + 2c mempunyai tanda yang sama. 
Tunjukkan bahwa persamaan ax2 + bx + c = 0 kedua akarnya tidak mungkin terletak pada 
interval (1, 2). 
B. Pers onen 
Dalam pembahasan hanya akan disinggung tentang sifat-sifat pada eksponen, yaitu : 
(ii) 
ii) 
Diketahui b memenuhi b1b2 = 2(c + 
amaan Eksp 
(i) ao = 1 untuk a ≠ 0 4421Lnkalinaaaaa⋅⋅⋅⋅= untuk n ∈ N. 
443 
cbcbaaa+=⋅ 
(i 
(iv) c = cba− untu 
ab 
k a ≠ 0 
(vi) 
a 
(v) ()bccbaa= mmaa1=− untuk a ≠ 0 
21aa= (vii) d a ≥ 0. engan syaratnmnmaa= 
(viii) Contoh 54 : 
Harga x yang memenuhi persamaan 45384++=xx adalah ⋅⋅⋅⋅⋅ 
Solusi : 
453+=x 8x+ 
4 
() () 4533222+ += xx (sifat (v) dan sifat (viii))
Pembinaan Olimpiade Matematika 
Eddy Hermanto, ST Aljabar 
30 
8(x + 3) = 3(x + 5) 
59−=x 
Jadi, nilai x yang memenuhi adalah x = 
−59 
. 
Contoh 55 : 
Manakah yang le 175 75 uktikan. 
2175 = (2 25 = 12825 dan 575 = (53)25 = 12525 
adi, 2175 lebih besar dari 575. 
bih besar : 2 atau 5 ? B 
Solusi : 
7) 12825 > 12525 
2J 175 > 575 
Contoh 56 : 
(OK 20 
S02) Bilangan () ()284 sama dengan ⋅⋅⋅⋅⋅ 
24 8 
Solusi : () 
( ) 122 
441628 
2 2 
32 
4 32 32 
= = = 
TIHAN 5.B 
aan 
8LA 
1. Persam()xx33243112273=⋅− memberikan nilai x sama dengan ⋅⋅⋅⋅⋅ 
2. Jika 53x = 8, maka 5 + x = ⋅⋅⋅⋅⋅⋅ 
. Jumlah akar-akar persamaan 5x+1 + 56−x = 11 adalah ⋅⋅⋅⋅⋅⋅⋅⋅⋅ 
. Himpunan penyelesaian dari 8−2 x + ⋅ − = adalah ⋅⋅⋅⋅⋅ 
rsamaan 3 2 2 x − x+ + 
1 2 enyelesaiannya, maka 
1 dan x2, maka (x1 ⋅ x2)2 = ⋅⋅⋅⋅⋅ 
C. Persamaan Logaritma 
a b = alog c. 
Sifat-sifat pada logaritma, yaitu : 
3 
3 
4 
025495 
3−x5. an pe1032=−xx. Jika x dan x adalah p 
Diberik 
33 
LL=+213xx 
6. Persamaan 54(6x) + 3x = 6(18x) + 9 mempunyai penyelesaian x Pengertian : Jika ab = c mak 
(i) aabplogloglog== dengan syarat a, p ≠ 1 dan a, b, p > 0 
b p b 
a log log 
i) = 1 dengan syarat a > 0 dan a ≠ 1. 
(i 
aalog
Pembinaan Olimpiade Matematika 
Eddy Hermanto, ST Aljabar 
31 
(iii) 
alogaaabbalog= dengan a,b ≠ 1 da 
1 
n a,b > 0 
(iv) log b + log c = log (bc) dengan syarat a ≠ 1 dan a, b, c > 0 
()cb aaacblogloglog=− 
(v) dengan syarat a ≠ 1 dan a, b, c > 0 bmbanamloglog⋅== dengan sya 
n 
b 
m 
an log rat a ≠ 1 dan a, b > 0 
alog bn = n ⋅ alog b > 0 
≠ 
(vi) (vii) b dengan syarat a ≠ 1 dan a, 
(viii) dengan syarat a, b 1 dan a, b, c > 0 
ccbabalogloglog=⋅ nm(ix) a b b am n log = dengan 
Catatan : Bentuk alog b kadang-kadang ditulis dengan log b. 
= a dan 3log 4 = b 
4 
erdasarkan sifat (iii) maka 4log 15 = 
syarat a ≠ 1 dan a, b > 0. a 
Contoh 57 : Jika 5log 3 , maka 4log 15 = ⋅⋅⋅⋅⋅⋅ Solusi : Berdi) maka didapat 5log 3 ⋅ 3log 4 = 5log 
asarkan sifat (vii = ab erdasarkan sifat (iv) maka 
B 
4log 15 = 4log 3 + 4log 5 
Bb1 
+ ab1 
Maka 4log 15 = 
ab 
a+1 
Contoh 58 : Himpunan penyelesaian pertidaksama : 
an() xlog2 − x32log2− 
xlog (x + 6) + xxlog12+ = 1 adalah ⋅⋅⋅⋅⋅ 
SB 
olusi : 
sifat (i) dan (iii) maka persoalan di atas ekivalen dengan 
x x 
xlog 
erdasarkan x 
log (2x − 3) − log (x + 6) + log (x + 2) = 1 Berdasarkan sifat (iv) dan (v) maka 
()() ( )623+ + xx = 1 2 x− 
kesamaan maka 
x − 3)(x + 2) = x(x + 6) 
6 = x2 + 6x 
dua nilai sebut ke persamaan awal. 
basis pada xlog (x + 6) yaitu bahwa x > 0 dan x ≠ 1. 
ai x yang memenuhi adalah {6}. 
Berdasarkan 
(2 
2x2 + x − 
x2 − 5x − 6 = 0 
(x − 6)(x + 1) = 0 
x1 = 6 dan x2 = −1 Uji kex ter 
x = −1 tidak memenuhi syarat hi syarat basis mapun numerus. 
Sedangkan x = 6 memenua nil 
Jadi, himpunan semu
Pembinaan Olimpiade Matematika 
Eddy Hermanto, ST Aljabar 
32 
Contoh 59 : 
(OSK 2004) Jika log p + log q = log (p + q), maka p dinyatakan dalam q adalah p = ⋅⋅⋅⋅ 
olusi : 
g p + log q = log (p + q) 
g (pq) = log (p + q) 
q = p + q 
S 
l o 
lo 
p 
p(q − 1) = q 
J 
adi, p = 1−qq 
Contoh 60 : 
Jika 21 
logab 
= dan kccb= logac , maka k = ⋅⋅⋅ 
olusi : 
erdasarkan sifat (i) akan didapat 
S 
B2loglogcc 
log 1 
log 
g 
1 = = = k+ 
c 
b 
c 
a 
Maka k + 1 = 2 
adi, k = 1 
Contoh 61 : 
(OSP 2003) Berapakah nilai x yang memenuhi 4log (2log x) + 2log (4log x) = 2 ? 
sifat (vi) maka 4log (2log x) = 
lologba 
J 
Solusi : 212 
Berdasarkan log (2log x) sehingga 4log (2log x) = 2log (2log x)1/2 
erdasarkan sifat (vi) maka 2log (4log x) = 2log ( 212log x) sehingga 2log (4log x) = 2log (2log x 
B ) 
x) = 2 
42 
log (log x) + 2log (4log x) = 2 sehingga 2log (2log x)1/2 + 2log (2log 
M 
aka log (xxloglog2212⋅⋅ 2 ) = 2 
42loglog22212==⋅⋅xx ()8232=/logx 
2log x = 4 2 x = 4 
LATIHAN 5.C 
Jadi, x = 16 
( 
)() 
1. 3315log 
2 3 2 log 45 − log5 = ⋅⋅⋅⋅⋅⋅⋅ 
. (OSK 2003) Misalkan 3a = 4, 4b = 5, 5c = 6, 6d = 7, 7e = 8, dan 8f = 9. Berapakah hasil kali abcdef ? 
3. (AIME 1984) Bilangan real x dan y me + 4log y2 = 5 dan 8log y + 4log x2 = 7. 
Tentukan xy. 
3 
2 
menuhi 8log x
Pembinaan Olimpiade Matematika 
Eddy Hermanto, ST Aljabar 
33 
(AHSME 1998) Tentukan nilai dari 
4. 
!100log!100log!100log!100log100432 
1 + 1 + 1 +L+ 1 
i p i x a 
3 log 
= 3 untuk x > y, maka x + y = ⋅⋅⋅⋅⋅ 
7. , u2, u3, u4, u5 dan u6 serta u1 + u6 = 11 dan 10log u3 + 10log u4 = 1. 
ika untuk setiap n = 1, 2, 3, ⋅⋅⋅, 5 berlaku un+1 = p⋅un dan p > 1, maka 10log p = ⋅⋅⋅⋅ 
. (AIME 1983) Diketahui x, y dan z adalah bilangan real lebih dari 1 dan w adalah bilangan real 
positif. Jika xlog w = 24, ylog w = 40 dan xyzlog w = 12, tentukan zlog w. 
9. (AIME 1988) Diberikan log (8log x) = 8log ( log x). Tentukan nilai dari ( log x)2. 
10. 
dengan n! = 1 x 2 x 3 x ⋅⋅⋅ x n. 
5. Batas-batas nilai p agar f(x) = 2log (px2 + px + 3) terdefinisada setiap niladalah ⋅⋅⋅⋅⋅ 
6. Jika yxxloglog33+ = 27 dan xylog3+ 
y 
Diketahui barisan 6 bilangan u1 
J 
8 222 
(AHSME 1997) Untuk bilangan asli n maka 8log n jika 8log n bilangan rasional 
f(n) = 0 untuk lainnya 1997 
Nilai dari 3()Σ= 
rupakan faktor dari N dan memenuhi 
2log (3log (5 )) 
12. (ARML 2000 Individual ka b = deret tak hingga berikut : 
1nnf 
11. (AHSME 199 
8) Ada berapa banyak bilangan prima yang me) = 11. 
log (7log 
N 
) Ji 2000, hitunglah nilai ()()()()()()...5log2log5log2log5log2lo 
g 0 40 1 41 2 42 b b + b b + b b + 
2 1 1 2 2 
D. 
1) Persamaan Lingkaran berpusat di (0,0) dan (a,b) 
ingkaran adalah kumpulan titik-titik yang memiliki jarak yang sama terhadap suatu titik 
aan : 
x2 + y2 = r2 yang merupakan persamaan lingkaran berpusat di (0,0) dan berjari-jari r. 
r2 yang merupakan persamaan lingkaran berpusat di (a,b) dan berjari-jari r. 
akan didapat persamaan umum lingkaran yang 
karan jika diketahui pusat lingkaran dan persamaan 
mus jarak titik ke 
n jari-jari 
13. (AIME 2002) Penyelesaian dari sistem persamaan 225log x + 64log y = 4 dan xlog 225 − ylog 64 = 1 adalah (x, y) = (x1, y1) dan (x2, y). Nilai dari 30log (xyxy) adalah ⋅⋅⋅⋅⋅⋅ 
Persamaan Lingkaran 
L 
tertentu, yaitu pusat lingkaran. Jadi ada dua hal yang sangat berkaitan dengan lingkaran yaitu jari-jari lingkaran, R, dan pusat lingkaran. Dari pengertian lingkaran tersebut jika diturunkan akan didapat persam 
(x − a)2 + (y − b)2 = 
Jika persamaan (x − a)2 + (y − b)2 = r2 dijabarkan 
berbentuk : x2 + y2 + Ax + By + C = 0 Salah satu cara menentukan persamaan ling 
garis yang menyinggung lingkaran tersebut adalah dengan memanfaatkan ru 
suatu garis lurus sebab jarak titik pusat ke garis singgung tersebut adalah merupaka
Pembinaan Olimpiade Matematika 
Eddy Hermanto, ST Aljabar 
34 
lingkaran. Misalkan suatu garis lurus memiliki persamaan Ax + By + C = 0. Maka rumus jarak titik 
(x1, y1) ke garis tersebut adalah 2211BACByAxd+ ++ =. Misalkan terdapat lingkaran den a 
2) Hubungan antara titik dengan lingkaran 
gan persamaan (x − )2 + (y − b)2 = r2 dan titik (p, q). Maka 
hubungan titik (p, q) dengan (x − a)2 + (y − b)2 = r2 akan memiliki tiga kemungkinan hubungan : 
) Jika (p − a)2 + (q − b)2 < r2 maka titik (p, q) terletak di dalam lingkaran 
(p, q) terletak pada lingkaran 
3) 
(y − b)2 = r2. Bagaimana 
ubtitusikan persamaan y = mx + c ke persamaan lingkaran (x − a)2 + (y − b)2 = r2 sehingga 
h x, yaitu Ax2 + Bx + C = 0. 
ng lingkaran 
ntuk mencari hubungan antara 
s dengan irisan kerucut yang 
4) 
sebut memiliki gradien m. Maka persamaan garis 
singgung dapat dinyatakan dengan 
a 
b) Jika (p − a)2 + (q − b)2 = r2 maka titik 
c) Jika (p − a)2 + (q − b)2 > r2 maka titik (p, q) terletak di luar lingkaran 
Hubungan antara garis lurus dengan lingkaran 
Misalkan diketahui suatu garis lurus y = mx + c dan lingkaran (x − a)2 + 
hubungan antara garis lurus dan lingkaran tersebut ? 
S 
didapat suatu persamaan kuadrat dalam peuba 
Dari persamaan tersebut dapat dihitung diskriminan = B2 − 4AC. (i) Jika B2 − 4AC < 0 maka garis lurus tidak memoto 
(ii) Jika B2 − 4AC = 0 maka garis lurus menyinggung lingkaran (iii) Jika B2 − 4AC > 0 maka garis lurus memotong lingkaran di dua titik 
Prinsip nilai diskriminan di atas tidak hanya dapat digunakan u 
garis lurus dengan lingkaran tetapi juga hubungan antara garis luru 
lain seperti parabola, elips maupun hiperbola. 
Persamaan Garis Singgung pada Lingkaran a) Garis singgung lingkaran dengan gradien tertentu Misalkan diketahui bahwa garis singgung ter 
(i) Untuk lingkaran x2 + y2 = r2 
Persamaan Garis Singgung, 1+±=mrmxy 
2 (ii) Untuk lingkaran (x − a)2 + (y −22 b) = r 
Persamaan Garis Singgung, 
()12+±−=−mraxmby 
b) 
Mis lkan titik (x , y ) terletak pada lingkaran maka persamaan garis singgung yang melalui 
titik t ngan 
) Untuk lingkaran x2 + y2 = r2 
r2 
(x − a) + (y1 − b)(y − b) = r2 
c) Persa ngkaran 
ini dapat dilakukan dengan beberapa cara : 
(i) ertian hubungan antara 
aris lurus dengan lingkaran 
ri 
Garis Singgung melalui titik pada lingkaran a11ersebut dapat ditentukan de 
(i 
Persamaan Garis Singgung, x1x + y1y = 
(ii) Untuk lingkaran (x − a)2 + (y − b)2 = r2 Persamaan Garis Singgung, (x1 − a) 
maan Garis Singgung melalui titik di luar li 
Untuk menentukan persamaan garis singgung 
Dengan mencari rumus diskriminan lalu memanfaatkan peng 
g 
(ii) Dengan menggunakan persamaan garis polar 
(iii) Dengan memanfaatkan persamaan garis singgung dengan gradien m untuk menca 
nilai m
Pembinaan Olimpiade Matematika 
Eddy Hermanto, ST Aljabar 
35 
Contoh 62 : 
SK 2005) Titik A(a, b) disebut titik letis jika a dan b keduanya adalah bilangan bulat. Banyaknya 
tik letis pada lingkaran yang berpusat di O dan berjari-jari 5 adalah 
olusi : 
ngkaran yang berpusat di O dan berjari-jari 5 adalah x2 + y2 = 25 
dan (−4, −3). 
adi, banyaknya titik letis pada lingkaran yang berpusat di O dan berjari-jari 5 ada 12. 
dalah ⋅⋅⋅⋅⋅⋅ 
olusi : 
arak pusat (1, 4) ke garis 3x − 4y − 2 = 0 sama dengan jari-jari lingkaran tersebut. 
(O 
ti 
S 
Persamaan li 
Karena 02 + 52 = 32 + 42 = 25 maka pasangan (x, y) bulat yang memenuhi ada 12, yaitu (0, 5), (0, −5), (5, 0), (−5, 0), (3, 4), (3, −4), (−3, 4), (−3, −4), (4, 3), (4, −3), (−4, 3) 
J 
Contoh 63 : Persamaan lingkaran yang berpusat di (1, 4) dan menyinggung garis 3x − 4y − 2 = 0 a 
S 
J 
Jarak tersebut = d = ()() 224324413−− = 3. 
+ 
Contoh 64 : 
rabola y = x2 + a tepat di satu 
olusi : 
2 + a maka x2 − 6x + a = 0 
agar y = 6x memotong parabola y = x2 + a di satu titik adalah Diskriminan = 0 
6 − 4a = 0 
9 
aris singgung x2 + y2 − 6x + 4y − 12 = 0 di titik (7, −5) adalah ⋅⋅⋅⋅⋅ 
olusi : 
ubtitusi titik (7, −5) ke persamaan x2 + y2 − 6x + 4y − 12 = 0 didapat 
6(7) + 4(−5) − 12 = 0 
ersamaan garis singgungnya adalah (x − 3)(7 − 3) + (y + 2)(−5 + 2) = 25 
samaan garis singgung lingkaran x2 + y2 − 6x + 4y − 12 = 0 di titik (7, −5) adalah 
olusi : 
arena 42 + 22 > 10 maka titik (4,2) terletak di luar lingkaran. 
Persamaan lingkaran berpusat di (1, 4) dan memiliki jari-jari 3 adalah (x − 1)2 + (y − 4)2 = 9 
(OSK 2002) Untuk nilai a yang manakah garis lurus y = 6x memotong pa 
titik ? 
S 
Karena 6x = x 
Disk = 62 − 4(1)(a) = 36 − 4a Syarat 
3 
Jadi, a = 
Contoh 65 : 
Persamaan g 
S 
S 
(7)2 + (−5)2 − 
Artinya titik (7, −5) terletak pada lingkaran x2 + y2 − 6x + 4y − 12 = 0. 
P 
Jadi, per 
4x − 3y = 43 
Contoh 66 : 
Persamaan garis singgung yang ditarik dari titik (4,2) ke lingkaran x2 + y2 = 10 adalah ⋅⋅⋅⋅⋅⋅ 
S 
K
Pembinaan Olimpiade Matematika 
Eddy Hermanto, ST Aljabar 
36 
Alternatif 1 : 
Persamaan garis melalui titik (4,2) dan gradien m adalah y − 2 = m(x − 4). Subtitusi garis tersebut 
maan lingkaran didapat 
2 + 1)x2 + 2(−4m2 + 2m)x + 16m2 − 16m − 6 = 0 
22(−4m2 + 2m)2 − 4(m2 + 1)(16m2 − 16m − 6) 
16m − 6) = 0 
4 + 16m3 m2 − m2 + 16m + 6 = 0 
ke persa 
x2 + (mx − 4m + 2)2 = 10 
(m 
Diskriminan = 
Agar y − 2 = m(x − 4) menyinggung lingkaran x2 + y2 = 10 maka diskriminan harus sama dengan 0. 22(−4m2 + 2m)2 − 4(m2 + 1)(16m2 − 
16m4 − 16m3 + 4m2 − 16m 
+ 6163m2 − 8m − 3 = 0 
(3m + 1)(m − 3) = 0 
Jika m − 
=3r1 maka gais singgung tersebut memiliki persamaan y − 2 = −31(x − 4). 
Jika m = 3 maka garis singgung tersebut memiliki persamaan y − 2 = 3(x − 4). 
aris singgung yang ditarik dari titik (4,2) adalah x + 3y = 10 dan 3x − y = 10. 
t 
yoy = 10 yaitu 
ubtitusikan persamaan garis polar tersebut ke lingkaran x + y = 10 didapat 
10 
maka y = 3 sehingga titik singgung dari garis singgung tersebut pada lingkaran adalah 
2 = −1 sehingga titik singgung dari garis singgung tersebut pada lingkaran adalah 
persamaan garis singgungnya adalah 3x − y = 10. 
aris singgung yang ditarik dari titik (4,2) adalah x + 3y = 10 dan 3x − y = 10. 
an garis singgung tersebut adalah 
Jadi, persamaan g Alternatif 2 : Misalkan tiik (xo, yo) = (4, 2). 
Persamaan garis polar titik (xo, yo) terhadap lingkaran x2 + y2 = 10 adalah xox + 
2x + y = 5 22 
S 
x2 + (5 − 2x)2 = 
x2 − 4x + 3 = 0 
x1 = 1 atau x2 = 3 
Jika x1 = 1 
1(1,3) sehingga persamaan garis singgungnya adalah x + 3y = 10. 
Jika x2 = 3 maka y 
(3,−1) sehingga 
Jadi, persamaan g 
Alternatif 3 : Misalkan gradien garis singgung tersebut adalah m. Maka persama 
12+±=mrmxy 
K 
arena r = 10 2 maka 10102+±=mmxy 
Karena garis tersebut melalui titik (4,2) maka ()mm4210102−±=+ 22 10m + 10 = 4 − 16m + 16m 
(3m + 1)(m − 3) = 0 
Jika m = −31 maka garis singgung tersebut memiliki persamaan y − 2 = 
−31 (x − 4). 
Jika m = 3 maka garis sing tersebut memiliki persamaan y − 2 = 3(x − 4). 
ng yang ditarik dari titik (4,2) adalah x + 3y = 10 dan 3x − y = 10. 
. Persamaan lingkaran dengan titik pusat (4,3) dan jari-jari = 4 adalah ⋅⋅⋅⋅⋅⋅ 
. Persamaan lingkaran yang titik pusatnya terletak pada garis y = x + 1 dan menyinggung sumbu X 
di titik (5,0) adalah ⋅⋅⋅⋅⋅ 
gung 
Jadi, persamaan garis singgu 
LATIHAN 5.D 
1 
2
Pembinaan Olimpiade Matematika 
Eddy Hermanto, ST Aljabar 
37 
3. Suatu lingkaran berjari-jari 5, melalui titik (0,0) dan pusatnya pada garis y = x + 1 mempunyai 
persamaan ⋅⋅⋅⋅⋅⋅ 
4. B(4,−3) dan P(x,y) terletak sedemikian sehingga (PA)2 + (PB)2 = (AB)2. 
Maka P merupakan titik-titik yang terletak pada busur lingkaran yang memotong sumbu X pada 
5. Persamaan garis singgung di titik (7,−1) pada lingkaran x2 + y2 = 50 adalah ⋅⋅⋅⋅⋅ 
6. 
7. Jari-jari lingkaran yang menyinggung sumbu x di (6,0) dan menyinggung garis y = √3 x adalah ⋅⋅⋅⋅ 
8. Persamaan garis singgung yang ditarik dari titik (7,−1) ke lingkaran x2 + y2 = 40 adalah ⋅⋅⋅⋅ 
9. Persamaan garis singgung pada lingkaran x2 + y2 = 36 yang tegak lurus garis 4y = −3x + 80 adalah 
10. Jarak terjauh dari titik (−12,5) ke lingkaran dengan persamaan x2 + y2 = 100 adalah ⋅⋅⋅⋅ 
11. Bilangan real x dan y memenuhi (x + 5)2 + (y − 12)2 = 142, maka nilai minimum dari x2 + y2 
ik potong 
jika k memenuhi a ≤ k ≤ b. Nilai dari b − a adalah ⋅⋅⋅⋅⋅ 
13. 6) Diberikan persamaan x + y = 14x + 6y + 6. Nilai terkecil dari 3x + 4y yang 
memenuhi adalah ⋅⋅⋅⋅⋅ 
E. 
Nila dengan ⏐x⏐ dan memiliki pengertian ⏐x⏐ = x jika x ≥ 0 dan ⏐x⏐ = −x jika 
< 0. Jika hanya memuat satu tanda mutlak maka penyelesaian persamaan dapat dengan 
enggunakan pengertian nilai mutlak atau dapat juga dengan mengkuadratkan tanda mutlak. 
olusi : 
lternatif 1 : 
ian didapat jika x ≥ 2 maka x − 2 = 8 sehingga x = 10 yang memenuhi persamaan. 
hingga x = −6 yang juga memenuhi persamaan. 
adi, penyelesaian x yang memenuhi adalah x = −6 atau x = 10. 
2 − 4x − 60 = 0 
= 0 
Diketahui titik A(−2,1) , 
koordinat ⋅⋅⋅⋅⋅⋅⋅⋅ 
Garis lurus 3x + 4y + k = 0 akan menyinggung lingkaran x2 + y2 + 6x + 8y = 0 jika k bernilai ⋅⋅⋅⋅⋅⋅⋅ 
adalah ⋅⋅⋅⋅⋅⋅ 
12. (AHSME 1998) Kedua grafik x2 + y2 = 4 + 12x + 6y dan x2 + y2 = k + 4x + 12y memiliki tit 
(AHSME 19922 
Persamaan Nilai mutlak i mutlak dari x ditulis 
x 
m 
Contoh 67 : Selesaikan persamaan ⏐x − 2⏐ = 8 
S 
A 
Dari pengert 
Sedangkan jika x < 2 maka 2 − x = 8 se 
J 
Alternatif 2 : 
Karena ⏐x − 2⏐ bernilai tak negatif maka penyelesaiannya dapat dilakukan dengan mengkuadratkan kedua ruas. 
(x − 2)2 = 64 
x 
(x − 10)(x + 6) 
x = 10 atau x = −6
Pembinaan Olimpiade Matematika 
Eddy Hermanto, ST Aljabar 
38 
Persoalan menjadi lebih rumit apabila dalam persamaan tersebut memuat lebih dari satu tanda 
annya dapat dilakukan dengan membagi kasus. 
x + 2⏐ + ⏐3x⏐ = 14 
Maka −x − 2 − 3x = 14 sehingga x = −4 (memenuhi bahwa x ≤ −2) 
Untuk −2 ≤ x ≤ 0 maka |x + 2| = x + 2 dan |3x| = −3x 
| + |3x| = 14. Maka x + 2 − 3x = 14 sehingga x = −6 (tidak memenuhi bahwa −2 ≤ x ≤ 0) 
d adalah = { −4, 3} 
. (OSK 2005) Tentukan semua solusi persamaan ⏐x − 1⏐ + ⏐x − 4⏐ = 2. 
2. (AIME 1983) Tentukan nilai minimum ⎪x − 15⎪ + ⎪x − p − 15⎪ untuk suatu nilai x 
dalam batas p ≤ x ≤ 15 dimana 0 < p < 15. 
3. Banyaknya bilangan bulat berbeda y yang memenuhi ⏐x − 7⏐ − ⏐x + 11⏐ = y adalah ⋅⋅⋅⋅ 
4. − a⎪ − 3. Jika grafik f memotong sumbu-x tepat di 
tiga titik, maka a = ···· 
5. (OSP 2006) Jika ⏐x⏐+ x + y = 10 dan x + ⏐y⏐ − y = 12, maka x + y = ⋅⋅⋅⋅⋅⋅ 
6. banyak tripel bilangan bulat (a,b,c) yang memenuhi 
⎪a + b⎪ + c = 19 dan ab + ⎪c⎪ = 97 
7. (AHSME 1977) Untuk a, b, c bilangan real taknol, semua kemungkinan nilai dari 
mutlak. Penyelesai 
Contoh 68 : (OSP 2003) Apakah himpunan jawab dari persamaan 
⏐ 
Solusi : 
• Untuk x ≤ −2, maka |x + 2| = −x − 2 dan |3x| = −3x 
|x + 2| + |3x| = 14. 
• 
|x + 2 
• Untuk x ≥ 0 maka |x + 2| = x + 2 dan |3x| = 3x 
|x + 2| + |3x| = 14. Maka x + 2 + 3x = 14 sehingga x = 3 (memenuhi bahwa x ≥ 0) 
Jai, himpunan jawab dari persamaan |x + 2| + |3x| = 14 
LATIHAN 5.E : 
1 
dari ⎪x − p⎪ + 
(OSP 2006) Diberikan fungsi f(x) = ⎪⎪x − 2⎪ 
(AHSME 1997) Ada berapa 
abccba+++ 
abccba 
adalah ⋅⋅⋅⋅⋅ 
8. (AHSME 1999) Grafik y = −⎪x − a⎪ + b dan y = ⎪x − c⎪ + d berpotongan di titik (2,5) dan (8,3). 
Nilai a + c adalah ⋅⋅⋅⋅⋅⋅⋅⋅ 
9. IME 1988) xi adalah bilangan real yang memenuhi −1 < xi < 1 dan ⎪x1⎪ + ⎪x2⎪ + ⋅⋅⋅ + ⎪xn⎪ = 19 + 
0. (AIME 2001) Fungsi f(x) memenuhi persamaan f(3x) = 3f(x) untuk semua bilangan real x dan 
(A 
⎪x1 + x2 + ⋅⋅⋅ + xn⎪. Tentukan nilai terkecil n yang memenuhi. 
1 
f(x) = 1 − ⎪x − 2⎪ untuk 1 ≤ x ≤ 3. Tentukan bilangan positif x terkecil yang memenuhi f(x) = f(2001).
Pembinaan Olimpiade Matematika 
Eddy Hermanto, ST Aljabar 
39 
6. SIST 
Sistem 
A. Sist inier 
istem persamaan umum yang dikenal adalah sistem persamaan linier, yaitu sistem persamaan yang 
angkat variabelnya tidak lebih dari satu. Untuk menyelesaikan n buah variabel dibutuhkan n buah 
ersamaan. Penyelesaian sistem persamaan dapat dilakukan dengan menggunakan subtitusi, 
dengan memanfaatkan matriks. 
x + y = 3log 29 ⋅⋅⋅⋅⋅⋅⋅⋅⋅⋅⋅⋅⋅⋅⋅⋅⋅⋅⋅⋅ (1) 
⋅⋅⋅⋅⋅⋅⋅⋅⋅⋅⋅⋅⋅⋅⋅⋅ (2) 
EM PERSAMAAN persamaan terdiri dari lebih dari satu persamaan dalam rangka mencari suatu penyelesaian. em Persamaan L 
S 
p 
p 
eliminasi maupun 
Contoh 69 : 
Harga x yang memenuhi : 3x + y = 29 dan x − y = 1 adalah ⋅⋅⋅ Solusi : 3x + y = 29 dan x − y = 1 
x − y = 1 ⋅⋅⋅⋅ 
Dari persamaan (1) dan (2) didapat 
x = 2 
3 log 29+1 dan y = 2129log3− 
Contoh 70 : 
(OSK 2003) Hari ini usiaku 31 kali usia ayahku. Lima tahun yang lalu, usiaku 41 kali usia ayahku pada 
h usiak g ? 
olusi : 
saat ini = X dan usia ayahku saat ini = Y, maka : 
waktu itu. Berapakau sekaran 
S 
Misal usiaku X = 3Y X − 5 = 141(Y − 5) 
X 
5 = − 41 (3X − 5) 
, usiaku saat ini 15 tahun 
persamaan 
c + d = 0 
+ b + d + e = 1 
+ c + d + e = 2 
= 4 
persamaan di atas didapat 
) = 12 
3 
hingga e = 3 
+ d = 3 sehingga d = −2 
sehingga c = 2 
m persamaan tersebut adalah (a, b, c, d, e) = (−1, 1, 2, −2, 3) 
4X − 20 = 3X − 5 X = 15 Jadi Contoh 71 : 
Selesaikan sistem 
a + b + 
a + b + c + e = 5 
a 
a 
b + c + d + e 
Solusi : 
Jumlahkan semua 
4(a + b + c + d + e 
a + b + c + d + e = 
Maka 0 + e = 3 se 
5 
1 + c = 3 
2 + b = 3 sehingga b = 1 
4 + a = 3 sehingga a = −1 
Jadi, penyelesaian siste
Pembinaan Olimpiade Matematika 
Eddy Hermanto, ST Aljabar 
40 
Contoh 72 : 
Tentukan persamaan lingkaran yang melalui titik (0, 0), (6, 8) dan (7,7) 
ersamaan umum lingkaran adalah 
2 + y2 + Ax + By + C = 0 
diketahui dan ada 3 variabel yang dicari yaitu A, B dan C maka soal ini 
ubtitusikan titik (0,0) ke persamaan lingkaran didapat 
njadi x2 + y2 + Ax + By = 0 
aka 6A + 8B = −100 
ersamaan umum lingkaran tersebut adalah x2 + y2 − 6x − 8y = 0 
B. 
bih dari satu atau merupakan perkalian 
ersamaan tak linier maka persoalan menjadi 
ontoh 73 : 
dengan x,y,z > 0 
entukan nilai 3x + 2y + 3z. 
amaan didapat 
))2 = 12 ⋅ 20 ⋅ 15 = (60)2 
= 60 
y − 2 = 4 
idapat x = 4, y = 6 dan z = 8 
+ 2y + 3z = 48. 
ersamaan tak linier dapat dibawa ke dalam persamaan-telah 
dijelaskan pada rumus Vieta maka hubungan suku banyak dengan 
enyelesaikan persoalan sistem persamaan tak linier. 
ontoh 74 : 
bc = 24 
an x3 − 9x2 + 26x − 24 = 0 
ma horner didapat 
= (x − 2)(x − 3)(x − 4) = 0 
b, c) yang memenuhi adalah (2, 3, 4) dan permutasinya. 
Solusi : 
P 
x 
Karena ada 3 titik yang 
merupakan sistem persamaan linier dengan 3 variabel (peubah). 
S 
C = 0 
Maka persamaan umum lingkaran me 
Berdasarkan titik (6, 8) m 
3A + 4B = −50 ⋅⋅⋅⋅⋅⋅⋅⋅⋅⋅⋅⋅⋅⋅⋅⋅⋅⋅⋅⋅⋅⋅⋅⋅⋅⋅⋅ (1) Berdasarkan titik (7, 7) maka 7A + 7B = −98 ⋅⋅⋅⋅⋅⋅⋅⋅⋅⋅⋅⋅⋅⋅⋅ (2) 
Dari persamaan (1) dan (2) didapat A = −6 dan B = −8 
Maka p 
Sistem Persamaan Tak Linier 
Dalam sistem persamaan tak linier pangkat variabel bisa le 
di antara variabel-variabel yang ada. Dalam sistem p 
lebih sulit dan membutuhkan teknik penyelesaian yang lebih tinggi. 
C 
Ditentukan 3 buah persamaan 
(x − 1)(y − 2) = 12 (y − 2)(z − 3) = 20 (z − 3)(x − 1) = 15 
T 
Solusi : 
Kalikan ketiga pers 
((x − 1)(y − 2)(z − 3 
(x − 1)(y − 2)(z − 3) 
Maka z − 3 = 5, x − 1 = 3 dan 
D 
Jadi, 3x 
Jika persamaan-persamaan dalam sistem p 
persamaan sebagaimana 
akar-akarnya dapat digunakan untuk m 
C 
Tentukan nilai a, b dan c yang memenuhi sistem persamaan berikut : a + b + c = 9 ab + ac + bc = 26 
a 
Solusi : 
Sesuai dengan rumus Vieta maka a, b, dan c adalah akar-akar persama 
Dengan teore 
x3 − 9x2 + 26x − 24 
Tripel (a,
Pembinaan Olimpiade Matematika 
Eddy Hermanto, ST Aljabar 
41 
Contoh 75 : 
0, 
n + m + n = 71 
880 sehingga mn(m + n) = 880 
(AIME 1991) m, n adalah bilangan asli yang memenuhi mn + m + n = 71 dan m2n + mn2 = 88 
tentukan m2 + n2. 
Solusi : 
m 
m 2n + mn2 = 
mn + mn880 = 71 mn) 
( 
2 − 71(mn) + 880 = 0 
)(mn − 55) = 0 
= 55 
, 2) dan (16, 1) tetapi tidak 
emenuhi m + n = 55. 
+ n = 71 − 55 = 16 
menuhi mn = 55 adalah (1, 55), (5, 11), (11, 5), (55, 1). 
+ n = 16 adalah m = 5 dan n = 11 atau m = 11 dan n = 5 
Jad 
Con 
Ten 
ab = de = 4 dan ea = 6 
olusi : 
alikan kelima persamaan didapat 
⋅ 32 
b = 1 dan cd = 3 maka e = 4 sehingga d = 1 dan a = 
(mn − 16 
mn = 16 atau mn 
• Jika mn = 16 maka m + n = 71 − 16 = 55 hi mn = 16 adalah (1, 16), (2, 8), (4, 4), (8Nilai (m, n) yang memenuada yang m 
• Jika mn = 55 maka m 
Nilai (m, n) yang me 
Yang memenuhi m 
m2 + n2 = 52 + 112 = 146 
i, nilai dari m2 + n2 sama dengan 146. 
toh 76 : 
tukan penyelesaian dari sistem persamaan 
1 ; bc = 2 ; cd = 3 ; 
S 
K 
(abcde)2 = 24 
abcde = 12 atau abcde = −12 
• Jika abcde = 12 
Mengingat a23 
. 
Maka b = 32 dan c = 3. 
Jadi, (a, b, c 
, d, e) = (2, 3 
32 
, 3, 1, 4) merupakan penyelesaian. 
= 
• Jika abcde = −12 23 Mengingat ab = 1 dan cd = 3 maka e = −4 sehingga d = −1 dan a − . 
32 Maka b = − dan c = −3. Jadi, (a, b, c, d, e) = (−23 , 
−32 , −3, −1, −4) merupakan penyelesaian. 
d b, c, d, e) adalah 
(23 ,32 
, 3, 1, 4) dan (−23 ,−32 
Jai, penyelesaian (a, , −3, −1, −4). 
Con 
Sele 
x2 − yz = 3 
2 − xy = 5 
− xy − xz − yz = 12 
toh 77 : saikan sistem persamaan : y2 − xz = 4 
z 
Solusi : 
Alternatif 1 : 
x2 + y2 + z2
Pembinaan Olimpiade Matematika 
Eddy Hermanto, ST Aljabar 
42 
2x2 + 2y2 + 2z2 − 2xy − 2xz − 2yz = 24 
2) + (x2 − 2xz + z2) + (y2 − 2yz + z2) = 24 
− y)2 + (x − z)2 + (y − z)2 = 24 ⋅⋅⋅⋅⋅⋅⋅⋅⋅⋅⋅ (1) 
(x2 − yz) = 1 
z(y − x) = 1 
⋅⋅⋅⋅⋅⋅⋅⋅⋅⋅⋅ (2) 
(2), (3) dan (4) ke persamaan (1) 
(x2 − 2xy + y 
(x 
y2 − xz − 
(y + x)(y − x) + 
(y − x)(x + y + z) = 1 ⋅⋅⋅⋅⋅⋅⋅⋅⋅⋅⋅⋅⋅⋅⋅⋅ 
z2 − xy − (y2 − xz) = 1 
(z + y)(z − y) + x(z − y) = 1 
(z − y)(x + y + z) = 1 ⋅⋅⋅⋅⋅⋅⋅⋅⋅⋅⋅⋅⋅⋅⋅⋅⋅⋅⋅⋅⋅⋅⋅⋅⋅⋅⋅ (3) 
z2 − xy − (x2 − yz) = 2 
(z + x)(z − x) + y(z − x) = 2 
(z − x)(x + y + z) = 2 ⋅⋅⋅⋅⋅⋅⋅⋅⋅⋅⋅⋅⋅⋅⋅⋅⋅⋅⋅⋅⋅⋅⋅⋅⋅⋅⋅ (4) 
Subtitusikan persamaan 
()21 
zyx++ + ()21zyx++ + ()22zyx++ = 24 
(x + y + z)2 = 41 
• Jika x + y + z = 2 
1 
Dari persamaan (2), (3) dan (4) 
y − x = 2 
z − y = 2 
z − x = 4 x + (2 + x) + z = 21 
2x + z = −23 
2x + (4 + x) = 
−23 
sehingga x = −611 
y = 2 + (−611 ) = 61 z = 4 + (−611) = 613 
• Jika x + y + z = 
−21 
) 
y − x = −2 z − y = −2 z − x = −4x + (−2 + x + z =−21 2x + z = 23 
2x + (−4 + x) = 23 
sehingga x = 611 
y = −2 + (611 ) = 61 z = −4 + (611) = 613 
−611,61,613 )( 611,−61,−613 Tripel (x, y, z) yang memenuhi adalah (( )) 
Alternat 
x2 − y 2 2xz 
(x − − y) + x(z − y) = 0 
− 
Subtitusikan ke persamaan pada soal didapat 
y) = 3 
⋅ (5) 
if 2 : yz + z2 − x = 2y − y)(x + y) + (z − y)(z + y) + z(x (x + y + z)(x + z 2y) = 0 * Jika x + y + z = 0 
x2 − y(−x − 
x2 + y2 + xy = 3 ⋅⋅⋅⋅⋅⋅⋅⋅⋅⋅⋅⋅⋅⋅⋅⋅ 
(−x − y)2 − xy = 5
Pembinaan Olimpiade Matematika 
Eddy Hermanto, ST Aljabar 
43 
x2 + y2 + xy = 5 ⋅⋅⋅⋅⋅⋅⋅⋅⋅⋅⋅⋅⋅⋅⋅⋅⋅⋅⋅⋅⋅⋅ (6) 
pel (x, y, z) yang memenuhi. 
* 
) 
e persamaan (7) maka 
ak memenuhi x2 − yz = 3 
ersamaan (7) maka 
Maka tidak ada tri 
Jika x + z = 2y ⋅⋅⋅⋅⋅⋅⋅⋅⋅⋅⋅⋅⋅⋅⋅⋅⋅⋅⋅⋅⋅⋅⋅ (7) 
x2 − y(2y − x) = 3 
x2 − 2y2 + xy = 3 ⋅⋅⋅⋅⋅⋅⋅⋅⋅⋅⋅⋅⋅⋅⋅⋅⋅⋅⋅ (8 
y2 − x(2y − x) = 4 
x2 + y2 − 2xy = 4 ⋅⋅⋅⋅⋅⋅⋅⋅⋅⋅⋅⋅⋅⋅⋅⋅⋅⋅⋅⋅ (9) 
4(x2 − 2y2 + xy) = 3(x2 + y2 − 2xy) 
x2 − 11y2 + 10xy = 0 
(x + 11y)(x − y) = 0 
• Jika x = y 
Subtitusikan k 
x = y sehingga x = y = z yang tid 
• Jika x = −11y 
Subtitusikan ke p 
z = 13y 
y2 − xz = 4 
y2 + 143y2 = 4 
144y2 = 4 
y = ± 
6y = 1 
* Jika 61 
Maka x = 
−611 dan z = 613 
* Jika y 
= −61 Mk a a x = 611 dan z = −613 
Tripel (x, y, z) yang memenuhi adalah (( 
−611,61,613 )( 611,−61,−613 )) 
LATIHAN 6 : 
1. − n ,− i-jari ⋅⋅⋅⋅⋅⋅ 
2. entukan semua nilai x + y real yang memenuhi sistem persamaan : 
x + y + 
Lingkaran yang melalui titik-titik (2,2), (2,4)da (51) mempunyai jar 
Tyx = 232 dan 
() yyxx+ = 2007 
4. (OSK 2005) Diberikan tiga bilang if ng s berbeda. Jika 
3. Tentukan semua penyelesaian pasangan (x, y) real yang memenuhi x2 + y2 + x + y = 12 
xy + x + y = 3 an posit x, y dan z yaemuanya zxy− 
= zyx+ = yx , 
maka nilai yx sama dengan 
5. 1969) Tunjukkan bahwa jika 
( 
Canadian MO332211baba ba == dan p1, p2 dan p3 semuanya tidak sama 
dengan nol, maka : nnnnnnn 
p b p b p b 
p a p a p a 
b 
a 
1 1 2 2 3 3 
1 1 2 2 3 3 
1 
1 
+ + 
+ + 
= ⎟ ⎟⎠ 
⎞ 
⎜ ⎜⎝ 
⎛
Pembinaan Olimpiade Matematika 
Eddy Hermanto, ST Aljabar 
44 
6. (AIME 1989/OSN 2004) Diberikan x1 + 4x2 + 9x3 + 16x4 + 25x + 36x + 49x = 1 4x 
5 6 7 
x4 + 36x5 + 49x6 + 64x7 = 12 
9x1 + 16x2 + 25x3 + 36x4 + 49x5 + 64x6 + 81x7 = 123. 
Tentukan nilai dari 16x1 + 25x2 + 36x3 + 49x4 + 64x5 + 81x6 + 100x7 
7. (Irish MO 1999) Selesaikan sistem persamaan berikut : 
8. ) yang memenuhi bahwa salah satu bilangan jika 
alah 2. 
menuhi sistem persamaan : 
1 + 9x2 + 16x3 + 25 
. 
y2 = (x + 8)(x2 + 2) 
y2 − (8 + 4x)y + (16 + 16x − 5x2) = 0 
(Canadian MO 1970) Tentukan semua tripel (x, y, z 
ditambahkan dengan hasil kali kedua bilangan yang lain hasilnya ad 
9. (COMC 1997) Tentukan bilangan real x, y dan z yang me 
42=−yzx 6=−xzy 
30−=−xyz 
10.(Vietnamese MO 1996) Selesaikan sistem persamaan berikut : 2113=⎟⎟⎠ ⎞ ⎜⎜⎝ ⎛ + + yxx 24117=⎟⎟⎞ 
⎠ 
⎜ ⎜⎝ 
⎛ 
+ 
− 
x y 
y 
7. KETAKSAMAAN 
A. Konsep Urutan dan Sifat-sifat dasar dari konsep urutan 
Sifat penting pada bilangan-bilangan real adalah adanya urutan sehingga dapat membandingkan dua 
ilangan sehingga didapat apakah kedua bilangan tersebut sama atau tidak sama. 
Sifat-sifat dari dari konsep urutan pada sistem bilangan real : 
(1) Setiap bilangan real a hanya memenuhi satu dan hanya satu dari kemungkinan 
(i) a = 0 
0 
(3) dan b > 0 maka a + b > 0 
(4) dan b > 0 atau a < 0 dan b < 0 maka ab > 0 
(5) dan b < c maka a < c 
(7) dan c < d maka a + c < b + d 
(8) dan c > 0 maka ac < bc 
(9) dan c < 0 maka ac > bc 
b 
(ii) a > 
(iii) a < 0 
(2) Setiap bilangan real a dan b hanya memenuhi satu dan hanya satu dari kemungkinan (i) a = b 
(ii) a > b 
(iii) a < b 
Jika a > 0 
Jika a > 0 
Jika a < b 
(6) Jika a < b maka a ± c < b ± c untuk setiap bilangan real c 
Jika a < b 
Jika a < b 
Jika a < b 
(10)Jika a > 0 maka 
a1 > 0 
(11)Jika a > 0 dan b > 0 maka ba > 0
Pembinaan Olimpiade Matematika 
Eddy Hermanto, ST Aljabar 
45 
ab11<(12)Jika 0 < a < b atau a < b < 0 maka . 
aka a > b. 
nda < diganti dengan tanda ≤ kecuali untuk sifat (12) yang 
d keduanya tak nol. 
sitif yang memenuhi 
(13)Jika a > 0 dan b > 0 serta a > b ma 
22 
Sifat-sifat tersebut juga berlaku jika tmensyaratkan bahwa aan b Contoh 78 : 
db 
a < c maka ddbb 
a < a c < c 
Buktikan bahwa jika a, b, c dan d adalah bilangan po+ olusi : 
+ . 
lternatif 1 : 
sitif serta 
S 
A 
Karena bd podcba< maka 
d + ab < bc + ab ⋅⋅⋅⋅⋅⋅⋅⋅⋅⋅⋅⋅⋅⋅⋅⋅⋅⋅⋅ (sifat 6) 
< b(a + c) 
) positif maka 
ad < bc ⋅⋅⋅⋅⋅⋅⋅⋅⋅⋅⋅⋅⋅⋅⋅ (sifat 7) 
a 
a(b + d) 
Karena b(b + ddbcaba+< ⋅⋅⋅⋅⋅⋅⋅⋅⋅⋅⋅⋅⋅⋅⋅⋅⋅⋅⋅⋅⋅ 7) ⋅⋅⋅⋅⋅⋅ + (sifat ⋅⋅⋅⋅⋅⋅⋅⋅⋅⋅⋅⋅⋅⋅⋅⋅⋅⋅⋅⋅⋅⋅⋅⋅⋅⋅⋅⋅⋅⋅⋅⋅ (1) 
Karena bd positif serta dcba< maka 
⋅⋅⋅⋅⋅⋅⋅⋅⋅⋅⋅⋅⋅⋅⋅⋅⋅⋅⋅ (sifat 6) 
Karena 
ad < bc ⋅⋅⋅⋅⋅⋅⋅⋅⋅⋅⋅⋅⋅⋅⋅ (sifat 7) 
ad + cd < bc + cd 
d(a + c) < c(b + d) d(b + d) positif maka ddb<+ ⋅⋅⋅⋅⋅⋅⋅⋅⋅⋅⋅⋅⋅⋅⋅⋅⋅⋅⋅⋅⋅ 7) ⋅⋅⋅⋅⋅⋅ a+c c (sifat ⋅⋅⋅⋅⋅⋅⋅⋅⋅⋅⋅⋅⋅⋅⋅⋅⋅⋅⋅⋅⋅⋅⋅⋅⋅⋅⋅⋅⋅⋅⋅⋅ (2) 
idapat 
Dari persamaan (1) dan (2) d 
ddbb<<+ (terbukti) ccaa + 
Alternatif 2 : 
Karena dcba< maka bc > ad badbca−+ + = ()dbadc+ b 
b − 
arena bc > ad sedangkan b dan (b + d) positif maka 
K 
bdb+ = ( 
+c − a 
) 
adbb+ 
bc−ad > 0 
Jadi, badba+ 
+c > ⋅⋅⋅⋅⋅⋅ (3) ⋅⋅⋅⋅⋅⋅⋅⋅⋅⋅⋅⋅⋅⋅⋅⋅⋅⋅⋅⋅⋅⋅⋅⋅⋅⋅ dbcadc+ +− = 
()dbdadbc+ − 
Karena bc > ad sedangkan d dan (b + d) positif maka dbcadc+ +−= 
( ) dbd > 0 adbc 
+ 
− 
Jadi, dbcadc+ +> ⋅⋅⋅⋅⋅⋅⋅⋅⋅⋅⋅⋅⋅⋅⋅⋅⋅⋅⋅⋅⋅⋅⋅⋅⋅⋅⋅⋅⋅⋅ (4) 
Dari pers aan (3) dan (4) didapat am 
ddbb+ (terbukti) a < a+c c 
Jika a ≥ b da buktikan bahwa 
< 
Contoh 79 : n x ≤ y maka2byax+ ≤ 2ba+ ⋅ 2yx+
Pembinaan Olimpiade Matematika 
Eddy Hermanto, ST Aljabar 
46 
Solusi : 
2 
ba+ ⋅ 
2yx+ − 2byax+ = 422byaxbybxayax−−+++ = ()() 4xyba−− Karena a ≥ b dan x≤ y maka (a − b)(y − x) ≥ 0. 
adi, 
J2ba+ ⋅ 2yx+ − 2byax+ ≥ 0. Tanda kesamaan terjadi jika a = b atau x = y. 
erbukti bahwa 
T2byax+ ≤ 2ba+ ⋅ 2yx+ . 
Contoh 80 : Jika 21<+x , maka batas-batas nilai x yang memenuhi adal x 0 − + 2 ah ⋅⋅⋅ 
etaksamaan pada soal memiliki syarat bahwa bilangan dalam tanda akar tidak boleh negatif 
hingga 
≥ −2 yang berakibat syarat pada soal adalah x ≥ −2 ⋅⋅⋅⋅⋅⋅⋅⋅⋅⋅⋅⋅⋅⋅⋅⋅⋅⋅⋅⋅⋅⋅⋅⋅⋅⋅⋅ (1) 
Solusi : 
K 
s e 
x ≥ −10 dan x10+x < 2 + 2+x 
Akar dari suatu bilangan selalu positif sehingga jika dikuadratkan tidak akan mempengaruhi tanda 
aan. 
ketaksam 
x + 10 < 2 + x + 2 + 42+x 
3 < 22+x 
9 < 4x + 8 
x > 41 ⋅⋅⋅⋅⋅⋅⋅⋅⋅⋅⋅⋅⋅⋅⋅⋅⋅⋅⋅⋅⋅⋅⋅⋅⋅⋅⋅⋅ (2) Jadi, batas-batas nilai x yang memenuhi adalah x > 41 . 
B. at Seba ang Bilangan Real Selalu Tak Negatif 
ita ketahui bahwa kuadrat dari suatu bilangan real tidak mungkin negatif. Konsep ini 
uk menyelesaikan suatu persoalan. 
a sebarang bilangan real maka a2 ≥ 0. Tanda kesamaan terjadi hanya jika a = 0. 
2 2 dan b. Kapan tanda kesamaan terjadi ? 
aan terjadi jika a = b 
= 2. Buktikan bahwa ab + bc tidak lebih dari 1. 
b) 
b + bc = 1 − (b − 1)2 
gan kuadrat tidak mungkin negatif maka ab + bc ≤ 1 dengan tanda kesamaan terjadi 
KuadrrSebag k 
aimanapenting untJika : 
Contoh 81 
Buktikan bahwa a + b ≥ 2ab untuk bilangan real a 
Solusi : Karena bilangan kuadrat tidak mungknegatif 
in maka (a − b)2 ≥ 0 
T 
anda kesam22− 
a + b 2ab ≥ 0 
a 
2 + b2 ≥ 2ab (terbukti) 
Contoh 82 : 
Diketahui a, 
b, c > 0 serta a + b + c 
Solusi : 
ab + bc = b(a + c) = b(2 − 
a 
Karena bilan 
jika b = 1. adi, terbukti bahwa ab + bc tidak lebih dari 1. 
J
Pembinaan Olimpiade Matematika 
Eddy Hermanto, ST Aljabar 
47 
Sifat bilangan kuadrat tidak mungkin negatif tidak hanya digunakan untuk menyelesaikan masalah 
i juga menyangkut persamaan. 
HSME 1997) Jika x, y dan z adalah bilangan real yang memenuhi (x − 3)2 + (y − 4)2 + (z − 5)2 = 0 
aka nilai dari x + y + z adalah 
arena bilangan kuadrat tidak mungkin negatif maka penyelesaian (x − 3)2 + (y − 4)2 + (z − 5)2 = 0 
anya didapat jika x − 3 = 0, y − 4 = 0 dan z − 5 = 0. 
esaian (x, y, z) yang memenuhi adalah x = 3, y = 4 dan z = 5. 
C. aan Rataan Kuadrat (QM), Rataan Aritmatik (AM), Rataaan Geometri (GM) dan Rataan 
asing rataan. 
ataan Kuadrat (QM) = 
pertidaksamaan tetap 
Contoh 83 : 
(A 
m 
Solusi : 
K 
h 
Maka, penyel 
Jadi, nilai dari x + y + z = 3 + 4 + 5 = 12. 
Ketaksam 
Harmonik (HM) Perlu dijelaskan terlebih dahulu pengertian masing-m 
Misalkan x1, x2, x3, ⋅⋅⋅, xn adalah bilangan real positif. 
R nx 
xxx222+++ n 
2 
1 2 3 L+ 
(AM) = 
Rataan Aritmatik 
nxxxxn++++L321 
Rataan Geometri (GM) = nnxxxxL321 Rataan Harmonik (HM) = n x x x x 
n 
1 1 1 1 
1 2 3 
+ + +L+ 
n HM dari bilangan-bilan 
Solusi : 
M = 
Contoh 84 : Hitunglah QM, AM, GM dagan 2, 3 dan 7. 
Q 3623732222=++ 
AM = 
3 
2+3+7 = 4 
GM = 3342732=⋅⋅ 
H 
M = 71 3121+ 
3 
+ = 41126 Hubungan antara QM, AM, GM dan HM adalah 
QM ≥ AM ≥ GM ≥ HM 
kesamaan terja ka x1 = x2 = x3 = ⋅⋅⋅ = xn Tandadi ji
Pembinaan Olimpiade Matematika 
Eddy Hermanto, ST Aljabar 
48 
Contoh 85 : 
positif a dan b. 
olusi : 
engan memanfaatkan ketaksamaan AM-GM didapat 
Buktikan bahwa a2 + b2 ≥ 2ab untuk bilangan real 
S 
D222ba+ ≥ 22ba = ab 
aan terjadi jika a2 = b2 sehingga a = b. 
Tentukan nilai minimal dari 
Tanda kesam 
a2 + b2 ≥ 2ab (terbukti) 
Contoh 86 : xzzyyx842++ dan kapan nilai terkecil itu terjadi ? 
e s an AM-GM maka 
Solusi : rdaarkan ketaksama 
B 
xzy842 
x + y + z ≥ 3 ⋅ 3842xzzyyx⋅⋅ = 43 Jadi, nilai minimal dari xzzyyx2+lah 4 8 + ada 43 . 
Tanda kesamaan terjadi jika xzzyyx842== . 
Misalkan xzzyyx842== = k maka 
k3 = xz8 = z 
y 
y 
x 
2 4 ⋅ ⋅ 641 k = 41 Jadi, y2 = x 
4 sehingga y = 2x 1 
x8 = z 
4 sehingga zJadi, tanda k 
1 = 2x 
esamaan terjadi jika y = z = 2x 
nilai min l ri Makaimada xzzyyx842++ adalah 43 
yang terjadi jika y = z = 2x. 
Untuk a, b dan c bilangan positif, buktikan ketaksamaan (a + b)(b + c)(c + a) ≥ 8abc. 
n AM ≥ GM 
Contoh 87 : 
Solusi : Berdasarkan ketaksamaamaka 2ba+ ≥ ab 
a + b ≥ 2ab 
Dengan cara yang sama maka 
a + c ≥ 2ac 
b + c ≥ 2bc 
Tanda kesam an terjadi jika a= b = c. 
(b + c) ≥ 2 
a(a + b)(a + c)ab ⋅ 2 ac ⋅ 2 bc 
(a + b)(b + c)(c + a) ≥ 8abc (terbukti)
Pembinaan Olimpiade Matematika 
Eddy Hermanto, ST Aljabar 
49 
Contoh 88 : Buktikan bahwa untuk x, y, z positif maka x2 + y2 + z2 ≥ xy + xz + yz dan kapan tanda kesamaan 
olusi : 
erdasarkan ketaksamaan AM-GM maka 
2 + y2 ≥ 2xy 
2 + z2 ≥ 2xz 
n ketiga persamaan didapat 
x2 + 2y2 + 2z2 ≥ 2xy + 2xz + 2yz 
z2 ≥ xy + xz + yz (terbukti) 
inimal dari 
terjadi. 
S 
B 
x 
x 
y2 + z2 ≥ 2yz 
Tanda kesamaan terjadi jika x = y = z Jumlahka 
2 
x2 + y2 + 
Contoh 89 : 
(OSP 2009/AIME 1983) Tentukan nilai m xxxxsin4sin922+ untuk 0 < x < π. 
Solusi : 
xxxxsin4sin922+ = 9x sin x + xxsin4 
Berdasarkan ketaksamaan AM-GM maka 
xxxxsin4sin922+ = 9x sin x + xxsin4 ≥ xxxxsin4sin92⋅ = 12 
Maka nilai minimum dari xxxxsin4sin922+ sama dengan 12. 
LATIHAN 7 : 
1. (OSK 2010) Bilangan bulat yang memenuhi pertidaksamaan x4 ≤ 8x2 − 16 sebanyak ⋅⋅⋅⋅⋅ 
2. SP 2004) Tentukan semua bilangan real x yang memenuhi x2< ⎪2x − 8⎪ 
3. IME 1987) Tentukan bilangan bulat terbesar n sehingga terdapat bilangan unik k yang memenuhi 
(O 
(A137 < < n+k 
n . 
158 
4. (India RMO 1995) Tunjukkan bahwa untuk sembarang bilangan real x maka 0cossin2122>+++xxxxx 
. (OSP 2009) Banyaknya bilangan real x yang memenuhi persamaan x − 2x3 + 5x2 − 176x + 2009 = 0 
6. Selesaikan persamaan berikut dalam bilangan real 
4 
5adalah ⋅⋅⋅⋅⋅⋅ v 
uzyxvuzyx++++=++−++22 
7. altic Way 2000) Tentukan semua bilangan real positif (x, y) yang memenuhi persamaan : 
(B 
x + y − x1 − y1 + 4 = 2()1212−+−yx
JUDUL
JUDUL
JUDUL
JUDUL
JUDUL
JUDUL
JUDUL
JUDUL
JUDUL
JUDUL
JUDUL
JUDUL
JUDUL
JUDUL
JUDUL
JUDUL
JUDUL
JUDUL
JUDUL
JUDUL
JUDUL
JUDUL
JUDUL
JUDUL
JUDUL
JUDUL
JUDUL
JUDUL
JUDUL
JUDUL
JUDUL
JUDUL
JUDUL
JUDUL
JUDUL
JUDUL
JUDUL
JUDUL
JUDUL
JUDUL
JUDUL
JUDUL
JUDUL
JUDUL
JUDUL
JUDUL
JUDUL
JUDUL
JUDUL
JUDUL
JUDUL
JUDUL
JUDUL
JUDUL
JUDUL
JUDUL
JUDUL
JUDUL
JUDUL
JUDUL
JUDUL
JUDUL
JUDUL
JUDUL
JUDUL
JUDUL
JUDUL
JUDUL
JUDUL
JUDUL
JUDUL
JUDUL
JUDUL
JUDUL
JUDUL
JUDUL
JUDUL
JUDUL
JUDUL
JUDUL
JUDUL
JUDUL
JUDUL
JUDUL
JUDUL
JUDUL
JUDUL
JUDUL
JUDUL
JUDUL
JUDUL
JUDUL
JUDUL
JUDUL
JUDUL
JUDUL
JUDUL
JUDUL
JUDUL
JUDUL
JUDUL
JUDUL
JUDUL
JUDUL
JUDUL
JUDUL
JUDUL
JUDUL

More Related Content

What's hot

Koefisien binomial
Koefisien binomialKoefisien binomial
Koefisien binomialoilandgas24
 
Transformasi (ppt)
Transformasi (ppt)Transformasi (ppt)
Transformasi (ppt)Mathbycarl
 
Bank soal-olimpiade-matematika
Bank soal-olimpiade-matematikaBank soal-olimpiade-matematika
Bank soal-olimpiade-matematikaokto feriana
 
Modul 1 bilangan bulat
Modul 1 bilangan bulatModul 1 bilangan bulat
Modul 1 bilangan bulatAcika Karunila
 
Pertemuan 3 relasi & fungsi
Pertemuan 3 relasi & fungsiPertemuan 3 relasi & fungsi
Pertemuan 3 relasi & fungsiaansyahrial
 
Trigonometri kelas XI
Trigonometri kelas XITrigonometri kelas XI
Trigonometri kelas XIinsan budiman
 
10 Strategi Pemecahan Masalah Matematika
10 Strategi Pemecahan Masalah Matematika10 Strategi Pemecahan Masalah Matematika
10 Strategi Pemecahan Masalah MatematikaRudi Hartono
 
Sistem bilangan bulat (ma kul teori bilangan)
Sistem bilangan bulat (ma kul teori bilangan)Sistem bilangan bulat (ma kul teori bilangan)
Sistem bilangan bulat (ma kul teori bilangan)Ig Fandy Jayanto
 
Soal dan Pembahasan Soal Geometri Olimpiade SMA
Soal dan Pembahasan Soal Geometri Olimpiade SMASoal dan Pembahasan Soal Geometri Olimpiade SMA
Soal dan Pembahasan Soal Geometri Olimpiade SMASuci Agustina
 
Modul Pembelajaran Kapita Selekta Matematika
Modul Pembelajaran Kapita Selekta MatematikaModul Pembelajaran Kapita Selekta Matematika
Modul Pembelajaran Kapita Selekta MatematikaAdelia Ibrahim
 
ppt pertidaksamaan linear satu variabel
ppt pertidaksamaan linear satu variabelppt pertidaksamaan linear satu variabel
ppt pertidaksamaan linear satu variabelNuurwashilaah -
 
20 Pembuktian Teorema Pythagoras oleh Kelompok 1
20 Pembuktian Teorema Pythagoras oleh Kelompok 120 Pembuktian Teorema Pythagoras oleh Kelompok 1
20 Pembuktian Teorema Pythagoras oleh Kelompok 1Rahma Siska Utari
 
Sistem Pertidaksamaan Dua Variabel
Sistem Pertidaksamaan Dua VariabelSistem Pertidaksamaan Dua Variabel
Sistem Pertidaksamaan Dua VariabelFranxisca Kurniawati
 

What's hot (20)

Koefisien binomial
Koefisien binomialKoefisien binomial
Koefisien binomial
 
Transformasi (ppt)
Transformasi (ppt)Transformasi (ppt)
Transformasi (ppt)
 
Ring
RingRing
Ring
 
11. soal soal lingkaran
11. soal soal lingkaran11. soal soal lingkaran
11. soal soal lingkaran
 
Turunan Fungsi Kompleks
Turunan Fungsi KompleksTurunan Fungsi Kompleks
Turunan Fungsi Kompleks
 
Bank soal-olimpiade-matematika
Bank soal-olimpiade-matematikaBank soal-olimpiade-matematika
Bank soal-olimpiade-matematika
 
Modul 1 bilangan bulat
Modul 1 bilangan bulatModul 1 bilangan bulat
Modul 1 bilangan bulat
 
Geometri netral (Neutral Geometry)
Geometri netral (Neutral Geometry)Geometri netral (Neutral Geometry)
Geometri netral (Neutral Geometry)
 
1 Bilangan Kompleks
1 Bilangan Kompleks1 Bilangan Kompleks
1 Bilangan Kompleks
 
Pertemuan 3 relasi & fungsi
Pertemuan 3 relasi & fungsiPertemuan 3 relasi & fungsi
Pertemuan 3 relasi & fungsi
 
Trigonometri kelas XI
Trigonometri kelas XITrigonometri kelas XI
Trigonometri kelas XI
 
10 Strategi Pemecahan Masalah Matematika
10 Strategi Pemecahan Masalah Matematika10 Strategi Pemecahan Masalah Matematika
10 Strategi Pemecahan Masalah Matematika
 
Sistem bilangan bulat (ma kul teori bilangan)
Sistem bilangan bulat (ma kul teori bilangan)Sistem bilangan bulat (ma kul teori bilangan)
Sistem bilangan bulat (ma kul teori bilangan)
 
Soal dan Pembahasan Soal Geometri Olimpiade SMA
Soal dan Pembahasan Soal Geometri Olimpiade SMASoal dan Pembahasan Soal Geometri Olimpiade SMA
Soal dan Pembahasan Soal Geometri Olimpiade SMA
 
Modul Pembelajaran Kapita Selekta Matematika
Modul Pembelajaran Kapita Selekta MatematikaModul Pembelajaran Kapita Selekta Matematika
Modul Pembelajaran Kapita Selekta Matematika
 
Struktur aljabar-2
Struktur aljabar-2Struktur aljabar-2
Struktur aljabar-2
 
ppt pertidaksamaan linear satu variabel
ppt pertidaksamaan linear satu variabelppt pertidaksamaan linear satu variabel
ppt pertidaksamaan linear satu variabel
 
20 Pembuktian Teorema Pythagoras oleh Kelompok 1
20 Pembuktian Teorema Pythagoras oleh Kelompok 120 Pembuktian Teorema Pythagoras oleh Kelompok 1
20 Pembuktian Teorema Pythagoras oleh Kelompok 1
 
Sistem Pertidaksamaan Dua Variabel
Sistem Pertidaksamaan Dua VariabelSistem Pertidaksamaan Dua Variabel
Sistem Pertidaksamaan Dua Variabel
 
Geometri euclid
Geometri euclidGeometri euclid
Geometri euclid
 

Viewers also liked

Hand Out Pembinaan Olimpiade Matematika SMA
Hand Out Pembinaan Olimpiade Matematika SMAHand Out Pembinaan Olimpiade Matematika SMA
Hand Out Pembinaan Olimpiade Matematika SMAputeriaprilianti
 
Kumpulan soal olimpiade matematika sma
Kumpulan soal olimpiade matematika smaKumpulan soal olimpiade matematika sma
Kumpulan soal olimpiade matematika smaRisca Wentiari
 
49758806 diktat-pembinaan-olimpiade-matematika-versi-4
49758806 diktat-pembinaan-olimpiade-matematika-versi-449758806 diktat-pembinaan-olimpiade-matematika-versi-4
49758806 diktat-pembinaan-olimpiade-matematika-versi-4AliSurahman
 
Pembahasan Olimpiade Matematika SMA Babak Penyisihan
Pembahasan Olimpiade Matematika SMA Babak PenyisihanPembahasan Olimpiade Matematika SMA Babak Penyisihan
Pembahasan Olimpiade Matematika SMA Babak Penyisihanhimatika_jaya
 
Contoh soal-olimpiade-matematika-smama-aime-omits-dll
Contoh soal-olimpiade-matematika-smama-aime-omits-dllContoh soal-olimpiade-matematika-smama-aime-omits-dll
Contoh soal-olimpiade-matematika-smama-aime-omits-dllNur Ahmad Abrori
 
Bab I teori bilangan
Bab I teori bilanganBab I teori bilangan
Bab I teori bilanganHaryono Yono
 
Buku siap osn matematika smp 2015
Buku siap osn matematika smp 2015Buku siap osn matematika smp 2015
Buku siap osn matematika smp 2015Wayan Sudiarta
 
26416792 draft-diktat-pembinaan-olimpiade-matematika-sma-n-5-bengkulu-versi-2...
26416792 draft-diktat-pembinaan-olimpiade-matematika-sma-n-5-bengkulu-versi-2...26416792 draft-diktat-pembinaan-olimpiade-matematika-sma-n-5-bengkulu-versi-2...
26416792 draft-diktat-pembinaan-olimpiade-matematika-sma-n-5-bengkulu-versi-2...Mas Nur
 
Pembahasan semifinal omits 2013
Pembahasan semifinal omits 2013Pembahasan semifinal omits 2013
Pembahasan semifinal omits 2013tafrikan
 
40 soal dan pembahasan dimensi 3
40 soal dan pembahasan dimensi 340 soal dan pembahasan dimensi 3
40 soal dan pembahasan dimensi 3Mamuk Prasetyo
 
Diktat Pembinaan OM Materi Dasar versi 5.1
Diktat Pembinaan OM Materi Dasar versi 5.1Diktat Pembinaan OM Materi Dasar versi 5.1
Diktat Pembinaan OM Materi Dasar versi 5.1Kia Hti
 
soal dan pembahasan geometri dan trigonometri
soal dan pembahasan geometri dan trigonometri soal dan pembahasan geometri dan trigonometri
soal dan pembahasan geometri dan trigonometri melanisha
 
SOAL MATEMATIKA SD OSN 2016
SOAL MATEMATIKA SD OSN 2016SOAL MATEMATIKA SD OSN 2016
SOAL MATEMATIKA SD OSN 2016MJUNAEDI1961
 
Diknas tot 2013-Geometri Dasar
Diknas tot 2013-Geometri DasarDiknas tot 2013-Geometri Dasar
Diknas tot 2013-Geometri DasarDidik Sadianto
 
1. soal tes i siswa- fix-cover
1. soal tes i  siswa- fix-cover1. soal tes i  siswa- fix-cover
1. soal tes i siswa- fix-coverDidik Sadianto
 
Pengembangan hand out untuk pembinaan olimpiade matematika sma
Pengembangan hand out untuk pembinaan olimpiade matematika smaPengembangan hand out untuk pembinaan olimpiade matematika sma
Pengembangan hand out untuk pembinaan olimpiade matematika smaputeriaprilianti
 

Viewers also liked (20)

Hand Out Pembinaan Olimpiade Matematika SMA
Hand Out Pembinaan Olimpiade Matematika SMAHand Out Pembinaan Olimpiade Matematika SMA
Hand Out Pembinaan Olimpiade Matematika SMA
 
Buku osn 2015-didik
Buku osn  2015-didikBuku osn  2015-didik
Buku osn 2015-didik
 
Kumpulan soal olimpiade matematika sma
Kumpulan soal olimpiade matematika smaKumpulan soal olimpiade matematika sma
Kumpulan soal olimpiade matematika sma
 
49758806 diktat-pembinaan-olimpiade-matematika-versi-4
49758806 diktat-pembinaan-olimpiade-matematika-versi-449758806 diktat-pembinaan-olimpiade-matematika-versi-4
49758806 diktat-pembinaan-olimpiade-matematika-versi-4
 
Pembahasan Olimpiade Matematika SMA Babak Penyisihan
Pembahasan Olimpiade Matematika SMA Babak PenyisihanPembahasan Olimpiade Matematika SMA Babak Penyisihan
Pembahasan Olimpiade Matematika SMA Babak Penyisihan
 
Contoh soal-olimpiade-matematika-smama-aime-omits-dll
Contoh soal-olimpiade-matematika-smama-aime-omits-dllContoh soal-olimpiade-matematika-smama-aime-omits-dll
Contoh soal-olimpiade-matematika-smama-aime-omits-dll
 
Bab I teori bilangan
Bab I teori bilanganBab I teori bilangan
Bab I teori bilangan
 
Buku siap osn matematika smp 2015
Buku siap osn matematika smp 2015Buku siap osn matematika smp 2015
Buku siap osn matematika smp 2015
 
Soal OSK Matematika 2015
Soal OSK Matematika 2015Soal OSK Matematika 2015
Soal OSK Matematika 2015
 
26416792 draft-diktat-pembinaan-olimpiade-matematika-sma-n-5-bengkulu-versi-2...
26416792 draft-diktat-pembinaan-olimpiade-matematika-sma-n-5-bengkulu-versi-2...26416792 draft-diktat-pembinaan-olimpiade-matematika-sma-n-5-bengkulu-versi-2...
26416792 draft-diktat-pembinaan-olimpiade-matematika-sma-n-5-bengkulu-versi-2...
 
Pembahasan semifinal omits 2013
Pembahasan semifinal omits 2013Pembahasan semifinal omits 2013
Pembahasan semifinal omits 2013
 
40 soal dan pembahasan dimensi 3
40 soal dan pembahasan dimensi 340 soal dan pembahasan dimensi 3
40 soal dan pembahasan dimensi 3
 
Diktat Pembinaan OM Materi Dasar versi 5.1
Diktat Pembinaan OM Materi Dasar versi 5.1Diktat Pembinaan OM Materi Dasar versi 5.1
Diktat Pembinaan OM Materi Dasar versi 5.1
 
Kuncijawaban
KuncijawabanKuncijawaban
Kuncijawaban
 
6. barisan deret
6. barisan deret6. barisan deret
6. barisan deret
 
soal dan pembahasan geometri dan trigonometri
soal dan pembahasan geometri dan trigonometri soal dan pembahasan geometri dan trigonometri
soal dan pembahasan geometri dan trigonometri
 
SOAL MATEMATIKA SD OSN 2016
SOAL MATEMATIKA SD OSN 2016SOAL MATEMATIKA SD OSN 2016
SOAL MATEMATIKA SD OSN 2016
 
Diknas tot 2013-Geometri Dasar
Diknas tot 2013-Geometri DasarDiknas tot 2013-Geometri Dasar
Diknas tot 2013-Geometri Dasar
 
1. soal tes i siswa- fix-cover
1. soal tes i  siswa- fix-cover1. soal tes i  siswa- fix-cover
1. soal tes i siswa- fix-cover
 
Pengembangan hand out untuk pembinaan olimpiade matematika sma
Pengembangan hand out untuk pembinaan olimpiade matematika smaPengembangan hand out untuk pembinaan olimpiade matematika sma
Pengembangan hand out untuk pembinaan olimpiade matematika sma
 

Similar to JUDUL

diktat pembinaanommateridasarversi51.pdf
diktat pembinaanommateridasarversi51.pdfdiktat pembinaanommateridasarversi51.pdf
diktat pembinaanommateridasarversi51.pdfromdhonbaehaqi
 
Diktat pembinaan olimpiade_matematika_ma
Diktat pembinaan olimpiade_matematika_maDiktat pembinaan olimpiade_matematika_ma
Diktat pembinaan olimpiade_matematika_maNazaruddinGunadi
 
Soal prediksi un ipa paket 3 2013
Soal prediksi un ipa paket 3 2013Soal prediksi un ipa paket 3 2013
Soal prediksi un ipa paket 3 2013widi1966
 
2010 2011 xii ips1 hartini, martha
2010 2011 xii ips1 hartini, martha2010 2011 xii ips1 hartini, martha
2010 2011 xii ips1 hartini, marthabasukimahatma
 
Bahas osp matematika sma 2011
Bahas osp matematika sma 2011Bahas osp matematika sma 2011
Bahas osp matematika sma 2011Mina Lim
 
Telaah kisi kisi (materi) ukg kompetensi profesional matematika smp 2013 bagi...
Telaah kisi kisi (materi) ukg kompetensi profesional matematika smp 2013 bagi...Telaah kisi kisi (materi) ukg kompetensi profesional matematika smp 2013 bagi...
Telaah kisi kisi (materi) ukg kompetensi profesional matematika smp 2013 bagi...Agoeng Siswantara
 
Persamaan Kuadrat
Persamaan KuadratPersamaan Kuadrat
Persamaan Kuadratgurumenulis
 
Pembahasan un-smk-2009-2010-matematika
Pembahasan un-smk-2009-2010-matematikaPembahasan un-smk-2009-2010-matematika
Pembahasan un-smk-2009-2010-matematikadedyiswanto
 
Pembahasan un-smk-2009-2010-matematika
Pembahasan un-smk-2009-2010-matematikaPembahasan un-smk-2009-2010-matematika
Pembahasan un-smk-2009-2010-matematikadedyiswanto
 
Pembahasan un-matematika-2012-5-paket
Pembahasan un-matematika-2012-5-paketPembahasan un-matematika-2012-5-paket
Pembahasan un-matematika-2012-5-paketIrviana Rozi
 
bahan-ajar-persamaan-kuadrat-smpppt.pptx
bahan-ajar-persamaan-kuadrat-smpppt.pptxbahan-ajar-persamaan-kuadrat-smpppt.pptx
bahan-ajar-persamaan-kuadrat-smpppt.pptxmuhammadfaisal797900
 
26416792 Draft Diktat Pembinaan Olimpiade Matematika Sma N 5 Bengkulu Versi 2
26416792 Draft Diktat Pembinaan Olimpiade Matematika Sma N 5 Bengkulu Versi 226416792 Draft Diktat Pembinaan Olimpiade Matematika Sma N 5 Bengkulu Versi 2
26416792 Draft Diktat Pembinaan Olimpiade Matematika Sma N 5 Bengkulu Versi 2marshel b
 
Jawab soal UNBK matematika SMK 2017 tipe soal A
Jawab soal UNBK matematika SMK 2017 tipe soal AJawab soal UNBK matematika SMK 2017 tipe soal A
Jawab soal UNBK matematika SMK 2017 tipe soal ASepriano Sepriano
 
Soal dan pembahasan un matematika smp 2012 lengkap
Soal dan pembahasan un matematika smp 2012 lengkapSoal dan pembahasan un matematika smp 2012 lengkap
Soal dan pembahasan un matematika smp 2012 lengkapNoviea Rienha
 
Kuncijawaban
KuncijawabanKuncijawaban
Kuncijawabanfondaessa
 
Soal Latihan dan Pembahasan UN Matematika SMK 2017
Soal Latihan dan Pembahasan UN Matematika SMK 2017Soal Latihan dan Pembahasan UN Matematika SMK 2017
Soal Latihan dan Pembahasan UN Matematika SMK 2017Muhtar Muhtar
 
Soal us matematika smk tp. 2015 2016
Soal us matematika smk tp. 2015 2016Soal us matematika smk tp. 2015 2016
Soal us matematika smk tp. 2015 2016Deni Iskandar
 
Soal to-un-2012-matematika-a-mkks-dki-jakarta
Soal to-un-2012-matematika-a-mkks-dki-jakartaSoal to-un-2012-matematika-a-mkks-dki-jakarta
Soal to-un-2012-matematika-a-mkks-dki-jakartanadiasenja
 
Kumpulan materi un sma matematika ipa dan ips [edukasicampus.net]
Kumpulan materi un sma matematika ipa dan ips [edukasicampus.net]Kumpulan materi un sma matematika ipa dan ips [edukasicampus.net]
Kumpulan materi un sma matematika ipa dan ips [edukasicampus.net]DionPratama5
 

Similar to JUDUL (20)

diktat pembinaanommateridasarversi51.pdf
diktat pembinaanommateridasarversi51.pdfdiktat pembinaanommateridasarversi51.pdf
diktat pembinaanommateridasarversi51.pdf
 
Diktat pembinaan olimpiade_matematika_ma
Diktat pembinaan olimpiade_matematika_maDiktat pembinaan olimpiade_matematika_ma
Diktat pembinaan olimpiade_matematika_ma
 
Soal prediksi un ipa paket 3 2013
Soal prediksi un ipa paket 3 2013Soal prediksi un ipa paket 3 2013
Soal prediksi un ipa paket 3 2013
 
2010 2011 xii ips1 hartini, martha
2010 2011 xii ips1 hartini, martha2010 2011 xii ips1 hartini, martha
2010 2011 xii ips1 hartini, martha
 
Bahas osp matematika sma 2011
Bahas osp matematika sma 2011Bahas osp matematika sma 2011
Bahas osp matematika sma 2011
 
Telaah kisi kisi (materi) ukg kompetensi profesional matematika smp 2013 bagi...
Telaah kisi kisi (materi) ukg kompetensi profesional matematika smp 2013 bagi...Telaah kisi kisi (materi) ukg kompetensi profesional matematika smp 2013 bagi...
Telaah kisi kisi (materi) ukg kompetensi profesional matematika smp 2013 bagi...
 
Persamaan Kuadrat
Persamaan KuadratPersamaan Kuadrat
Persamaan Kuadrat
 
Pembahasan un-smk-2009-2010-matematika
Pembahasan un-smk-2009-2010-matematikaPembahasan un-smk-2009-2010-matematika
Pembahasan un-smk-2009-2010-matematika
 
Pembahasan un-smk-2009-2010-matematika
Pembahasan un-smk-2009-2010-matematikaPembahasan un-smk-2009-2010-matematika
Pembahasan un-smk-2009-2010-matematika
 
Pembahasan un-matematika-2012-5-paket
Pembahasan un-matematika-2012-5-paketPembahasan un-matematika-2012-5-paket
Pembahasan un-matematika-2012-5-paket
 
Mat kelas xii ipa
Mat kelas xii ipaMat kelas xii ipa
Mat kelas xii ipa
 
bahan-ajar-persamaan-kuadrat-smpppt.pptx
bahan-ajar-persamaan-kuadrat-smpppt.pptxbahan-ajar-persamaan-kuadrat-smpppt.pptx
bahan-ajar-persamaan-kuadrat-smpppt.pptx
 
26416792 Draft Diktat Pembinaan Olimpiade Matematika Sma N 5 Bengkulu Versi 2
26416792 Draft Diktat Pembinaan Olimpiade Matematika Sma N 5 Bengkulu Versi 226416792 Draft Diktat Pembinaan Olimpiade Matematika Sma N 5 Bengkulu Versi 2
26416792 Draft Diktat Pembinaan Olimpiade Matematika Sma N 5 Bengkulu Versi 2
 
Jawab soal UNBK matematika SMK 2017 tipe soal A
Jawab soal UNBK matematika SMK 2017 tipe soal AJawab soal UNBK matematika SMK 2017 tipe soal A
Jawab soal UNBK matematika SMK 2017 tipe soal A
 
Soal dan pembahasan un matematika smp 2012 lengkap
Soal dan pembahasan un matematika smp 2012 lengkapSoal dan pembahasan un matematika smp 2012 lengkap
Soal dan pembahasan un matematika smp 2012 lengkap
 
Kuncijawaban
KuncijawabanKuncijawaban
Kuncijawaban
 
Soal Latihan dan Pembahasan UN Matematika SMK 2017
Soal Latihan dan Pembahasan UN Matematika SMK 2017Soal Latihan dan Pembahasan UN Matematika SMK 2017
Soal Latihan dan Pembahasan UN Matematika SMK 2017
 
Soal us matematika smk tp. 2015 2016
Soal us matematika smk tp. 2015 2016Soal us matematika smk tp. 2015 2016
Soal us matematika smk tp. 2015 2016
 
Soal to-un-2012-matematika-a-mkks-dki-jakarta
Soal to-un-2012-matematika-a-mkks-dki-jakartaSoal to-un-2012-matematika-a-mkks-dki-jakarta
Soal to-un-2012-matematika-a-mkks-dki-jakarta
 
Kumpulan materi un sma matematika ipa dan ips [edukasicampus.net]
Kumpulan materi un sma matematika ipa dan ips [edukasicampus.net]Kumpulan materi un sma matematika ipa dan ips [edukasicampus.net]
Kumpulan materi un sma matematika ipa dan ips [edukasicampus.net]
 

More from Moh Hari Rusli

Parabola dan Persamaan garis singgung pada parabola
Parabola dan Persamaan garis singgung pada parabolaParabola dan Persamaan garis singgung pada parabola
Parabola dan Persamaan garis singgung pada parabolaMoh Hari Rusli
 
Fungsi Komposisi dan Fungsi Invers
Fungsi Komposisi dan Fungsi InversFungsi Komposisi dan Fungsi Invers
Fungsi Komposisi dan Fungsi InversMoh Hari Rusli
 
soal Olimpiade Matematika SMA Tingkat Nasional PDIM UB 2014
soal Olimpiade Matematika SMA Tingkat Nasional PDIM UB 2014soal Olimpiade Matematika SMA Tingkat Nasional PDIM UB 2014
soal Olimpiade Matematika SMA Tingkat Nasional PDIM UB 2014Moh Hari Rusli
 
Soal Olimpiade Matematika SMA Tingkat Nasional PDIM UB 2013
Soal Olimpiade Matematika SMA Tingkat Nasional PDIM UB 2013Soal Olimpiade Matematika SMA Tingkat Nasional PDIM UB 2013
Soal Olimpiade Matematika SMA Tingkat Nasional PDIM UB 2013Moh Hari Rusli
 
Kunci jawaban Olimpiade Matematika SMA Tingkat Nasional PDIM UNIVERSITAS BRAW...
Kunci jawaban Olimpiade Matematika SMA Tingkat Nasional PDIM UNIVERSITAS BRAW...Kunci jawaban Olimpiade Matematika SMA Tingkat Nasional PDIM UNIVERSITAS BRAW...
Kunci jawaban Olimpiade Matematika SMA Tingkat Nasional PDIM UNIVERSITAS BRAW...Moh Hari Rusli
 
Soal Olimpiade Matematika SMA Tingkat Nasional PDIM UB 2012
Soal Olimpiade Matematika SMA Tingkat Nasional PDIM UB 2012Soal Olimpiade Matematika SMA Tingkat Nasional PDIM UB 2012
Soal Olimpiade Matematika SMA Tingkat Nasional PDIM UB 2012Moh Hari Rusli
 
Tradisi tujuh hari dalam Islam
Tradisi tujuh hari dalam IslamTradisi tujuh hari dalam Islam
Tradisi tujuh hari dalam IslamMoh Hari Rusli
 
Konsep, Hukum, dan Dalil Membaca al qur’an di kuburan
Konsep, Hukum, dan Dalil Membaca al qur’an di kuburanKonsep, Hukum, dan Dalil Membaca al qur’an di kuburan
Konsep, Hukum, dan Dalil Membaca al qur’an di kuburanMoh Hari Rusli
 
Kosep dan Dalil Maulid Nabi Muhammad SAW
Kosep dan Dalil Maulid Nabi Muhammad SAWKosep dan Dalil Maulid Nabi Muhammad SAW
Kosep dan Dalil Maulid Nabi Muhammad SAWMoh Hari Rusli
 
Konsep Kehidupan alam barzakh
Konsep Kehidupan alam barzakhKonsep Kehidupan alam barzakh
Konsep Kehidupan alam barzakhMoh Hari Rusli
 
Konsep dan Dalil tabarruk
Konsep dan Dalil tabarrukKonsep dan Dalil tabarruk
Konsep dan Dalil tabarrukMoh Hari Rusli
 
Konsep dan Dalil Tawasul dan istighatsah
Konsep dan Dalil Tawasul dan istighatsahKonsep dan Dalil Tawasul dan istighatsah
Konsep dan Dalil Tawasul dan istighatsahMoh Hari Rusli
 
Konsep dan Dalil Hadiah pahala al qur’an
Konsep dan Dalil Hadiah pahala al qur’anKonsep dan Dalil Hadiah pahala al qur’an
Konsep dan Dalil Hadiah pahala al qur’anMoh Hari Rusli
 
Konsep dan Dalil Yasin fadhilah
Konsep dan Dalil Yasin fadhilahKonsep dan Dalil Yasin fadhilah
Konsep dan Dalil Yasin fadhilahMoh Hari Rusli
 
KEAJAIBAN DIALOG AL-QUR’AN DENGAN PERKEMBANGAN ILMU PENGETAHUAN
KEAJAIBAN DIALOG AL-QUR’AN DENGAN PERKEMBANGAN ILMU PENGETAHUANKEAJAIBAN DIALOG AL-QUR’AN DENGAN PERKEMBANGAN ILMU PENGETAHUAN
KEAJAIBAN DIALOG AL-QUR’AN DENGAN PERKEMBANGAN ILMU PENGETAHUANMoh Hari Rusli
 
Konsep dan Dalil tentang bid'ah hasanah
Konsep dan Dalil tentang bid'ah hasanahKonsep dan Dalil tentang bid'ah hasanah
Konsep dan Dalil tentang bid'ah hasanahMoh Hari Rusli
 
BSE Pendidikan Kewarganegaraan SMP/MTs Kelas 9
BSE Pendidikan Kewarganegaraan SMP/MTs Kelas 9BSE Pendidikan Kewarganegaraan SMP/MTs Kelas 9
BSE Pendidikan Kewarganegaraan SMP/MTs Kelas 9Moh Hari Rusli
 
BSE BAHASA INDONESIA untuk SMP/MTs Kelas 9
BSE BAHASA INDONESIA untuk SMP/MTs Kelas 9BSE BAHASA INDONESIA untuk SMP/MTs Kelas 9
BSE BAHASA INDONESIA untuk SMP/MTs Kelas 9Moh Hari Rusli
 

More from Moh Hari Rusli (20)

Irisan kerucut
Irisan kerucutIrisan kerucut
Irisan kerucut
 
Parabola dan Persamaan garis singgung pada parabola
Parabola dan Persamaan garis singgung pada parabolaParabola dan Persamaan garis singgung pada parabola
Parabola dan Persamaan garis singgung pada parabola
 
Fungsi Komposisi dan Fungsi Invers
Fungsi Komposisi dan Fungsi InversFungsi Komposisi dan Fungsi Invers
Fungsi Komposisi dan Fungsi Invers
 
soal Olimpiade Matematika SMA Tingkat Nasional PDIM UB 2014
soal Olimpiade Matematika SMA Tingkat Nasional PDIM UB 2014soal Olimpiade Matematika SMA Tingkat Nasional PDIM UB 2014
soal Olimpiade Matematika SMA Tingkat Nasional PDIM UB 2014
 
Soal Olimpiade Matematika SMA Tingkat Nasional PDIM UB 2013
Soal Olimpiade Matematika SMA Tingkat Nasional PDIM UB 2013Soal Olimpiade Matematika SMA Tingkat Nasional PDIM UB 2013
Soal Olimpiade Matematika SMA Tingkat Nasional PDIM UB 2013
 
Kunci jawaban Olimpiade Matematika SMA Tingkat Nasional PDIM UNIVERSITAS BRAW...
Kunci jawaban Olimpiade Matematika SMA Tingkat Nasional PDIM UNIVERSITAS BRAW...Kunci jawaban Olimpiade Matematika SMA Tingkat Nasional PDIM UNIVERSITAS BRAW...
Kunci jawaban Olimpiade Matematika SMA Tingkat Nasional PDIM UNIVERSITAS BRAW...
 
Soal Olimpiade Matematika SMA Tingkat Nasional PDIM UB 2012
Soal Olimpiade Matematika SMA Tingkat Nasional PDIM UB 2012Soal Olimpiade Matematika SMA Tingkat Nasional PDIM UB 2012
Soal Olimpiade Matematika SMA Tingkat Nasional PDIM UB 2012
 
Tradisi tujuh hari dalam Islam
Tradisi tujuh hari dalam IslamTradisi tujuh hari dalam Islam
Tradisi tujuh hari dalam Islam
 
Konsep, Hukum, dan Dalil Membaca al qur’an di kuburan
Konsep, Hukum, dan Dalil Membaca al qur’an di kuburanKonsep, Hukum, dan Dalil Membaca al qur’an di kuburan
Konsep, Hukum, dan Dalil Membaca al qur’an di kuburan
 
Kosep dan Dalil Maulid Nabi Muhammad SAW
Kosep dan Dalil Maulid Nabi Muhammad SAWKosep dan Dalil Maulid Nabi Muhammad SAW
Kosep dan Dalil Maulid Nabi Muhammad SAW
 
Konsep Kehidupan alam barzakh
Konsep Kehidupan alam barzakhKonsep Kehidupan alam barzakh
Konsep Kehidupan alam barzakh
 
ISLAM DAN TRADISI
ISLAM DAN TRADISIISLAM DAN TRADISI
ISLAM DAN TRADISI
 
Konsep dan Dalil tabarruk
Konsep dan Dalil tabarrukKonsep dan Dalil tabarruk
Konsep dan Dalil tabarruk
 
Konsep dan Dalil Tawasul dan istighatsah
Konsep dan Dalil Tawasul dan istighatsahKonsep dan Dalil Tawasul dan istighatsah
Konsep dan Dalil Tawasul dan istighatsah
 
Konsep dan Dalil Hadiah pahala al qur’an
Konsep dan Dalil Hadiah pahala al qur’anKonsep dan Dalil Hadiah pahala al qur’an
Konsep dan Dalil Hadiah pahala al qur’an
 
Konsep dan Dalil Yasin fadhilah
Konsep dan Dalil Yasin fadhilahKonsep dan Dalil Yasin fadhilah
Konsep dan Dalil Yasin fadhilah
 
KEAJAIBAN DIALOG AL-QUR’AN DENGAN PERKEMBANGAN ILMU PENGETAHUAN
KEAJAIBAN DIALOG AL-QUR’AN DENGAN PERKEMBANGAN ILMU PENGETAHUANKEAJAIBAN DIALOG AL-QUR’AN DENGAN PERKEMBANGAN ILMU PENGETAHUAN
KEAJAIBAN DIALOG AL-QUR’AN DENGAN PERKEMBANGAN ILMU PENGETAHUAN
 
Konsep dan Dalil tentang bid'ah hasanah
Konsep dan Dalil tentang bid'ah hasanahKonsep dan Dalil tentang bid'ah hasanah
Konsep dan Dalil tentang bid'ah hasanah
 
BSE Pendidikan Kewarganegaraan SMP/MTs Kelas 9
BSE Pendidikan Kewarganegaraan SMP/MTs Kelas 9BSE Pendidikan Kewarganegaraan SMP/MTs Kelas 9
BSE Pendidikan Kewarganegaraan SMP/MTs Kelas 9
 
BSE BAHASA INDONESIA untuk SMP/MTs Kelas 9
BSE BAHASA INDONESIA untuk SMP/MTs Kelas 9BSE BAHASA INDONESIA untuk SMP/MTs Kelas 9
BSE BAHASA INDONESIA untuk SMP/MTs Kelas 9
 

Recently uploaded

Modul 1.2.a.8 Koneksi antar materi 1.2.pdf
Modul 1.2.a.8 Koneksi antar materi 1.2.pdfModul 1.2.a.8 Koneksi antar materi 1.2.pdf
Modul 1.2.a.8 Koneksi antar materi 1.2.pdfSitiJulaeha820399
 
Modul Ajar Pendidikan Pancasila Kelas 5 Fase C
Modul Ajar Pendidikan Pancasila Kelas 5 Fase CModul Ajar Pendidikan Pancasila Kelas 5 Fase C
Modul Ajar Pendidikan Pancasila Kelas 5 Fase CAbdiera
 
PPT AKSI NYATA KOMUNITAS BELAJAR .ppt di SD
PPT AKSI NYATA KOMUNITAS BELAJAR .ppt di SDPPT AKSI NYATA KOMUNITAS BELAJAR .ppt di SD
PPT AKSI NYATA KOMUNITAS BELAJAR .ppt di SDNurainiNuraini25
 
Tugas 1 pembaruan dlm pembelajaran jawaban tugas tuton 1.docx
Tugas 1 pembaruan dlm pembelajaran jawaban tugas tuton 1.docxTugas 1 pembaruan dlm pembelajaran jawaban tugas tuton 1.docx
Tugas 1 pembaruan dlm pembelajaran jawaban tugas tuton 1.docxmawan5982
 
Sesi 1_PPT Ruang Kolaborasi Modul 1.3 _ ke 1_PGP Angkatan 10.pptx
Sesi 1_PPT Ruang Kolaborasi Modul 1.3 _ ke 1_PGP Angkatan 10.pptxSesi 1_PPT Ruang Kolaborasi Modul 1.3 _ ke 1_PGP Angkatan 10.pptx
Sesi 1_PPT Ruang Kolaborasi Modul 1.3 _ ke 1_PGP Angkatan 10.pptxSovyOktavianti
 
PEMANASAN GLOBAL - MATERI KELAS X MA.pptx
PEMANASAN GLOBAL - MATERI KELAS X MA.pptxPEMANASAN GLOBAL - MATERI KELAS X MA.pptx
PEMANASAN GLOBAL - MATERI KELAS X MA.pptxsukmakarim1998
 
Aksi Nyata Modul 1.1 Calon Guru Penggerak
Aksi Nyata Modul 1.1 Calon Guru PenggerakAksi Nyata Modul 1.1 Calon Guru Penggerak
Aksi Nyata Modul 1.1 Calon Guru Penggeraksupriadi611
 
PELAKSANAAN + Link2 Materi TRAINING "Effective SUPERVISORY & LEADERSHIP Sk...
PELAKSANAAN  + Link2 Materi TRAINING "Effective  SUPERVISORY &  LEADERSHIP Sk...PELAKSANAAN  + Link2 Materi TRAINING "Effective  SUPERVISORY &  LEADERSHIP Sk...
PELAKSANAAN + Link2 Materi TRAINING "Effective SUPERVISORY & LEADERSHIP Sk...Kanaidi ken
 
Perumusan Visi dan Prakarsa Perubahan.pptx
Perumusan Visi dan Prakarsa Perubahan.pptxPerumusan Visi dan Prakarsa Perubahan.pptx
Perumusan Visi dan Prakarsa Perubahan.pptxadimulianta1
 
tugas karya ilmiah 1 universitas terbuka pembelajaran
tugas karya ilmiah 1 universitas terbuka pembelajarantugas karya ilmiah 1 universitas terbuka pembelajaran
tugas karya ilmiah 1 universitas terbuka pembelajarankeicapmaniez
 
Tugas 1 ABK di SD prodi pendidikan guru sekolah dasar.docx
Tugas 1 ABK di SD prodi pendidikan guru sekolah dasar.docxTugas 1 ABK di SD prodi pendidikan guru sekolah dasar.docx
Tugas 1 ABK di SD prodi pendidikan guru sekolah dasar.docxmawan5982
 
Dinamika Hidrosfer geografi kelas X genap
Dinamika Hidrosfer geografi kelas X genapDinamika Hidrosfer geografi kelas X genap
Dinamika Hidrosfer geografi kelas X genapsefrida3
 
Kontribusi Islam Dalam Pengembangan Peradaban Dunia - KELOMPOK 1.pptx
Kontribusi Islam Dalam Pengembangan Peradaban Dunia - KELOMPOK 1.pptxKontribusi Islam Dalam Pengembangan Peradaban Dunia - KELOMPOK 1.pptx
Kontribusi Islam Dalam Pengembangan Peradaban Dunia - KELOMPOK 1.pptxssuser50800a
 
2 KISI-KISI Ujian Sekolah Dasar mata pelajaranPPKn 2024.pdf
2 KISI-KISI Ujian Sekolah Dasar  mata pelajaranPPKn 2024.pdf2 KISI-KISI Ujian Sekolah Dasar  mata pelajaranPPKn 2024.pdf
2 KISI-KISI Ujian Sekolah Dasar mata pelajaranPPKn 2024.pdfsdn3jatiblora
 
BAHAN SOSIALISASI PPDB SMA-SMK NEGERI DISDIKSU TP. 2024-2025 REVISI.pptx
BAHAN SOSIALISASI PPDB SMA-SMK NEGERI DISDIKSU TP. 2024-2025 REVISI.pptxBAHAN SOSIALISASI PPDB SMA-SMK NEGERI DISDIKSU TP. 2024-2025 REVISI.pptx
BAHAN SOSIALISASI PPDB SMA-SMK NEGERI DISDIKSU TP. 2024-2025 REVISI.pptxJamhuriIshak
 
Bab 6 Kreatif Mengungap Rasa dan Realitas.pdf
Bab 6 Kreatif Mengungap Rasa dan Realitas.pdfBab 6 Kreatif Mengungap Rasa dan Realitas.pdf
Bab 6 Kreatif Mengungap Rasa dan Realitas.pdfbibizaenab
 
REFLEKSI MANDIRI_Prakarsa Perubahan BAGJA Modul 1.3.pdf
REFLEKSI MANDIRI_Prakarsa Perubahan BAGJA Modul 1.3.pdfREFLEKSI MANDIRI_Prakarsa Perubahan BAGJA Modul 1.3.pdf
REFLEKSI MANDIRI_Prakarsa Perubahan BAGJA Modul 1.3.pdfirwanabidin08
 
Contoh Laporan Observasi Pembelajaran Rekan Sejawat.pdf
Contoh Laporan Observasi Pembelajaran Rekan Sejawat.pdfContoh Laporan Observasi Pembelajaran Rekan Sejawat.pdf
Contoh Laporan Observasi Pembelajaran Rekan Sejawat.pdfCandraMegawati
 
421783639-ppt-overdosis-dan-keracunan-pptx.pptx
421783639-ppt-overdosis-dan-keracunan-pptx.pptx421783639-ppt-overdosis-dan-keracunan-pptx.pptx
421783639-ppt-overdosis-dan-keracunan-pptx.pptxGiftaJewela
 
Aksi nyata Malaikat Kebaikan [Guru].pptx
Aksi nyata Malaikat Kebaikan [Guru].pptxAksi nyata Malaikat Kebaikan [Guru].pptx
Aksi nyata Malaikat Kebaikan [Guru].pptxsdn3jatiblora
 

Recently uploaded (20)

Modul 1.2.a.8 Koneksi antar materi 1.2.pdf
Modul 1.2.a.8 Koneksi antar materi 1.2.pdfModul 1.2.a.8 Koneksi antar materi 1.2.pdf
Modul 1.2.a.8 Koneksi antar materi 1.2.pdf
 
Modul Ajar Pendidikan Pancasila Kelas 5 Fase C
Modul Ajar Pendidikan Pancasila Kelas 5 Fase CModul Ajar Pendidikan Pancasila Kelas 5 Fase C
Modul Ajar Pendidikan Pancasila Kelas 5 Fase C
 
PPT AKSI NYATA KOMUNITAS BELAJAR .ppt di SD
PPT AKSI NYATA KOMUNITAS BELAJAR .ppt di SDPPT AKSI NYATA KOMUNITAS BELAJAR .ppt di SD
PPT AKSI NYATA KOMUNITAS BELAJAR .ppt di SD
 
Tugas 1 pembaruan dlm pembelajaran jawaban tugas tuton 1.docx
Tugas 1 pembaruan dlm pembelajaran jawaban tugas tuton 1.docxTugas 1 pembaruan dlm pembelajaran jawaban tugas tuton 1.docx
Tugas 1 pembaruan dlm pembelajaran jawaban tugas tuton 1.docx
 
Sesi 1_PPT Ruang Kolaborasi Modul 1.3 _ ke 1_PGP Angkatan 10.pptx
Sesi 1_PPT Ruang Kolaborasi Modul 1.3 _ ke 1_PGP Angkatan 10.pptxSesi 1_PPT Ruang Kolaborasi Modul 1.3 _ ke 1_PGP Angkatan 10.pptx
Sesi 1_PPT Ruang Kolaborasi Modul 1.3 _ ke 1_PGP Angkatan 10.pptx
 
PEMANASAN GLOBAL - MATERI KELAS X MA.pptx
PEMANASAN GLOBAL - MATERI KELAS X MA.pptxPEMANASAN GLOBAL - MATERI KELAS X MA.pptx
PEMANASAN GLOBAL - MATERI KELAS X MA.pptx
 
Aksi Nyata Modul 1.1 Calon Guru Penggerak
Aksi Nyata Modul 1.1 Calon Guru PenggerakAksi Nyata Modul 1.1 Calon Guru Penggerak
Aksi Nyata Modul 1.1 Calon Guru Penggerak
 
PELAKSANAAN + Link2 Materi TRAINING "Effective SUPERVISORY & LEADERSHIP Sk...
PELAKSANAAN  + Link2 Materi TRAINING "Effective  SUPERVISORY &  LEADERSHIP Sk...PELAKSANAAN  + Link2 Materi TRAINING "Effective  SUPERVISORY &  LEADERSHIP Sk...
PELAKSANAAN + Link2 Materi TRAINING "Effective SUPERVISORY & LEADERSHIP Sk...
 
Perumusan Visi dan Prakarsa Perubahan.pptx
Perumusan Visi dan Prakarsa Perubahan.pptxPerumusan Visi dan Prakarsa Perubahan.pptx
Perumusan Visi dan Prakarsa Perubahan.pptx
 
tugas karya ilmiah 1 universitas terbuka pembelajaran
tugas karya ilmiah 1 universitas terbuka pembelajarantugas karya ilmiah 1 universitas terbuka pembelajaran
tugas karya ilmiah 1 universitas terbuka pembelajaran
 
Tugas 1 ABK di SD prodi pendidikan guru sekolah dasar.docx
Tugas 1 ABK di SD prodi pendidikan guru sekolah dasar.docxTugas 1 ABK di SD prodi pendidikan guru sekolah dasar.docx
Tugas 1 ABK di SD prodi pendidikan guru sekolah dasar.docx
 
Dinamika Hidrosfer geografi kelas X genap
Dinamika Hidrosfer geografi kelas X genapDinamika Hidrosfer geografi kelas X genap
Dinamika Hidrosfer geografi kelas X genap
 
Kontribusi Islam Dalam Pengembangan Peradaban Dunia - KELOMPOK 1.pptx
Kontribusi Islam Dalam Pengembangan Peradaban Dunia - KELOMPOK 1.pptxKontribusi Islam Dalam Pengembangan Peradaban Dunia - KELOMPOK 1.pptx
Kontribusi Islam Dalam Pengembangan Peradaban Dunia - KELOMPOK 1.pptx
 
2 KISI-KISI Ujian Sekolah Dasar mata pelajaranPPKn 2024.pdf
2 KISI-KISI Ujian Sekolah Dasar  mata pelajaranPPKn 2024.pdf2 KISI-KISI Ujian Sekolah Dasar  mata pelajaranPPKn 2024.pdf
2 KISI-KISI Ujian Sekolah Dasar mata pelajaranPPKn 2024.pdf
 
BAHAN SOSIALISASI PPDB SMA-SMK NEGERI DISDIKSU TP. 2024-2025 REVISI.pptx
BAHAN SOSIALISASI PPDB SMA-SMK NEGERI DISDIKSU TP. 2024-2025 REVISI.pptxBAHAN SOSIALISASI PPDB SMA-SMK NEGERI DISDIKSU TP. 2024-2025 REVISI.pptx
BAHAN SOSIALISASI PPDB SMA-SMK NEGERI DISDIKSU TP. 2024-2025 REVISI.pptx
 
Bab 6 Kreatif Mengungap Rasa dan Realitas.pdf
Bab 6 Kreatif Mengungap Rasa dan Realitas.pdfBab 6 Kreatif Mengungap Rasa dan Realitas.pdf
Bab 6 Kreatif Mengungap Rasa dan Realitas.pdf
 
REFLEKSI MANDIRI_Prakarsa Perubahan BAGJA Modul 1.3.pdf
REFLEKSI MANDIRI_Prakarsa Perubahan BAGJA Modul 1.3.pdfREFLEKSI MANDIRI_Prakarsa Perubahan BAGJA Modul 1.3.pdf
REFLEKSI MANDIRI_Prakarsa Perubahan BAGJA Modul 1.3.pdf
 
Contoh Laporan Observasi Pembelajaran Rekan Sejawat.pdf
Contoh Laporan Observasi Pembelajaran Rekan Sejawat.pdfContoh Laporan Observasi Pembelajaran Rekan Sejawat.pdf
Contoh Laporan Observasi Pembelajaran Rekan Sejawat.pdf
 
421783639-ppt-overdosis-dan-keracunan-pptx.pptx
421783639-ppt-overdosis-dan-keracunan-pptx.pptx421783639-ppt-overdosis-dan-keracunan-pptx.pptx
421783639-ppt-overdosis-dan-keracunan-pptx.pptx
 
Aksi nyata Malaikat Kebaikan [Guru].pptx
Aksi nyata Malaikat Kebaikan [Guru].pptxAksi nyata Malaikat Kebaikan [Guru].pptx
Aksi nyata Malaikat Kebaikan [Guru].pptx
 

JUDUL

  • 1. DIKTAT PEMBINAAN OLIMPIADE MATEMATIKA TAHUN PELAJARAN 2010-2011 DISUSUN OLEH : EDDY HERMANTO, ST SMA Negeri 5 Bengkulu Jalan Cendana Nomor 20 Bengkulu
  • 2. SINGKATAN AHSME : American High School Math Exam AIME : American Invitational Mathematics Examination APMO : Asian Pasific Mathematical Olympiad ARML : American Regions Mathematics League COMC : Canadian Open Mathematics Challenge Hongkong PSC : Hongkong Preliminary Selection Contest India RMO : India Regional Mathematical Olympiad MATNC : Mu Alpha Theta National Convention ME VXNY : Mathematical Excalibur Volume X Nomor Y NHAC : Niels Henrik Abel Contest OSK : Olimpiade Sains Indonesia SMA/MA Tingkat Kabupaten/Kota OSK SMP/MTs : Olimpiade Sains Indonesia SMP/MTs Tingkat Kabupaten/Kota OSN : Olimpiade Sains Indonesia SMA/MA Tingkat Nasional OSN SMP/MTs : Olimpiade Sains Indonesia SMP/MTs Tingkat Nasional OSP : Olimpiade Sains Indonesia SMA/MA Tingkat Provinsi OSP SMP/MTs : Olimpiade Sains Indonesia SMP/MTs Tingkat Provinsi QAMT : Queensland Association of Mathematics Teacher USAMTS : USA Mathematical Talent Search ii
  • 3. KATA PENGANTAR Alhamdulillah Penulis ucapkan kepada Allah, SWT karena dengan karunia-Nya Penulis dapat menyelesaikan penulisan diktat ini. Diktat ini Penulis tulis dalam rangka mempermudah tugas dalam mempersiapkan siswa-siswa SMA menghadapi olimpiade matematika pada tahap-tahap awal. Menurut pengamatan Penulis, masih ada jurang pemisah yang cukup jauh antara siswa- siswa dari pulau Jawa dengan dari luar pulau Jawa. Masih sangat banyak siswa-siswa di luar pulau Jawa yang belum memahami persoalan-persoalan dasar di bidang Olimpiade Matematika sehingga mengalami kesulitan besar ketika akan menghadapi OSN. Buku ini berusaha menjawab tentang persoalan-persoalan mendasar di bidang Olimpiade Matematika tersebut. Para guru pembina olimpiade diharapkan dalam membina siswa-siswa juga memberikan soal-soal pada tingkatan OSN pada kegiatan umpan balik yang dapat dilaksanakan setelah guru menyelesaikan pembinaan pada setiap babnya. Penulis menyarankan sebelum menyampiakan isi materi buku ini, Pembina Olimpiade di sekolah dapat menyampaikan materi pendahuluan tentang pembuktian langsung maupun tidak langsung serta Strategi Penyelesaian Masalah (Problem Solving Strategies) yang dapat dipelajari dari beberapa buku yang ada seperti Langkah Awal Menuju ke Olimpiade Matematika karya Wono Setya Budhi, Problem-Solving Strategies karya Arthur Engel maupun Problem-Solving Through Problems karya Loren C. Larson. Ucapan terima kasih kepada semua pihak yang telah membantu dalam penyelesaian diktat ini, khususnya kepada isteri tercinta Penulis, Rosya Hastaryta, S. Si, yang telah memberi dukungan yang besar kepada Penulis serta juga telah melahirkan puteri pertama kami, Kayyisah Hajidah, pada tanggal 2 Desember 2009. Penulis merasa bahwa diktat ini masih jauh dari sempurna. Untuk itu Penulis mengharapkan saran dan kritik dari Pembaca yang budiman sebagai bahan perbaikan diktat ini. Akhir kata semoga buku ini dapat bermanfaat yang sebesar-besarnya bagi Pembaca sekalian. Bengkulu, September 2010 EDDY HERMANTO, ST Email : eddyhbkl@yahoo.com iii
  • 4. DAFTAR ISI HALAMAN JUDUL …………………………………………………………………… i SINGKATAN …………………………………………………………………… ii KATA PENGANTAR …………………………………………………………………… iii DAFTAR ISI …………………………………………………………………… iv BAB I ALJABAR 1. Pemfaktoran dan Penguraian …………………… 1 2. Barisan dan Deret …………………… 5 3. Fungsi …………………… 15 4. Suku Banyak …………………… 19 5. Persamaan …………………… 25 6. Sistem Persamaan …………………… 39 7. Ketaksamaan …………………… 44 BAB II TEORI BILANGAN 1. Sifat-Sifat Penjumlahan Dan Perkalian Dua Bilangan …………………… 52 2. Sifat-sifat Keterbagian …………………… 53 3. Uji Habis dibagi …………………… 56 4. Faktor Persekutuan Terbesar (FPB) Dan Persekutuan Terkecil (KPK) …………………… 57 5. Banyaknya Faktor Positif …………………… 60 6. Kekongruenan …………………… 61 7. Bilangan Bulat, Rasional dan Prima …………………… 64 8. Bilangan Kuadrat Sempurna …………………… 68 9. Fungsi Tangga dan Ceiling …………………… 69 BAB III GEOMETRI 1. Trigonometri …………………… 73 2. Garis …………………… 77 3. Segitiga …………………… 79 4. Segi Empat …………………… 93 5. Segi-n Beraturan …………………… 98 6 Lingkaran …………………… 99 BAB IV KOMBINATORIK 1. Kaidah Pencacahan Dan Penjabaran Binom Newton …………………… 105 2. Kejadian dan Peluang Suatu Kejadian, Pengambilan Contoh …………………… 130 Dengan dan Tanpa Pengembalian 3. Prinsip Inklusi Eksklusi, Peluang Kejadian Majemuk …………………… 139 4. Pigeon Hole Principle (Prinsip Lubang Merpati) …………………… 146 KUNCI JAWABAN LATIHAN …………………… 151 iv
  • 5. Pembinaan Olimpiade Matematika Eddy Hermanto, ST Aljabar 1 BAB I ALJABAR 1. PEMFAKTORAN DAN PENGURAIAN Beberapa bentuk pemfaktoran maupun penguraian yang harus diketahui adalah : (i) x2 − y2 = (x + y)(x − y) (ii) x3 − y3 = (x − y)(x2 + xy + y2) (iii) x3 + y3 = (x + y)(x2 − xy + y2) (iv) x3 + y3 + z3 − 3xyz = (x + y + z)(x2 + y2 + z2 − xy − xz − yz) (v) (x + y)(x − y)2 = x3 − x2y − xy2 + y3 (vi) (an − bn) = (a − b)(an-1 + an-2b + an-3b2 + ⋅⋅⋅ + abn-2 + bn-1) dengan n ∈ bilangan asli (vii) (an + bn) = (a + b)(an-1 − an-2b + an-3b2 − ⋅⋅⋅ − abn-2 + bn-1) dengan n ∈ bilangan ganjil (viii) (x + 1)(y + 1)(z + 1) = xyz + xy + xz + yz + x + y + z + 1 (ix) x4 + 4y4 = (x2 + 2y2 + 2xy)(x2 + 2y2 − 2xy) (x) (x + y)2 = x2 + 2xy + y2 (xi) (x + y + z)2 = x2 + y2 + z2 + 2xy + 2xz + 2yz (xii) (x − y)2 = x2 − 2xy + y2 (xiii) (x + y)3 = x3 + y3 + 3xy(x + y) (xiv) (x − y)3 = x3 − y3 − 3xy(x − y) (xv) (x + y)4 = x4 + 4x3y + 6x2y2 + 4xy3 + y3 (xvi) (x − y)4 = x4 − 4x3y + 6x2y2 − 4xy3 + y3 Penguraian bentuk (x + y)n untuk n > 4 dapat menggunakan binomial Newton yang akan diterangkan dalam bagian lain. Berdasarkan bentuk (vi) dan (vii) didapat fakta bahwa (a − b) membagi (an − bn) untuk n asli dan (a + b) membagi (an + bn) untuk n ganjil yang terkadang digunakan untuk menyelesaikan soal pada teori bilangan. Contoh 1 : (OSK 2004 SMP/MTs) Nilai dari LL=−2249505050 Solusi : Perhatikan bahwa a2 − b2 = (a + b)(a − b) maka ()( ) 49505050495050504950505022−+=− ()()1000000100100004950505022==− 000.14950505022=− Contoh 2 : (OSK 2005 SMP/MTs) Salah satu faktor dari 173 − 53 adalah ⋅⋅⋅⋅⋅⋅⋅ A. 5 B. 17 C. 13 D. 273 E. 399 Solusi : Perhatikan bahwa a3 − b3 = (a − b)(a2 + ab + b2) maka 173 − 53 = (17 − 5)(172 + 17 ⋅ 5 + 52) 173 − 53 = 12 ⋅ 399 (Jawaban : E)
  • 6. Pembinaan Olimpiade Matematika Eddy Hermanto, ST Aljabar 2 Contoh 3 : Jika a2 + b2 = 6ab maka tentukan nilai dari baba− + untuk a, b ≠ 0. Solusi : Misalkan x = baba− + maka x2 = ()2baba− + x2 = abbaabba222222−+ ++ Karena a2 + b2 = 6ab maka x2 = abab48 = 2 Jadi, baba− + = 2 Contoh 4 : Hitunglah 32+ − 32−. Solusi : Misalkan X = 32+ − 32− X2 = (2 + 3) + (2 − 3) − 2322− X2 = 4 − 2 X2 = 2 Maka 32+ − 32− = 2 Contoh 5 : (AIME 1989) Nilai dari 313029281⋅⋅⋅+ adalah ⋅⋅⋅⋅⋅⋅ Solusi : Mengingat bahwa 1 + (n − 1)(n)(n + 1)(n + 2) = (n(n + 1) − 1)2 maka 313029281⋅⋅⋅+ = 29 ⋅ 30 − 1 313029281⋅⋅⋅+ = 869. Contoh 6 : Tentukan nilai x yang memenuhi persamaan 32+x − x−7 = 1 Solusi : 32+x − x−7 = 1 Karena bilangan dalam akar harus tak negatif maka penyelesaian persamaan tersebut harus memenuhi syarat −23 ≤ x ≤ 7 32+x = 1 + x−7 2x + 3 = 1 + 7 − x + 2x−7 3x − 5 = 2x−7
  • 7. Pembinaan Olimpiade Matematika Eddy Hermanto, ST Aljabar 3 Kuadratkan kedua ruas dengan syarat 3x − 5 ≥ 0 sebab x−7 ≥ 0 9x2 − 30x + 25 = 4(7 − x) 9x2 − 26x − 3 = 0 (9x + 1)(x − 3) = 0 x = 3 atau x = −91 (tidak memenuhi syarat x ≥ 35) Untuk x = 3 maka 32+x − x−7 = 3 − 2 = 1. Jadi, nilai x yang memenuhi adalah x = 3. Contoh 7 : Jika a2 = 7b + 1945 dan b2 = 7a + 1945 dengan a dan b adalah bilangan real berbeda, maka nilai dari ab adalah ⋅⋅⋅⋅⋅ Solusi : a2 = 7b + 1945 dan b2 = 7a + 1945 a2 − b2 = 7(b − a) Karena a ≠ b maka a + b = −7 a2 + b2 = 7(b + a) + 3890 (a + b)2 − 2ab = 7(a + b) + 3890 49 − 2ab = −49 + 3890 ab = −1896 Contoh 8 : a, b, c dan d adalah bilangan real tak nol yang memenuhi a2 + b2 = 1 c2 + d2 = 1 ac + bd = 0 Buktikan bahwa ab + cd = 0. Solusi : Karena ac + bd = 0 maka ba = −cd ⋅⋅⋅⋅⋅⋅⋅⋅⋅⋅⋅⋅⋅⋅⋅⋅⋅⋅⋅⋅⋅⋅⋅⋅⋅⋅⋅ (1) Misalkan ba = −cd = k maka a = bk dan d = −ck a2 + b2 = 1 b2(k2 + 1) = 1 k2 + 1 = 21b ⋅⋅⋅⋅⋅⋅⋅⋅⋅⋅⋅⋅⋅⋅⋅⋅⋅⋅ (2) c2 + d2 = 1 c2(k2 + 1) = 1 Subtitusikan persamaan (2). b2 = c2 ab + cd = bk ⋅ b + c (−ck) ab + cd = b2k − c2k ab + cd = (b2 − c2)k Karena b2 = c2 maka ab + cd = (b2 − c2)k = 0 Jadi, terbukti bahwa ab + cd = 0.
  • 8. Pembinaan Olimpiade Matematika Eddy Hermanto, ST Aljabar 4 LATIHAN 1 : 1. Jika baba2243− + = 5 maka tentukan nilai dari abba222+ 2. (AIME 1986) Tentukan nilai dari ()()()()765765765765++−+−−+++. 3. Jika 183=+++yxyx dan 152=−−−yxyx, maka x⋅y = ⋅⋅⋅⋅⋅ 4. Tentukan nilai X yang memenuhi ()()⎟⎠⎞ ⎜⎝⎛ −++⎟⎠⎞ ⎜⎝⎛ +−=53535353X 5. Jika diketahui bahwa 4820142+−yy + 1520142−−yy = 9, maka tentukan nilai dari 4820142+−yy − 1520142−−yy. 6. (OSP 2006) Himpunan semua x yang memenuhi (x − 1)3 + (x − 2)2 = 1 adalah ⋅⋅⋅⋅⋅ 7. (Canadian MO 1992) Selesaikan persamaan x2 + ()221+xx = 3. 8. (OSP 2007) Tentukan semua bilangan real x yang memenuhi x4 − 4x3 + 5x2 − 4x + 1 = 0 9. (AIME 1983) w dan z adalah bilangan kompleks yang memenuhi w2 + z2 = 7 dan w3 + z3 = 10. Apakah nilai terbesar yang mungkin dari w + z ? 10. (Baltic Way 1999) Tentukan semua bilangan real a, b, c dan d yang memenuhi sistem persamaan berikut : abc + ab + bc + ca + a + b + c = 1 bcd + bc + cd + db + b + c + d = 9 cda + cd + da + ac + c + d + a = 9 dab + da + ab + bd + d + b + a = 9 11. (AIME 2000) Tentukan tepat kedua akar real persamaan 2000x6 + 100x5 + 10x3 + x − 2 = 0. 12. (AIME 1987) Tentukan nilai dari ()()()()() ()()()()( ) 32452324403242832416324432458324463243432422324104444444444+++++ +++++. 13. (Baltic Way 1993 Mathematical Team Contest) Tentukan semua bilangan bulat n yang memenuhi nn−−+−+46252254625225 adalah bilangan bulat 14. (Canadian MO 1998) Tentukan penyelesaian x real yang memenuhi persamaan : x = xx1− + x1 1− 15. (AIME 1990) Bilangan real a, b, x dan y memenuhi ax + by = 3, ax2 + by2 = 7, ax3 + by3 = 16 dan ax4 + by4 = 42. Tentukan nilai dari ax5 + by5. 16. (OSN 2003 SMP/MTs) Diketahui a + b + c = 0. Tunjukkan bahwa a3 + b3 + c3 = 3abc
  • 9. Pembinaan Olimpiade Matematika Eddy Hermanto, ST Aljabar 5 2. BARISAN DAN DERET 1, 2, 3, 4, 5, ⋅⋅⋅ dikatakan sebagai barisan karena mempunyai suatu pola tertentu dengan rumus suku ke- n adalah n. 1 + 2 + 3 + 4 + ⋅⋅⋅⋅ disebut sebagai deret. Ada beberapa barisan dan deret yang akan dibahas. A. Barisan dan Deret Aritmatika 1. Pengertian, rumus suku ke-n dan rumus Jumlah n suku pertama Barisan aritmatika adalah barisan yang setiap dua suku berurutan memiliki selisih yang konstan. a, a + b, a + 2b, a + 3b ⋅⋅⋅ adalah barisan aritmatika dengan suku pertama = a dan beda = b. Suku ke-n, Un, dirumuskan dengan : Un = a + (n − 1)b Jumlah n bilangan pertama, Sn, dirumuskan dengan Sn = 2n(2a + (n − 1)b) = 2n(a + Un) Contoh 9 : Diketahui barisan 2, 5, 8, 11, ⋅⋅⋅. Tentukan suku ke-10 dan jumlah 4 suku pertama. Solusi : 2, 5, 8, 11, ⋅⋅⋅ adalah barisan aritmatika dengan suku pertama 2 dan beda 3. Suku ke-10, U10 = 2 + (10 − 1) ⋅ 3 = 29 Jumlah 4 suku pertama = ()3)14(2224 ⋅−+⋅⋅ = 26 Contoh 10 : Sebuah barisan jumlah n buah suku pertama dirumuskan dengan Sn = 3n2 − 15n, maka U3 = ⋅⋅⋅⋅ Solusi : Perhatikan bahwa jika kita mengurangkan jumlah n suku pertama, Sn dengan jumlah n − 1 suku pertama, Sn-1, maka akan didapatkan suku ke-n, Un. Jadi, Un = Sn − Sn-1. Un = (3n2 − 15n) − (3(n − 1)2 − 15(n − 1)) Un = 3n2 − 15n − 3n2 + 6n − 3 + 15n − 15 = 6n − 18 Maka U3 = 6(3) − 18 = 0 Cara lain adalah dengan langsung menghitung U3 = S3 − S2. Contoh 11 : Nilai dariΣ= =+ nkk1)32(LL Solusi : Jika nilai k dari 1 sampai n dijalankan didapat (Σ= + nkk132 ) = 5 + 7 + 9 + ⋅⋅⋅ + (2n + 3) merupakan deret aritmatika dengan a = 5 serta b = 2. Sn = ()()()21522−+nn Jadi, = n(Σ= + nkk132 ) 2 + 4n
  • 10. Pembinaan Olimpiade Matematika Eddy Hermanto, ST Aljabar 6 Contoh 12 : (OSK 2006) Pada sebuah barisan aritmatika, nilai suku ke-25 tiga kali nilai suku ke-5. Suku yang bernilai dua kali nilai suku pertama adalah suku ke ⋅⋅⋅⋅⋅⋅ Solusi : u25 = 3(u5), maka a + 24b = 3(a + 4b) sehingga a = 6b un = a + (n − 1)b = 2u1 = 2a 6b + (n − 1)b = 2(6b) n = 7 Suku tersebut adalah suku ke-7 Contoh 13 : Sisi-sisi sebuah segitiga siku-siku membentuk barisan aritmatika. Jika sisi hipotenusa sama dengan 20, maka keliling segitiga tersebut adalah ⋅⋅⋅⋅ Solusi : Karena sisi terpanjang segitiga sama dengan 20 dan membentuk barisan aritmatika maka sisi-sisi segitiga tersebut dapat dimisalkan dengan 20, 20 − x dan 20 − 2x dengan x adalah bilangan positif. Karena ketiga sisi membentuk segitiga siku-siku maka (20 − 2x)2 + (20 − x)2 = 202 400 − 80x + 4x2 + 400 − 40x + x2 = 400 5x2 − 120x + 400 = 0 (x − 4)(x − 20) = 0 x = 4 atau x = 20 Jika x = 20 maka sisi-sisi segitiga tersebut adalah 20, 0 dan −20 yang tidak mungkin merupakan sisi-sisi segitiga. Jika x = 4 maka sisi-sisi segitiga tersebut adalah 20, 16 dan 12 yang membentuk sisi-sisi segitiga siku-siku. Jadi, keliling segitiga tersebut = 20 + 16 + 12 = 48. 2. Suku Tengah Misalkan Ut menyatakan suku tengah dari suatu barisan aritmatika maka : 21nUUtU+= dengan n merupakan bilangan ganjil Contoh 14 : Diketahui 3, ⋅⋅⋅, 13, 15, ⋅⋅⋅ adalah barisan aritmatika. Tentukan suku tengah barisan tersebut. Solusi : 3, ⋅⋅⋅, 13, 15 adalah barisan aritmatika. Maka U1 = a = 3 dan Un = 15. Maka suku tengah, Ut = 21(3 + 15) = 9 3. Sisipan Misalkan setiap dua bilangan berurutan pada barisan aritmatika disisipi k buah bilangan namun tetap membentuk barisan aritmatika. Maka beda barisan tersebut akan memiliki perubahan dengan suku pertama tetap. Misalkan bB = beda barisan yang baru dan bL = beda barisan yang lama. Hubungan keduanya adalah bB = 1+kbL
  • 11. Pembinaan Olimpiade Matematika Eddy Hermanto, ST Aljabar 7 Contoh 15 : Pada setiap dua bilangan berurutan dari barisan 2, 12, 22, 32, 42. ⋅⋅⋅⋅ disisipi sebanyak 4 bilangan. Tentukan suku ke-100 dari barisan yang baru. Solusi : Beda barisan yang baru adalah bB = 14101++=kbL = 2 Suku pertama, a = 2. U100 = a + 99bB = 2 + 99 ⋅ 2 = 200 Suku ke-100 = 200. Jadi, suku ke-100 barisan tersebut adalah 200. 4. Barisan Aritmatika Bertingkat Misalkan ada barisan u1, u2, u3, ⋅⋅⋅, un bukanlah merupakan barisan aitmatika sebab un − un-1 tidak konstan. Tetapi apabila diambil D1(n) = un − un-1 lalu D2(n) = D1(n) − D1(n − 1) dan seterusnya sampai pada suatu saat Dk(n) − Dk(n − 1) bernilai konstan. Maka kita dapat mengambil kesimpulan bahwa rumus jumlah n suku pertama, Sn, barisan tersebut merupakan polinomial pangkat n. Contoh 16 : Diketahui barisan 1, 3, 6, 10, 15, 21, ⋅⋅⋅. Tentukan rumus jumlah n suku pertama, Sn. Solusi : Kalau diperhatikan, barisan 1, 3, 6, 10, 15, 21, ⋅⋅⋅ bukanlah barisan aritmatika. Tetapi rumus suku ke-n barisan tersebut ternyata merupakan rumus jumlah n suku pertama dari barisan 1, 2, 3, ⋅⋅⋅, n yang merupakan barisan aritmatika. Maka kita dapat menyelesaikan soal tersebut dengan menganggapnya merupakan barisan aritmatika bertingkat. n S(n) D1(n) = S(n) – S(n − 1) D2(n) = D1(n) − D1(n − 1) D3(n) = D2(n) − D2(n − 1) 1 1 2 4 3 3 10 6 3 4 20 10 4 1 5 35 15 5 1 Karena D3(n) konstan maka dapat diambil kesimpulan bahwa rumus Sn merupakan polinomial pangkat 3. Misalkan S(n) = an3 + bn2 + cn + d. n S(n) D1(n) = S(n) – S(n − 1) D2(n) = D1(n) − D1(n − 1) D3(n) = D2(n) − D2(n − 1) 1 a+b+c+d 2 8a+4b+2c+d 7a+3b+c 3 27a+9b+3c+d 19a+5b+c 12a+2b 4 64a+16b+4c+d 37a+7b+c 18a+2b 6a 5 125a+25b+5c+d 61a+9b+c 24a+2b 6a Dari kedua tabel didapat bahwa : 6a = 1 ⋅⋅⋅⋅⋅⋅⋅⋅⋅⋅⋅⋅⋅⋅⋅⋅⋅⋅⋅⋅⋅⋅⋅⋅⋅⋅⋅⋅⋅⋅⋅ (1) 12a + 2b = 3 ⋅⋅⋅⋅⋅⋅⋅⋅⋅⋅⋅⋅⋅⋅⋅⋅⋅⋅⋅⋅⋅⋅⋅⋅⋅⋅⋅⋅⋅⋅⋅⋅⋅⋅⋅⋅ (2) 7a + 3b + c = 3 ⋅⋅⋅⋅⋅⋅⋅⋅⋅⋅⋅⋅⋅⋅⋅⋅⋅⋅⋅⋅⋅⋅⋅⋅⋅⋅⋅⋅⋅⋅⋅⋅⋅⋅⋅⋅⋅⋅⋅⋅ (3) a + b + c + d = 1 ⋅⋅⋅⋅⋅⋅⋅⋅⋅⋅⋅⋅⋅⋅⋅⋅⋅⋅⋅⋅⋅⋅⋅⋅⋅⋅⋅⋅⋅⋅⋅⋅⋅⋅⋅⋅⋅⋅⋅⋅⋅⋅⋅⋅ (4) 61=a Dari pers (1) didapat
  • 12. Pembinaan Olimpiade Matematika Eddy Hermanto, ST Aljabar 8 b2122 3 = = − Dari pers (2) didapat Dari pers (3) didapat ()()31697182161373==−−=−−c Dari pers (4) didapat 0162316312161==−−−=−−−d Maka rumus jumlah n 31 n = ()() 621++nnn 61n3 + 21nsuku pertama, S(n) = 2 + B. arisan dan Deret Geometri e-n dan rumus Jumlah n suku pertama iliki perbandingan yang Un = a ⋅ rn-1 uska B 1. Pengertian, rumus suku kBarisan geometri adalah barisan yang setiap dua suku berurutan mem2 konstan. Misalkan a, ar, ar, ⋅⋅⋅ adalah barisan geometri dengan suku pertama = a dan rasio = r maka : Suku ke-n, Un, dirumuskan dengan : Jumlah n bilangan pertama, Sn, dirum n dengan Sn = ( ) 11− −ra r ontoh 17 : risan 2, 6, 18, 54, ⋅⋅⋅ . Tentukan suku ke-5 dan jumlah 4 suku pertama barisan ⋅⋅⋅ adalah contoh barisan geometri dengan suku pertama 2 dan rasio 3. 5-1 n C Diketahui batersebut. Solusi : 2, 6, 18, 54, Suku ke-5, U5 = 2 ⋅ 3 = 162 Jumlah 4 suku pertama = () 13132− ⋅4− = 80 ontoh 18 : geometri diketahui U8 = 36 dan S7 = 52, maka S8 = ⋅⋅⋅⋅⋅ 7 = 52 tika maupun geometri berlaku Sn − Sn−1 = Un. ontoh 19 : n membentuk deret geometri dengan jumlah 65. Jika suku ke-3 dikurangi 20 bilangan-bilangan tersebut adalah a, ar dan ar2. Maka 2 C Pada barisan Solusi : U 8 = 36 dan S Pada barisan aritma S8 − S7 = U8S = 52 + 36 = 88. 8 C Tiga bilanga terbentuklah deret aritmatika, maka rasio barisan tersebut adalah ⋅⋅⋅⋅ Solusi : Misalkan a, ar dan ar − 20 membentuk barisan aritmatika sehingga 2 2ar = a + ar − 202 ⋅⋅⋅⋅⋅⋅⋅⋅⋅⋅⋅⋅⋅⋅⋅⋅⋅ (1) 2ar + 20 = a + ar 2⋅⋅ a + ar + ar = 65 ⋅⋅⋅⋅⋅⋅⋅⋅⋅⋅⋅⋅⋅⋅⋅⋅ (2) 2ar + 20 + ar = 65 ar = 15 ⋅⋅⋅⋅⋅⋅⋅⋅⋅⋅⋅⋅⋅⋅⋅⋅⋅⋅⋅⋅⋅ (3)
  • 13. Pembinaan Olimpiade Matematika Eddy Hermanto, ST Aljabar 9 Subtitusikan persamaan (3) ke (1) didapat 0 = a + 15r ⋅⋅⋅⋅⋅⋅⋅⋅⋅⋅⋅⋅⋅⋅⋅⋅⋅⋅ (4) 5 Subtitusikan persamaan (4) ke (3). (50 − 15r)r = 15 10r − 3r2 = 3 (3r − 1)(r − 3) = 0 r = 31 atau r = 3 Jika r = 3 maka k1 etiga bilangan tersebut adalah 45, 15 dan 5 yang membentuk barisan geometri − dan 45, 15, 15 yang membentuk barisan aritmatika. Jika r = 3 maka ketiga bilangan tersebut adalah 5, 15 dan 45 yang membentuk barisan geometri dan 5, 15, 25 yang membentuk barisan aritmatika. Jadi, rasio barisan tersebut adalah 31 atau 3. 2. isalkan Ut menyatakan suku tengah dari suatu barisan geometri maka : 2 iketahui 2, 6, 18, 54, 162, ⋅⋅⋅⋅ adalah barisan geometri. Tentukan suku tengah dari barisan , 6, 18, 54, 162 adalah barisan geometri. Maka U1 = a = 2 dan Un = 162. Suku Tengah M nt1 dengan n merupakan bilangan ganjil UUU⋅= Contoh 20 : D tersebut. Solusi : 2 Maka suk u tengah, 181622=⋅=tU 3. Misalkan setiap dua bilangan berurutan pada barisan geometri disisipi k buah bilangan namun embentuk barisan geometri. Maka rasio barisan tersebut akan memiliki perubahan Sisipan tetap m dengan suku pertama tetap. Misalkan rB = rasio barisan yang baru dan rL = rasio barisan yang lama. Hubungan keduanya adalah 1+=kLBrr Contoh 21 : ada setiap dua bilangan berurutan dari barisan 2, 32, 512, 8192, ⋅⋅⋅⋅ disisipi sebanyak 3 tukan suku ke-7 dari barisan yang baru. ng baru, P bilangan. Ten Solusi : R asio ya 21641===+kLBrr . a, a = 2. 128 4. tak hingga Suku pertamU7 = ar6 = (2)(26) = Suku ke-7 = 128. Barisan geometri D ari persamaan Sn = () 1−r jika n a rn −1 Æ ∞ maka S∞ = ra−1 dengan syarat −1 < r < 1. an rumus jumlah dari suatu barisan tak hingga dengan suatu syarat Rumus tersebut merupaktertentu.
  • 14. Pembinaan Olimpiade Matematika Eddy Hermanto, ST Aljabar 10 Contoh 22 : entukan nilai dari 2 + 1 + T21 + 41 + ⋅⋅⋅ ak hingga dengan a = 2 dan r = ½ + 1 + ½ + ¼ + ⋅⋅⋅ = S∞ = Solusi : Persoalan di atas termasuk barisan geometri t 2ra−1 . 2 + 1 + ½ + ¼ + ⋅⋅⋅ = 2112− = 4. Maka nilai dari 2 + 1 + 21+ 41 + ⋅⋅⋅ = 4. mlah suatu deret geometri tak hingga adalah 6 dan jumlah dari suku-suku yang bernomor anjil adalah 4. Suku ke-6 barisan tersebut adalah ⋅⋅⋅⋅ sebut adalah a dan rasio r. + ar + ar2 + ⋅⋅⋅ = Contoh 23 : Ju g Solusi : Misalkan suku pertama barisan geometri tak hingga ter ara−1 = 6 ⋅⋅⋅⋅⋅⋅⋅⋅⋅⋅⋅⋅⋅⋅⋅⋅⋅⋅⋅⋅⋅⋅ (1) dengan suku ⋅⋅⋅ Suku-suku yang ganjil adalah a, ar2, ar4, ⋅⋅⋅ yang membentuk barisan tak hinggao r2pertama a dan rasi. 246 a + ar + ar + ar + = 21r− = 4 ⋅⋅⋅ (2) Subtitusikan persamaan a⋅⋅⋅⋅⋅⋅⋅⋅⋅⋅⋅⋅⋅⋅⋅⋅⋅⋅⋅ (1) ke ()()rra+−111 = 4 sehingga r+11 = 32 3 = 2 + 2r r = 2 1 maan di atas ke persamaan (1) didapat a ku ke-6 = U = ar5 = Subtitusikan persaa = 3 Mak su 632 3 . C. Bari Bentuk Tak Hingga Suatu barisan tidak harus masuk ke dalam salah satu dari dua bentuk di atas. Sebagai contoh adalah arisan yang berbentuk 1, 1, 2, 3, 5, 8, 13, 21, ⋅⋅⋅ yang merupakan penjumlahan dari dua bilangan ditanyakan memerlukan pengetahuan terhadap san dan Deret Lainnya serta b sebelumnya. Untuk menyelesaikan persoalan yang pola dari barisan tersebut. Beberapa contoh rumus deret lainnya : 12 + 22 + 32 + ⋅⋅⋅ + n2 = ()( ) 6121+nn n+ 13 + 23 + 33 + ⋅⋅⋅ + n3 = ()()21+nn 2 Berikut ini dibahas bentuk-bentuk tak hingga yang dapat diselesaikan dengan memisalkan dengan itunglah nilai dari suatu variabel. Contoh 24 : HL2222
  • 15. Pembinaan Olimpiade Matematika Eddy Hermanto, ST Aljabar 11 Solusi : Misalkan L2222 = X. Kuadratkan kedua ruas. Pada ruas kiri akan didapatkan bentuk tak hingga semula. Maka 2X = X2 X(X − 2) = 0 Karena L2222 tidak mungkin sama dengan 0 maka L2222 = 2. Jadi, L2222 = : itunglah bentuk tak hingga berikut, 2 Contoh 25 H M323232+ + + . Solusi : isalkan MM323232+ + + = X maka X3 2 + = X X2 − 2X − 3 = 0 (X − 3)(X + 1) = 0 Karena M32 32+ + 32+ tidak mungkin sama dengan −1 maka M323232+ + + = 3. Jadi, M323232+ + + = 3 D. ip Teleskopik rinsip teleskopik banyak digunakan untuk menyederhanakan suatu deret. Ada dua bentuk umum dan perkalian sebagai berikut : PrinsP y ang dikenal, yaitu penjumlahan a. ()()()()()()111134231211aaaaaaaaaaaaaannnnniii−=−+−++−+−+−=−++− = +ΣL nb. 111134231211aaaaaaaaaaaaaannnnnniii++ −= +=⋅⋅⋅⋅⋅=ΠL Contoh 26 : ()− ( − )( − )L( − )( − )( + )( + )( + )L( + )( + ) =L 2006 1 2004 1 6 1 4 1 2 1 2005 1 2003 1 7 1 5 1 3 1 1 1 1 1 1 1 1 1 1 1
  • 16. Pembinaan Olimpiade Matematika Eddy Hermanto, ST Aljabar 12 Solusi : isal S = M ()()()()()()()()()()20061200416141212005120031 71 51311111111111+++++−−−−−LL S = 20062007674523 200520047653 2 ⋅ 4 ⋅ ⋅L⋅ ⋅ ⋅ ⋅ ⋅L⋅ S = 20062007200420052005200467764554232 3 ⋅ ⋅ ⋅ ⋅L⋅ ⋅ ⋅ ⋅⋅ S = 20062007 Cont27 oh : entukan nila T i 211⋅ + 321⋅ + 431⋅ + 541⋅ + ⋅⋅⋅ + 200620051⋅ . prinsip ik. Solusi : Soal di atas merupakan contoh penerapan teleskop2 1⋅1 = 2111 − ; 321⋅ = 3121− ; 431⋅ = 4131− ; ⋅⋅⋅⋅⋅⋅⋅⋅⋅⋅ ; 200620051⋅ = 2006120051− 21⋅ 1 + 32⋅1 + 43⋅ 1 + 54⋅ 1 + ⋅⋅⋅ + 20062005⋅ 1 = ()()()()()20061200515141433221 1 − 1 + 1 − 1 + 1 − 1 + − +L+ − 211⋅ + 321⋅ + 43⋅ + 1541⋅ ⋅⋅⋅ + +2002005⋅ 6 = 1 − 1 2006 1 Jadi, 211⋅ + 321⋅ + 431⋅ + 541⋅ + ⋅⋅⋅ + 2005⋅ 20 06 1 = 20062005 anadian MO 1997) Buktikan bahwa Contoh 28 : (C19991 < 1998199765 43 21⋅⋅⋅⋅L < 441 . Solusi : Misal P = 1998199765 43 21⋅⋅⋅⋅L dan Q = 19991998765432 ⋅ ⋅ ⋅ ⋅L = PQ 1998199765 43 2 1 ⋅ ⋅ ⋅L⋅ ⋅19991998765432 ⋅⋅⋅⋅L = 19991 Jelas bahwa P < Q hingga P2 < P2 < PQ se19991 < 2441. Maka P < 441 P > 1999753⋅⋅⋅⋅L = 3 5 1997 1 19991 Maka didapat 19991 < P < 44 1 1999 1 < 1998 1997 6 5 4 3 2 1 ⋅ ⋅ ⋅L⋅ < 44 1 (terbukti) LATIHAN 2 : 1. ebuah deret aritmatika terdiri dari n suku (ganjil). Jumlah semua sukunya 260, besar suku ngahnya 20, serta beda deret tersebut a U6 = ⋅⋅⋅⋅ S te dalah 3. Maka 2. Perhatikan barisan bilangan 500, 465, 430, 395, ⋅⋅⋅. Suku negatifnya yang pertama adalah ⋅⋅⋅⋅⋅⋅ 11711 ()Σ=+()LL=+Σ 3. Diketahui =17724iik dan Σ = =114iik, maka nilai = deret aritmatika berlaku u2 + u5 + Maka S10 = ⋅⋅⋅⋅⋅ 5. (OSK 2009) Jika xk+1 = xk + 824iik 4. Pada suatu u6 + u9 = 40.21 untuk k = 1, 2, ⋅⋅⋅ dan x1 = 1 maka x1 + x2 + ⋅⋅⋅ + x400 = ⋅⋅⋅⋅⋅⋅⋅
  • 17. Pembinaan Olimpiade Matematika Eddy Hermanto, ST Aljabar 13 32006 dan 2006 6. (OSP 2006) Hasil penjumlahan semua bilangan bulat di antara adalah ⋅⋅ . (OSK 2006) Diketahui a + (a + 1) + (a + 2) + ⋅⋅⋅ + 50 = 1139. Jika a bilangan positif, maka a = ⋅⋅⋅⋅⋅ 1 2 3 98 n+1 n n = 1, ⋅⋅⋅, 97 dan mempunyai 0 1 2 n 11. dan x − 1 adalah tiga suku pertama deret geometri tak hingga, maka jumlah 78. (AIME 1984) Barisan a, a, a, ⋅⋅⋅, a memenuhi a = a + 1 untuk 2, 3, jumlah sama dengan 137. Tentukan nilai dari a2 + a4 + a6 + ⋅⋅⋅ + a98. 9. Misalkan un adalah suku ke-n dari suatu barisan aritmatika. Jika uk = t dan ut = k maka tentukan nilai dari suku ke-(k + t). 10.(OSK 2004) Agar bilangan 2 + 2 + 2 + ⋅⋅⋅ + 2 sedekat mungkin kepada 2004, haruslah n = ⋅ Jika 9 − 7x, 5x − 6 suku-sukunya adalah ⋅⋅⋅ 12. Pada suatu deret tak hingga, suku-suku yang bernomor ganjil berjumlah 9/4 sedangkan suku-suku yang bernomor genap berjumlah 3/4 , maka suku pertamanya adalah ⋅⋅⋅⋅⋅ 13. Batas-batas nilai a supaya deret geometri tak berhingga dengan suku pertama a konvergen dengan jumlah 2 adalah ⋅⋅⋅⋅⋅ 14. (OSP 2006) Afkar memilih suku-suku barisan geometri takhingga 1, 21, 41, 81 , ⋅⋅⋅ untuk membuat barisan geometri tak71 hingga baru yang jumlahnya . Tiga suku pertama pilihan Afkar adalah ⋅⋅⋅⋅⋅ 15. Tentukan jumlah dari LL+−+−+−494278749432 4 16. Tiga buah bilangan merupakan barisan aritmatika. Bila suku tengahnya dikurangi 5, maka terbentuk dengan rasio sama dengan 2. umlah barisan aritmatika itu adalah ⋅⋅⋅⋅⋅ ⋅⋅ 18. gan a dengan 1, maka nilai a1 20. agian Pertama) Diketahui 0 < a < b < c < d adalah bilangan bulat yang memenuhi a, b, c 0 suatu barisan geometri J 17. Tentukan rumus jumlah n suku pertama dari barisan 4, 10, 20, 35, 56, ⋅⋅⋅ (AIME 1992) Misalkan A adalah barisan a1, a2, a3, ⋅⋅⋅ dengan a19 = a92 = 0 dan ΔA didenisikan den barisan a2 − a1, a3 − a2, a4 − a3, ⋅⋅⋅. Jika semua suku-suku barisan Δ(ΔA) samadalah ⋅⋅⋅⋅⋅⋅ 19. (MATNC 2001) Tentukan jumlah 100 bilangan asli pertama yang bukan bilangan kuadrat sempurna. (AIME 2003 B membentuk barisan aritmatika sedangkan b, c, d membentuk barisan geometri. Jika d − a = 3maka tentukan nilai dari a + b + c + d. 21. (OSK 2009) Bilangan bulat positif terkecil n dengan n > 2009 sehingga nn3333321++++ L 22. (AIME 1985) Barisan bilangan bulat a1, a2, a3, ⋅⋅⋅ memenuhi a = an+1 − an untuk n > 0. Jumlah 1492 985 bilangan pertama adalah 1492. Tentukan jumlah merupakan bilangan bulat adalah ⋅⋅⋅⋅⋅⋅⋅⋅⋅ n+2 bilangan pertama adalah 1985 dan jumlah 12001 bilangan pertama.
  • 18. Pembinaan Olimpiade Matematika Eddy Hermanto, ST Aljabar 14 23. Nilai x yang memenuhi persamaan : ...444.....+++=xxxxxx adalah ⋅⋅⋅⋅⋅⋅⋅ 24. Hitunglah nilai dari M43 43+ 3 4 5 + + 25. (OSK 2006/AIME 1990) Barisan 2, 3, 5, 6, 7, 10, 11, ⋅⋅⋅ terdiri dari semua bilangan asli yang bukan kuadrat atau pangkat tiga bilangan bulat. Suku ke-250 barisan tersebut adalah ⋅⋅⋅⋅⋅ 26. HSME 1996) Barisan 1, 2, 1, 2, 2, 1, 2, 2, 2, 1, 2, 2, 2, 2, 1, 2, 2, 2, 2, 2, 1, 2, ⋅⋅⋅ memiliki blok 27. Misalkan f adalah adalah fungsi yang memenuhi f(n) = f(n − 1) + 6− (A angka 1 yang berisi n buah angka 2 pada blok ke-n. Tentukan jumlah 1234 bilangan pertama. 2007Jika f(0) = 1945 maka tentukan f(2007). n untuk setiap n bilangan asli. 28. (NHAC 1997-1998 Second Round) Tentukan nilai dari 199819971996432321xxxxxx 1 1 + 1 ++ L 29. . (OSK 2003) Berapakah hasil perkalian ()()()()()22222432111 2003 1 2002 1 1 1 1− 1 1− −−−L 30. Tentukan jumlah dari : 100991431321211++++++++L x2, x3, ⋅⋅⋅ memenuhi 31. (AIME 2002) Barisan x1, kkkx+=21 . Jika terdapat bilangan berurutan sehingga xm + xm+1 + ⋅⋅⋅ + xn = 291 , maka tentukan semua pasangan (m, n) yang memenuhi. uhi a1 = a = a −1 − a + a − a . tentukan nilai dari a + a + a . 3. (AIME 1998) Barisan 1000, n, 1000 − n, n − (1000 − n), (1000 − n) − (n − (1000 − n), ⋅⋅⋅ dengan n arisan tersebut negatif. Jadi, untuk n = 100 maka barisan tersebut memiliki panjang 3. Tentukan n sehingga panjang barisan tersebut 34. 32. (AIME 2001) Barisan a1, a2, a3, a4, ⋅⋅⋅ memen211, a2 = 375, a3 = 420 dan a4 = 523 serta nnn−2n−3n−4531753975 3 bilangan bulat berakhir ketika bilangan negatif pertama muncul. Sebagai contoh untuk n = 100 maka barisan tersebut adalah 1000, 100, 900, −800. Suku ke-4 b maksimal. (USAMTS 1999-2000 Round 4) Tentukan nilai dari S = 222223222000111++ + 111++ + ⋅⋅⋅ + 111++ 1 2 1999 angan 1133+ − kk , k = 2, 3, 4, ⋅⋅⋅, 100 lebih dari 32 35. (Baltic Way 1992) Buktikan bahwa hasil kali 99 bil .
  • 19. Pembinaan Olimpiade Matematika Eddy Hermanto, ST Aljabar 15 3. Misalkan diketahui fungsi y = f(x) = FUNGSI A. Pengertian xx231− + . k mencari nilai dari f(2) maka cukup mengganti x di ruas kanan dengan 2. Untu Jadi, f(2) = () ()22312− + = −3 lam kelas adalah fungsi kuadrat, yaitu fungsi yang berbentuk y = meny n y maksimum adalah xp = Salah satu fungsi yang dibahas di da f(x) = ax2 + bx + c Nilai x yang ebabkaab2− Nilai y maksimum = ymaks = a(xp)2 + bxp + c atau ymaks = () aacb442−− Terkadang suatu fungsi tidak hanya memiliki satu variabel, tetapi dapat lebih dari satu variabel. f(1, 2) cukup mengganti x = 1 dan y = 2 = bulat positif ke bilangan bulat positif dan idefinisikan dengan : f(ab) = b⋅f(a) + a⋅f(b). Jika f(10) = 19 ; f(12) = 52 dan f(15) = 26. Tentukan ilai dari f(8). ) = 8f(15) + 15f(8) = 8 ⋅ 26 + 15f(8) = 208 + 15f(8) ⋅⋅⋅⋅⋅⋅⋅⋅⋅⋅⋅ (2) 48 = 208 + 15f(8) ) = 36 isalkan f(x) adalah fungsi yang memenuhi ) untuk setiap bilangan real x dan y maka f(x + y) = x + f(y) dan ) f(0) = 2 998) adalah ⋅⋅⋅⋅⋅ f(y) dan f(0) = 2 x) = f(x + 0) = x + f(0) 1998 + 1998) = −1998 + f(1998) i yang tidak terdefinisi untuk x = 0 dengan f(x) + 2f( Sebagai contoh adalah f(x,y) = xy + x2y + y3. Untuk mencaridari persamaan tersebut didapat f(1, 2) = 2 + 2 + 8 12. Contoh 29 : Misal f adalah suatu fungsi yang memetakan dari bilangan d n Solusi : f(120) = f(10 ⋅ 12) = 12f(10) + 10f(12) = 12 ⋅ 19 + 10 ⋅ 52 = 748 ⋅⋅⋅⋅⋅⋅⋅⋅⋅⋅⋅⋅ (1) f(120) = f(8 ⋅ 15 7 Jadi, f(8 Contoh 30 : (AHSME 1998) M (a (b Nilai dari f(1 Solusi : f(x + y) = x + Alternatif 1 : f( f(x) = x + 2 Maka f(1998) = 2000 Alternatif 2 : 2 = f(0) = f(− Maka f(1998) = 2000 Contoh 31 : Jika f(x) adalah fungsx1 ) = 3x. Tentukan f(x). Solusi : f(x) + 2f(x1 ) = 3x ⋅⋅⋅⋅⋅⋅⋅⋅⋅⋅⋅⋅⋅⋅⋅⋅⋅⋅⋅ (1) Jika y = x1 maka
  • 20. Pembinaan Olimpiade Matematika Eddy Hermanto, ST Aljabar 16 f(y1) + 2f(y) = y3 sehingga f(x1 ) + 2f(x) = x3 2f(x1 ) + 4f(x) = x6 ⋅⋅⋅⋅⋅⋅⋅⋅⋅⋅⋅⋅⋅⋅⋅⋅⋅⋅⋅⋅⋅⋅⋅⋅⋅ (2) r a n (2) dengan (1) didapat Kuangkan persmaa3f(x) = x − 3x 6 Jadi, f(x) = xx22− B. osis ungsi komposisi merupakan gabungan lebih dari satu fungsi. isalkan diketahui fungsi f(x) dan g(x). Jika ingin mencari pemetaan suatu nilai terhadap fungsi f(x) jutkan terhadap fungsi g(x), maka akan digunakan fungsi komposisi. ) ditulis sebagai (g(x) o f(x)). − 3x. Tentukan pemetaan x = 2 oleh fungsi f(x) dilanjutkan g(x). (11) = 7 − 3(11) = −26 etaan x = 2 oleh fungsi f(x) dilanjutkan oleh g(x) menghasilkan nilai −26. gan memanfaatkan definisi fungsi komposisi. x + 5) = 7 − 3(3x + 5) = −8 − 9x −26. iketahui f(x) = x + 7 dan (fog)(x) = 5x + 3. Tentukan g(x). x + 3 = g(x) + 7 − 4 ika g(x − 5) = 7x + 3 maka tentukan g(x). ada alternatif 1 ini kita ubah variabel x pada ruas kanan ke dalam (x − 5). + 3 = 7(x − 5) + 38 + 38 y sehingga x = y + 5. aka g(y) = 7(y + 5) + 3 Fungsi Kompi F M y ang hasilnya dilan Pemetaan terhadap fungsi f(x) yang dilanjutkan oleh fungsi g(x Didefinisikan (g(x)of(x)) = g(f(x)). Contoh 32 : Diketahui f(x) = 3x + 5 dan g(x) = 7 Solusi : f(2) = 3(2) + 5 = 11 g Jadi, pem Cara lain adalah den (g(x)of(x)) = g(f(x)) = g(3 Untuk x = 2 maka nilai g(f(2)) = −8 − 9(2) = −26. Jadi, pemetaan x = 2 oleh fungsi f(x) dilanjutkan oleh g(x) adalah Contoh 33 : D Solusi : f(g(x)) = g(x) + 7 5 g(x) = 5x Contoh 34 : J Solusi : Alternatif 1 : P g(x − 5) = 7x Maka g(x) = 7x Alternatif 2 : Kita misalkan x − 5 = M g(y) = 7y + 38 Maka g(x) = 7x + 38
  • 21. Pembinaan Olimpiade Matematika Eddy Hermanto, ST Aljabar 17 Contoh 35 : Diketahui (gof)(x) = 2x2 + 5x − 5 dan f(x) = x − 1. Maka g(x) = ⋅⋅⋅⋅⋅⋅ (f(x)) = 2x2 + 5x − 5 2 + 5x − 5 − 2 + 5x − 5 (x − 1) = 2(x − 1)2 + 9x − 7 1)2 + 9(x − 1) + 2 = y + 1 (y) = 2(y + 1)2 + 5(y + 1) − 5 + 2 iketahui (gof)(x) = 4x2 + 4x dan g(x) = x2 − 1 dan berlaku f(x) > 1 untuk x > Solusi : (gof)(x) = 2x2 + 5x − 5 g g(x − 1) = 2x Alternatif 1 : g(x − 1) = 2(x − 1)2 + 4x g g(x − 1) = 2(x − Maka g(x) = 2x2 + 9x + 2 Alternatif 2 : Misalkan y = x − 1 maka x g g(y) = 2y2 + 9y Maka g(x) = 2x2 + 9x + 2 Contoh 36 : D −21 , maka f(x − 2) 2 + 4x (x)) − 1 = 4x2 + 4x 4x2 + 4x + 1 = (2x + 1)2 uk x > adalah ⋅⋅⋅⋅ Solusi : g(f(x)) = 4x2 (f (f(x))2 = Karena f(x) > 1 unt −21 maka = 2x − 3 C. x) erdasarkan fungsi y = f(x) = f(x) = 2x + 1 f(x − 2) = 2(x − 2) + 1 f(x − 2) = 2x − 3 Jadi, f(x − 2) Fungsi Invers dari y = f( Bxx231− + dari keterangan sebelumnya jika diketahui nilai x kita dengan agaimana caranya bila yang diketahui adalah nilai y dan kita diminta mudah mencari nilai y. Bmencari nilai x untuk nilai y tersebut ? Persoalan ini dapat diselesaikan apabila kita bisa mendapatkan fungsi inversnya yaitu x = f(y). Contoh 37 : Tentukan invers dari fungsi y = f(x) = 3x − 8 Solusi : y = 3x − 8 x = 3 y+8 gsi inversnya adalah f-1(x) = Didapat fun28+x
  • 22. Pembinaan Olimpiade Matematika Eddy Hermanto, ST Aljabar 18 Contoh 38 : Tentukan invers dari fungsi y = f(x) = xx231− + . Solusi : Dari y = xx231− + seh ga (2y + 1) = 3y − 1 didapat 3y − 2yx = x + 1 ing x x = 12+y-113−y ersnya adalah f (x) = Didapat fungsi inv1213 + − x D. mp isi. isalkan f (x) dan g (x) berturut-turut menyatakan fungsi invers dari f(x) dan g(x). Maka o g)−1(x) = (g−1 o f−1)(x) = x Hubungan fungsi invers dengan fungsi koos−1−1 M (f (g o f)−1(x) = (f−1 o g−1)(x) Contoh 39 : Jika f(x) = 5x + 3 dan g(x) xx− + 532 maka tentukan (f o g)−1(x). omposisi akan didapat = Solusi : Alternatif 1 : asarkan keterangan dalam pembahasan mengenai fungsi k Berd (f o g)(x) xx− + 5307 . ri (f o g)(x) tersebut adalah Maka invers da (f o g)−1(x) = 7305+ − ipelajari sebelumnya akan didapat (x) = xx Alternatif 2 : Berdasarkan apa yang d−1 f53−x dan g− (x) = 1235+ − xx sehingga (g−1 o f−1)(x) = 7+x−1 5x−30 LATIHAN 3 : . Jika f(x) = − + 3, maka f(x2) + (f(x))2 − 2f(x) = ⋅⋅⋅⋅⋅⋅⋅ 2. Diketahui g(x) = x + 1 dan (gof)(x) = 3x2 + ⋅⋅⋅⋅⋅⋅⋅ . (OSK 2007) Misalkan f(x) = 2x - 1, dan g(x) = Jadi, didapat (f o g)(x) = (g −1−1 o f)(x). 1 x 4. Maka f(x) = 3x . Jika f(g(x)) = 3, maka x = ⋅⋅⋅⋅⋅⋅⋅⋅⋅ 4. iketahui (fog)(x) = 5x. Jika g(x) = D 151−x , maka f(x) = ⋅⋅⋅⋅⋅⋅⋅⋅ 5. ungsi g(x) = x2 + 2x + 5 dan (f(g(x)) = 3x2 + 6x − 8, maka f(x) = ⋅⋅⋅⋅⋅⋅⋅⋅ 6. ika f(x) = 2x + 1 ; g(x) = 5x + 3 dan h(x) = 7x, maka (fogoh)(x) = ⋅⋅⋅⋅ F 2 J 7. itentukan Dxaxxf − + 2 =1)(. Jika f−1(4) = 1, maka f(3) = ⋅⋅⋅⋅⋅⋅⋅
  • 23. Pembinaan Olimpiade Matematika Eddy Hermanto, ST Aljabar 19 8. ika J 11)(+ −=xxxf dan , maka 12)(1−=−xxg()()=−xfg1o ⋅⋅⋅⋅⋅⋅ = 1 dan (fog)(x) x2 + = 9. Jika f(x) 2 dan berlaku ≥ 0 u 2 4 5 − − + x x x g(x) ntuk x > 2, maka g(x − 3) = ⋅⋅⋅⋅⋅ 10. SK 2003) Misalkan f suatu fungsi yang memenuhi (O()()xxfx21 f x 1 =− + untuk setiap bilangan real x ≠ 0. Berapakah nilai f(2) ? 11. (AHSME 1996) Sebuah fungsi f : Z Æ Z dan memenuhi n + 3 jika n ganjil f(n) = 2n jika n genap Misalka k ada f(f(f(k))) = 27. Tentukan penjumlahan digit-digit dari k. 12. (OSP 20 4) Misalkan s ang memenuhi f(x) f(y) − f(xy) = x + y, untuk setiap bilangan 14. n f adalah fungsi untuk semua bilangan bulat x dan y yang i dari f(3) sama dengan ⋅⋅⋅⋅⋅⋅ n bilangan ganjil dan lah0 febuah fungsi y bulat x dan y. Berapakah nilai f(2004) ? 13. (OSK 2006) Jika f(xy) = f(x + y) dan f(7) = 7, maka f(49) = ⋅⋅⋅⋅ NHAC 1998-1999 Second Round) Misalka ( memenuhi f(x + y) = f(x) + f(y) + 6xy + 1 dan f(−x) = f(x). Nila 15. (OSP 2009) Suatu fungsi f : Z Æ Q mempunyai sifat ())(1)(11xfxfxf− +=+ untuk setiap x ∈ Z. Jika f(2) = 2, maka nilai fungsi f(2009) adalah ⋅⋅⋅⋅⋅ 16. (AIME 1988) Misalkan f(n) adalah kuadrat dari angka n. Misalkan juga f jumlah angka-snya. Tentukan nilai dari f (11). 4. UK . Pengertian Suku Banyak Perhatikan bentuk-bentuk aljabar berikut : 2 + 7 u bar di atas disebut juga dengan suku banyak atau polinom dalam peubah erajat suatu sukubanyak dalam peubah x adalah pangkat tertinggi dari banyak tersebut. B. sebut. Pembagian f(x) oleh p(x) dapat ditulis sebagai berikut : f(x) = p(x) ⋅ g(x) + s(x) 2(n) didefiniskan sebagai f(f(n)), f3(n) sebagai f(f(f(n))) dan seteru1998 U BANYAK SA (i) x − 3x3 (ii) 4x + 6x − 2x 432 (iii) 2x − 7x + 8x + x − 5 54x3 − 8x2 + 3x − 4 (iv)−2x + x + 7 Bentk-bentuk alja (variabel) x. Yang dimaksud d peubah x yang termuat dalam suku Suku banyak pada (i) memiliki derajat 2 sedangkan suku banyak pada (ii), (iii) dan (iv) berturut- turut berderajat 3, 4 dan 5. Pembagian Suku Banyak Sebagaimana pembagian dalam bilangan, pembagian suku banyak pun memiliki kemiripan dengan pembagian pada bilangan ter dengan f(x) adalah suku banyak yang akan dibagi
  • 24. Pembinaan Olimpiade Matematika Eddy Hermanto, ST Aljabar 20 p(x) adalah pembagi ah hasil bagi embagian bilangan, persyaratan s(x) adalah bahwa pangkat tertinggi (derajat) dari p(x). banyak pun mengikuti dalam bilangan. 4x − 2 arena f(x) berderajat 4 maka q(x) akan berderajat 2 sehingga q(x) = ax + bx + c fisen x4 dari f(x) sama dengan 4 maka koefisien x2 dari q(x) juga 4 sehingga a = 4. x2. Kurangkan 4x4 + 3x3 − 2x2 + x − 7 dengan maka koefisien x 2) ⋅ (4x2 − 13x + 58) − 257x + 109. 109. dapat dilakukan dengan cara horner. entukan hasil bagi dan sisanya jika f(x) = x + 2x + 3x − 5 dengan x − 2 g(x) adal s(x) adalah sisa pembagian Sebagaimana dalam p dari s(x) harus kurang Cara pembagian dalam suku Contoh 40 : Tentukan hasil bagi dan sisanya jika 4x4 + 3x3 − 2x2 + x − 7 dibagi x2 + Solusi : f(x) = 4x4 + 3x3 − 2x2 + x − 7 = (x2 + 4x − 2) ⋅ q(x) + s(x) 2 K Kare koe Kalikan 4x2 dengan (x2 + 4x − 2) didapat 4x4 + 16x3 − 8 4x4 + 16x3 − 8x2 didapat −13x3 + 6x2 + x − 7. Karena koefisien x3 sama dengan −13 dari q(x) sama dengan −13 sehingga b = −13. Kalikan −13x dengan (x2 + 4x − 2) didapat −13x3 − 52x2 + 26x. Kurangkan −13x3 + 6x2 + x − 7 dengan −13x4 − 52x2 + 26x didapat 58x2 − 25x − 7. Karena koefisien x2 sama dengan 58 maka konstanta dari q(x) sama dengan 58 sehingga c = 58. Kalikan 58 dengan (x2 + 4x − 2) didapat 58x2 + 232x − 116. Kurangkan 58x2 − 25x − 7 dengan 58x2 + 232x − 116 didapat −257x + 109. Jadi, 4x4 + 3x3 − 2x2 + x − 7 = (x2 + 4x − Maka pembagi dan sisa jika suku banyak 4x4 + 3x3 − 2x2 + x − 7 dibagi x2 + 4x − 2 berturut-turut adalah 4x2 − 13x + 58 dan − 257x + Contoh 40 merupakan pembagian suku banyak dengan cara pembagian bersusun. Apabila pembaginya linier maka pembagian juga Contoh 41 : 32 T Solusi : 2282 5321− 2 1 4 11 + Maka pembagian f(x) = x3 + 2x2 + 3x − 5 oleh x − 2 akan menghasilkan x2 + 4x + 11 dengan sisa 17. C. ari penjelasan sebelumnya telah kita dapatkan bahwa f(x) = p(x) ⋅ g(x) + s(x) x) = x − k maka akan didapat f(x) = (x − k) ⋅ g(x) + s Jadi, didapat suatu teorema bahw )dibagi oleh x − k maka sisanya orema sisa atau dalil sisa. Leb ah Teorema Sisa D Jika diambil p( Jika diambil x = k maka didapat f(k) = s a jika suku banyak f(x adalah f(k). Teorema di atas dikenal dengan nama te ih lanjut dengan cara yang sama didapat bahwa jika f(x) dibagi (ax + b) maka sisanya adal f()b. a − 17
  • 25. Pembinaan Olimpiade Matematika Eddy Hermanto, ST Aljabar 21 Contoh 42 : Tkan entu sisanya jika f(x) = x4 − 6x3 − 6x2 + 8x + 6 dibagi x − 2 lah f(2). jika f(x) = x4 − 6x3 − 6x2 + 8x + 6 dibagi x − 2 adalah −34. D. tnya akan dipelajari pengertian faktor dalam suku anyak. Pengertian faktor dalam suku banyak dapat dinyatakan dalam bentuk teorema faktor (1) pakan akar-akar persamaan f(x) = 0. a ada paling banyak n buah akar real a 3 2 ari f(x). Terbukti. ±24. Kita akan mencoba akar-akar jika ada. Solusi : Dengan teorema sisa akan didapat sisa jika f(x) dibagi x − 2 adaisa = f(2) = 2 S 4 − 6 ⋅ 23 − 6 ⋅ 22 + 8 ⋅ 2 + 6 = −34. Jadi, sisa Coba kerjakan soal di atas dengan cara Horner. Teorema Faktor Setelah mempelajari teorema sisa, maka selanju b berikut : Misalkan f(x) adalah suku banyak. (x − k) merupakan faktor dari f(x) jika dan hanya jika f(k) = 0 Perhatikan bahwa pernyataan di atas merupakan biimplikasi. Sehingga pernyataan di atas memiliki arti : Jika (x − k) merupakan faktor dari f(x) maka f(k) = 0 (2) Jika f(k) = 0 maka (x − k) merupakan faktor dari f(x) Pada contoh di atas memiliki arti juga bahwa k adalah meru Jikaf(x) merupakan suku banyak dalam derajat n makersmaan f(x) = 0. p Contoh 43 : Tunjukkan bahwa (x + 2) merupakan faktor dari f(x) = x4 + 3x3 + 4x2 + 8x + 8. Solusi : 4 f(−2) = (−2) + 3(−2) + 4(−2) + 8(−2) + 8 = 0 arena f(−2) = 0 maka sesuai teorema faktor maka (x + 2) merupakan faktor d K Contoh 44 : entukan semua faktor linier dari x T 4 − 2x3 − 13x + 14x + 24 = 0 Solusi : Kita akan me ncoba menyelesaikannya dengan horner. aktor bulat dari 24 adalah ±1, ±2, ±3, ±4, ±6, ±8, ±12 dan F t ersebut 014111−− 24141321−− 1 −1 −14 0 + Karena sisanya tidak sama dengan 0 maka (x − 1) bukan faktor. 242602−− 2 4141321−− 2 1 0 −13 −12 + Karena sisanya sama dengan 0 maka (x − 2) adalah faktor linier. 4 3 − x = (x − 3 x − 2x 13x + 14 + 24 2)(x − 13x 12) 24 0 −
  • 26. Pembinaan Olimpiade Matematika Eddy Hermanto, ST Aljabar 22 1211 121301−− 1− − 1 −1 −12 + Karena sisanya sama dengan 0 maka x + 1) adalah faktor linier. x4 − 2x3 − 13x + 14x + 24 = (x − 2)(x + 1)(x2 − x − 12) 1 14x + )(x + 3) aka faktor-faktor linier dari x4 − 2x3 − 13x + 14x + 24 adalah (x + 1), (x − 2), (x + 3) dan (x − 4). E. : 1, x2, x3, ⋅⋅⋅, xn, (dengan kata lain x1, x2, x3, ⋅⋅⋅, xn adalah akar-akar p(x) = 0) maka hubungan-t berlaku : ( x4 − 2x3 − 3x + 24 = (x − 2)(x +1)(x − 4 M Teorema Vieta Jika 0112211)(axaxaxaxaxpnnnnnn+++++=− − − −L adalah polinomial dengan pembuat nol x 0 hubungan berikunnaannxxxx x1+ 1 2 3 1 + + + + = − − L − nna annjijixxxxxxxxxxxx2142323121−=++++++=− <Σ LL nnaannnkjikjixxxxxxxxx x x x x x x x x x 3 1 2 3 1 3 4 2 3 4 2 4 5 2 1 = + + + + + + = − − − − < < Σ L L M ()naannnxxxxx011321−=−L Contoh 45 : Persamaan kuadrat x2 + 5x − = 0 memiliki akar-akar x1 dan x2. Tentukan nilai x1 3 + x2 3. olusi : n x2 + 5x − 7 = 0 didapat nilai A = 1, B = 5 dan C = −7 7 S Dari persamaa x1 + x2 = −AB = −5 x1x2 = AC = −7 x − 3x1x2(x1 + x2) x + x = (− 3(−7)(−5) = −125 − 105 3 . banyak x − 5x − 16x + 41x − 15 = 0 memiliki akar-akar a, b, c dan d. Maka nilai ari . a2 + b2 + c2 + d2 x13 + x23 = (x1 + x2)3 − 3x12x2 − 3x1x22 3x13 + x23 = (1 + x2) 1 333− 25) x13 + x2= −230 Contoh 46 : 432 Persamaan suku d a b. a1 + b1 + cadalah ⋅⋅⋅⋅⋅⋅ 1 + + bc + bd + cd = −16 Solusi : a + b + c d =5 ab + ac + ad
  • 27. Pembinaan Olimpiade Matematika Eddy Hermanto, ST Aljabar 23 abc + abd + acd + bcd = −41 . a2 + b2 + c2 + d2 = (a + b + c + d)2 − 2(ab + ac + ad + bc + bd + cd) = 52 − 2(−16) = 57 abcd = −15 a b . a + 1 b + 1 c + 1 d = 1 abcd abc+abd +acd +bcd = 1541− − = 1541 Contoh 47 : (OSP 2005/Canadian Mα, β O 1996) Jika dan γ adalah akar-akar x3 − x − 1 = 0 maka tentukan nilai αα − + 1 1 + ββ − + 11 + γγ − + 11 . Dengan melihat Ax + Bx + Cx + D = 0 dan x x 1 = 0 didapat A = 1, B = 0, C = 1 dan D = 1. Solusi : 323−−−− γβα++ = AB− = 0 βγαγαβ++ = AC = 11− = −1 αβγ = AD = − 1)1(− = 1 − αα − + 11 + ββ − + 11 + γγ = − + 1 1 ()()( ) ( )( )( ) ( )( )( ) ( α )( β )( γ ) γ β α γ γ α β − − − + + − − + + − − 1 1 1 1 1 1 1 1 1 = βα−−+111 ()() ()()αβγβγαγαβγβααβγβγαγαβγβα −+++++− +++−++− 133 = ()()() ()()()110113103−−+− +−−− = −7 LATIHAN 4 : 1. ika f(x) dibagi dengan (x − 2) sisanya 24, sedangkan jika dibagi dengan (x + 5) sisanya 10. Jika f(x) ibagi dengan x2 + 3x − 10 sisanya adalah ⋅⋅⋅⋅⋅⋅ 2. Jika v(x) dibagi x2 − x dan x2 + x berturut-turut akan bersisa 5x + 1 dan 3x + 1, maka bila v(x) dibagi . (OSP 2006) Jika (x − 1)2 membagi ax4 + bx3 + 1, maka ab = ⋅⋅⋅⋅ 4. adalah bilangan-bilangan bulat dan x2 − x − 1 merupakan faktor dari ax3 + bx + 1, maka b = ⋅⋅⋅⋅⋅⋅⋅⋅⋅⋅ 5. HSME 1999) Tentukan banyaknya titik potong maksimal dari dua grafik y = p(x) dan y = q(x) ersebut adalah 1. alah ⋅⋅⋅⋅⋅⋅ J d x2 − 1 sisanya adalah ⋅⋅⋅⋅ 3 (OSK 2008) Jika a dan b 2 (A dengan p(x) dan q(x) keduanya adalah suku banyak berderajat empat dan memenuhi koefisien x4 dari kedua suku banyak t 6. Suku banyak f(x) dibagi (x + 1) sisanya −2 dan dibagi (x − 3) sisanya 7. Sedangkan suku banyak g(x) jika dibagi (x + 1) akan bersisa 3 dan jika dibagi (x − 3) akan bersisa 2. Diketahui h(x) = f(x) ⋅ g(x). Jika h(x) dibagi x2 − 2x − 3, maka sisanya ad
  • 28. Pembinaan Olimpiade Matematika Eddy Hermanto, ST Aljabar 24 7. (OSP 2009) Misalkan p(x) = x2 − 6 dan A = {x ∈ R⏐p(p(x)) = x}. Nilai maksimal dari {⏐x⏐ : x ∈ A} adalah ⋅⋅⋅⋅⋅⋅ 8. Jika persamaan (3x2 − x + 1)3 dijabarkan dalam suku-sukunya maka akan menjadi persamaan a4 + a3 + a2 + a1 + a0 b) a6 − a5 + a4 − a3 + a2 − a1 + a0 9. + x − 2 mempunyai tiga pembuat nol yaitu a, b, dan c. Nilai dari 10. Tentukan semua nilai m sehingga persamaan x4 − (3m + 2)x2 + m2 = 0 memiliki 4 akar real yang 1. (OSP 2008) Misalkan a, b, c, d bilangan rasional. Jika diketahui persamaan x4 + ax3 + bx2 + cx + d = 0 polinomial a6x6 + a5x5 + a4x4 + a3x3 + a2x2 + a1x + a0. Tentukan nilai dari : a) a6 + a5 + c) a6 + a5 + a4 + a3 + a2 + a1 d) a6 + a4 + a2 + a0 (OSK 2010) Polinom P(x) = x3 − x2 a3 + b3 + c3 adalah ⋅⋅⋅⋅⋅⋅⋅⋅ membentuk barisan aritmatika. 12 dan mempunyai 4 akar real, dua di antaranya adalah 2008. Nilai dari a + b + c + d adalah ⋅⋅ 12. (OSP 2010) Persamaan kuadrat x − px − 2p = 0 mempunyai dua akar real α dan β. Jika α + β= 16, maka hasil tambah semua nilai p yang memenuhi adalah ⋅⋅⋅⋅⋅⋅ 2 3 3 13. IME 1996) Akar-akar x3 + 3x2 + 4x − 11 = 0 adalah a, b dan c. Persamaan pangkat tiga dengan akar- 14. (OSK 2003) Misalkan bahwa f(x) = x + ax + bx + cx + dx + c dan bahwa f(1) = f(2) = f(3) = f(4) = 15. (AIME 1993) Misalkan po(x) = x + 313x − 77x − 8 dan pn(x) = pn−1(x − n). Tentukan koefisien x dari 16. (OSP 2009) Misalkan a, b, c adalah akar-akar polinom x3 − 8x2 + 4x − 2. Jika f(x) = x3 + px2 + qx + r 17. (NAHC 1995-1996 Second Round) Misalkan p(x) = x6 + ax5 + bx4 + cx3 + dx2 + ex + f adalah polinomial 18. (OSP 2010) Diberikan polinomial P(x) = x + ax + bx + cx + d dengan a, b, c, dan d konstanta. Jika (A akar a + b, a + c dan b + c adalah x3 + rx2 + sx + t = 0. Tentukan nilai t. 5432 f(5). Berapakah nilai a ? 32 p20(x). adalah polinom dengan akar-akar a + b − c, b + c − a, c + a − b maka f(1) = ⋅⋅⋅⋅⋅⋅ yang memenuhi p(1) = 1, p(2) = 2, p(3) = 3, p(4) = 4, p(5) = 5 dan p(6) = 6. Nilai dari p(7) adalah ⋅⋅⋅⋅⋅ 432 P(1) = 10, P(2) = 20, dan P(3) = 30, maka nilai ()() 10812−+PP = ⋅⋅⋅⋅⋅⋅⋅⋅⋅⋅ (AIME 2003 Bagian Kedua) Akar-akar persamaa 19. n x4 − x3 − x2 − 1 = 0 adalah a, b, c dan d. Tentukan nilai dari p(a) + p(b) + p(c) + p(d) jika p(x) = x6 − x5 − . 20. anadian MO 1970) Diberikan polinomial f(x) = xn + a xn-1 + a2xn-2 + ⋅⋅⋅ + an-1x + an dengan koefisien uhi f(k) = 8. x3 − x2 − x (C 1a1, a2, ⋅⋅⋅, an semuanya bulat dan ada 4 bilangan bulat berbeda a, b, c dan d yang memenuhi f(a) = f(b) = f(c) = f(d) = 5. Tunjukkan bahwa tidak ada bilangan bulat k yang memen
  • 29. Pembinaan Olimpiade Matematika Eddy Hermanto, ST Aljabar 25 5. ERSAMAAN da beberapa persamaan yang akan dibahas, yaitu : . Persamaan Kuadrat rsamaan kuadrat adalah Ax2 + Bx + C = 0. Misalkan x dan x2 adalah nilai x yang memenuhi persamaan kuadrat di atas. Nilai x1 dan x2 gan akar-akar. Maka berlaku. 2) ilai x yang memenuhi dapat dicari dengan cara kuadrat sempurna, dengan menggunakan rumus x1,2 = P A A Bentuk pe 1) Pengertian akar 1dikenal juga den Ax12 + Bx1 + C = 0 Ax22 + Bx2 + C = 0 Menentukan nilai akar-akar persamaan kuadrat Untuk mencari n m emfaktorkan maupunAACBB242−±− sebagaimana yang telah an. eda. rta x dan x keduanya real. 3) dituliskan ke dalam bentuk isalkan terdapat persamaan kuadrat Ax2 + Bx + C = 0 yang memiliki akar-akar x1 dan x2. Maka x adalah sebagai berikut. didapatkan dari pelajaran di kelas. Persamaan B2 − 4AC dikenal dengan nama diskriminan. Nilai diskriminan ini menentukan jenis- jenis akar (nilai x1 dan x2). Ada tiga kemungkinan nilai diskrimin• Jika B2 − 4AC > 0 maka x1 dan x2 keduanya real dan berb• Jika B2 − 4AC = 0 maka x1 = x2 se 12• Jika B2 − 4AC < 0 maka x1 dan x2 keduanya tidak real. Hubungan kedua akar Persamaan kuadrat yang memiliki akar-akar x1 dan x2 dapat persamaan x2 − (x1 + x2)x + x1x2 = 0. M hubungan antara x1 dan 2x1 + x2 = AB− x1 ⋅ x2 = AC 4) an rsamaan kuadrat baru. persamaan kuadrat Ax2 + Bx + C = 0 memiliki akar-akar x1 dan x2. Ada beberapa cara n enentukan persamaan kuadrat yang memiliki akar-akar x3 dan x4 dan memiliki ubungan tertentu dengan x1 dan x2. (x + x )x + x x = 0. x + x4 dan x3x4 ke dalam bentuk x1 + x2 dan x1x2 maka ehingga akan didapat persamaan b. -akar x3 dan x4. 5) M MenentukpeMisalkanjika ingim h a. Membawa ke dalam persamaan x2 − 3434Misalkan terdapat persamaan kuadrat Ax2 + Bx + C = 0 yang memiliki akar-akar x1 dan x2. Dari keterangan sebelumnya akan didapatkan nilai dari x1 + x2 dan x1x2. Jika dapat ditentukan nilai dari 3berarti nilai dari x3 + x4 dan x3x4 dapat ditentukan s kuadrat yang memiliki akar-akar x3 dan x4 yaitu x2 − (x3 + x4)x + x3x4 = 0. Melakukan subtitusi setelah menghilangkan indeks Jika dari hubungan x3 dan x4 yang memiliki hubungan tertentu dengan x1 dan x2 kita hilangkan indeksnya lalu kita subtitusikan ke persamaan semula dan mendapatkan persamaan kuadrat baru. Maka persamaan kudarat tersebut memiliki akar enentukan nilai suatu bilangan yang berbentuk abba2++ dan abba2−+ ka ba+ dan ba− keduanya dikuadratkan akan didapat Ji ()abbaba22++=+ ()abbaba22 − = + −
  • 30. Pembinaan Olimpiade Matematika Eddy Hermanto, ST Aljabar 26 Sehingga dapat ditentukan nilai dari a+ b+2ab dan abba2−+ , yaitu baabba+=++2 baabba−=−+2 dengan syarat a ≥ b. Contoh 48 : Jika salah satu akar x2 + (a + 1)x + (3a + 2) = 0 adalah 5, maka akar lainnya adalah ⋅⋅⋅⋅ Solusi : esuai pengertian akar maka akan didapat 5 + (3a + 2) = 0 1)x − (3(−4) + 2) = 0 an x2 = −2 -akar persamaan cx2 + bx + a = 0, maka S 52 + (a + 1) ⋅ a = −4 ersamaan kuadrat tersebut adalah P x2 + (−4 + x2 − 3x − 10 = 0 (x − 5)(x + 2) = 0 x1 = 5 d Jadi, akar lainnya adalah −2. Contoh 49 : Jika x1 dan x2 akar222111xx+ = ⋅⋅⋅⋅⋅⋅ olusi : S x1 + x2 = −cb x1x2 = ca 2221xx 1 + 1 = ()2212221xxxx+ 2211xx+= () 1 2 ()221212212xxxxxx−+ 222111+ x x = 222aacb− Contoh 50 : Persamaan kuadrat yang akar-akarnya 3 lebihnya dari akar-akar persamaan kuadrat x2 + 5x − 24 = 0 adalah ⋅ olusi : an x2 adalah akar-akar persamaan kuadrat x2 + 5x − 24 = 0. 1 2 an x1x2 = −24 (x1 + x2) + 6 = 1 ⋅⋅⋅⋅⋅⋅ S Misalkan x1 d Maksud soal adalah menentukan persamaan kuadrat yang akar-akarnya x3 = x1 + 3 dan x4 = x2 + 3. Alternatif 1 : + x = −5 d x x3 + x4 = x3 ⋅ x4 = (x1 + 3)(x2 + 3) = x1x2 + 3(x1 + x2) + 9 = −24 − 15 + 9 = −30 Persamaan kuadrat baru adalah x2 − (x3 + x4)x + x3x4 = 0. n kuadrat yang diminta adalah x2 − x − 30 = 0 Jadi, persamaa
  • 31. Pembinaan Olimpiade Matematika Eddy Hermanto, ST Aljabar 27 Alternatif 2 : Misalkan y3 = x1 + 3 dan y4 = x2 + 3 Jika indeks dihilangkan akan didapat y = x + 3. Subtitusikan x = y − 3 ke persamaan semula. − y − 30 = 0 2 − y − 30 = 0 merupakan persamaan kuadrat yang akar-akarnya x1 + 3 dan x2 + 3. n kuadrat yang diminta adalah x2 − x − 30 = 0 (y − 3)2 + 5(y − 3) − 24 = 0 2 y y Jadi, persamaa Contoh 51 : LL=+348 olusi : S262261228348⋅++=+=+ . Memperhatikan baabba+=++2 , maka 26348+=+ Contoh 52 : (AIME 1983) Tentukan hasil kali semua akar-akar real 4518230182+xx + = x2 + x + . olusi : isalkan y = x2 + 18x + 30 dan kuadratkan kedua ruas persamaan semula, maka ) = 0 aka nilai y yang memenuhi hanya y = 10. adi, terdapat dua akar yang tidak real. + 30 = 10 sehingga x2 + 18x + 20 = 0 l yang berbeda sehingga S M y 2 = 4(y + 15) (y − 10)(y + 6Karena akar dari suatu bilangan tak mungkin negatif m J x2 + 18x Jadi, hasil kali kedua akar persamaan x2 + 18x + 20 = 0 sama dengan 20. Contoh 53 : (OSK 2002) Misalkan a dan b bilangan reaabbaba1010+ ++ = 2 T entukan nilai ba. Solusi : Karena abbaba1010+ + + = 2 maka bab101+ = 2 b a a +10 + isa M l xba= , maka xx+101x + 10 = + 19x x= + − 2 − 10x2 (5x − 4) ) = 0 x = 1 atau x 210(x − 1 =54 Jadi, karena a ≠ b, maka x ≠ 1. Jadi, 54=ba
  • 32. Pembinaan Olimpiade Matematika Eddy Hermanto, ST Aljabar 28 LATIHAN 5.A r-akarnya dua lebih besar dari akar-akar x2 + px + 1 = 0 tapi tiga bih ke il dari akar-akar persamaan 2x2 − 3x + q = 0 adalah ⋅⋅⋅⋅ . Jika p = 1. Persamaan kuadrat yang aka lec 22231xxx− + maka batas-batas p supaya x r ⋅⋅⋅ 2 rsamaan kuadrat yang akar-akarnya eal adalah 3. Jika kedua akar persamaan kuadrat x2 − px + p = 0 bernilai real positif, maka batas-batas nilai p yang memenuhi adalah ⋅⋅⋅⋅ 4. Jika x1 dan x akar-akar persamaan x + 2x + 4 = 0, maka pe 211 dan 1 − x 1 1 x2 − entukan adalah ⋅⋅⋅⋅⋅ 5. (OSK 2005) Misalkan a dan b adalah bilangan real taknol yang memenuhi 9a2 − 12ab + 4b2 = 0. T ba . 6. (AIME 1990) Tentukan nilai dari ()()2/32/34365243652−−+ . 7. (AIME 1990) Tentukan penyelesaian positif 6910211222 −10 −29 −10 −45 − − =+ x x x x x x 8. RML 1999 Individual) Jika a dan b adalah akar-akar persamaan kuadrat 11x2 − 4x − 2 = 0. (1 + a + a + )(1 + b + b + ) tuk x2 + bx + c = 0. Berapa banyakkah persamaan demikian yang memiliki akar-akar real jika koefisien b dan c hanya boleh dipilih dari himpunan 10. an bulat sehingga . (Ahitunglah nilai dari : 2 ⋅⋅⋅2 ⋅⋅⋅ 9. (OSP 2002) Tinjau persamaan yang berben {1,2,3,4,5,6} ? (OSK 2010) Jika a dan b bilang200922010+ merupakan solusi kuadrat x2 + ax + b = 0 maka nilai a + b adalah ⋅⋅⋅⋅⋅ (OSK 2010) Diketahui bahwa ada tepat 1 bilangan asli n sehingga n 11. rna. Bilangan asli n tersebut adalah ⋅⋅⋅⋅⋅⋅⋅ ah x1 dan x2 sedan dan v. Jika u + v = −uv maka nilai dari x1 3x2 + x1x2 3 adalah ⋅⋅⋅⋅⋅⋅⋅⋅ 13. )x + 2a + 4b = 0. Jika α = 2β maka nilai dari a + b = ⋅⋅⋅⋅⋅⋅⋅⋅ 14. 2 + n + 2010 merupakan uadrat sempu k12. Akar-akar persamaan kuadrat x2 + 6x + c = 0 adalgkan akar-akar persamaan kuadrat x2 + (x12 + x22)x + 4 = 0 adalah u α dan β adalah akar-akar persamaan kuadrat x2 − 3(a − 12 Jika suku pertama deret geometri tak hingga adalah 2 dan rasionya adalah r = αm = m β untuk ilai m > 0 d n an α, β akar-akar x2 − (3m + 2) + (4m + 12) = 0, maka jumlah deret geometri tak 15. + 1)x + p = 0 dengan akar-akar x1 dan x2 serta 3x2 − (q − 1)x − 1 = 0 dengan kar-akar x3 dan x4. Jika x1x3 = 1 dan x2x4 = 1, maka nilai dari p − 2q + 13 = ⋅⋅⋅⋅⋅⋅⋅⋅⋅⋅ hingga tersebut adalah ⋅⋅⋅ iketahui x Da 2 − (2p
  • 33. Pembinaan Olimpiade Matematika Eddy Hermanto, ST Aljabar 29 16. Jika a ≠ b dan jika persamaan-persamaan x2 + ax + bc = 0 dan x2 + bx + ac = 0 mempunyai tepat sebuah akar persekutuan, tunjukkan bahwa akar-akar yang lain dari kedua persamaan tersebut memenuhi persamaan x2 + cx + ab = 0. α β 2 δ 18. (AIME 1991) Misalkan k adalah penjumlahan semua nilai ilai x yang 17. isalkan dan adalah akar-akar persamaan x + px + 1 = 0 sedangkan γ dan adalah akar-akar persamaan x M 2 + qx + 1 = 0. Buktikan bahwa (α − γ)(β − γ)(α + δ)(β + δ) = q2 − p2. mutlak dari nilai-nmemenuhi 2. xx91199119+ += . Tentukan nilai dari k 9119911919+ + + 91 19. 1, c1, b2 dan c2 adalah bilangan real yang 1 c2). Tunjukkan bahwa sedikitnya satu dari dua persamaan x2 + b1x + c1 = 0 dan x2 + b2x + c2 = 0 memiliki akar-akar real. 20. Diberikan a, b, c ∈ bilangan real serta a dan 4a + 3b + 2c mempunyai tanda yang sama. Tunjukkan bahwa persamaan ax2 + bx + c = 0 kedua akarnya tidak mungkin terletak pada interval (1, 2). B. Pers onen Dalam pembahasan hanya akan disinggung tentang sifat-sifat pada eksponen, yaitu : (ii) ii) Diketahui b memenuhi b1b2 = 2(c + amaan Eksp (i) ao = 1 untuk a ≠ 0 4421Lnkalinaaaaa⋅⋅⋅⋅= untuk n ∈ N. 443 cbcbaaa+=⋅ (i (iv) c = cba− untu ab k a ≠ 0 (vi) a (v) ()bccbaa= mmaa1=− untuk a ≠ 0 21aa= (vii) d a ≥ 0. engan syaratnmnmaa= (viii) Contoh 54 : Harga x yang memenuhi persamaan 45384++=xx adalah ⋅⋅⋅⋅⋅ Solusi : 453+=x 8x+ 4 () () 4533222+ += xx (sifat (v) dan sifat (viii))
  • 34. Pembinaan Olimpiade Matematika Eddy Hermanto, ST Aljabar 30 8(x + 3) = 3(x + 5) 59−=x Jadi, nilai x yang memenuhi adalah x = −59 . Contoh 55 : Manakah yang le 175 75 uktikan. 2175 = (2 25 = 12825 dan 575 = (53)25 = 12525 adi, 2175 lebih besar dari 575. bih besar : 2 atau 5 ? B Solusi : 7) 12825 > 12525 2J 175 > 575 Contoh 56 : (OK 20 S02) Bilangan () ()284 sama dengan ⋅⋅⋅⋅⋅ 24 8 Solusi : () ( ) 122 441628 2 2 32 4 32 32 = = = TIHAN 5.B aan 8LA 1. Persam()xx33243112273=⋅− memberikan nilai x sama dengan ⋅⋅⋅⋅⋅ 2. Jika 53x = 8, maka 5 + x = ⋅⋅⋅⋅⋅⋅ . Jumlah akar-akar persamaan 5x+1 + 56−x = 11 adalah ⋅⋅⋅⋅⋅⋅⋅⋅⋅ . Himpunan penyelesaian dari 8−2 x + ⋅ − = adalah ⋅⋅⋅⋅⋅ rsamaan 3 2 2 x − x+ + 1 2 enyelesaiannya, maka 1 dan x2, maka (x1 ⋅ x2)2 = ⋅⋅⋅⋅⋅ C. Persamaan Logaritma a b = alog c. Sifat-sifat pada logaritma, yaitu : 3 3 4 025495 3−x5. an pe1032=−xx. Jika x dan x adalah p Diberik 33 LL=+213xx 6. Persamaan 54(6x) + 3x = 6(18x) + 9 mempunyai penyelesaian x Pengertian : Jika ab = c mak (i) aabplogloglog== dengan syarat a, p ≠ 1 dan a, b, p > 0 b p b a log log i) = 1 dengan syarat a > 0 dan a ≠ 1. (i aalog
  • 35. Pembinaan Olimpiade Matematika Eddy Hermanto, ST Aljabar 31 (iii) alogaaabbalog= dengan a,b ≠ 1 da 1 n a,b > 0 (iv) log b + log c = log (bc) dengan syarat a ≠ 1 dan a, b, c > 0 ()cb aaacblogloglog=− (v) dengan syarat a ≠ 1 dan a, b, c > 0 bmbanamloglog⋅== dengan sya n b m an log rat a ≠ 1 dan a, b > 0 alog bn = n ⋅ alog b > 0 ≠ (vi) (vii) b dengan syarat a ≠ 1 dan a, (viii) dengan syarat a, b 1 dan a, b, c > 0 ccbabalogloglog=⋅ nm(ix) a b b am n log = dengan Catatan : Bentuk alog b kadang-kadang ditulis dengan log b. = a dan 3log 4 = b 4 erdasarkan sifat (iii) maka 4log 15 = syarat a ≠ 1 dan a, b > 0. a Contoh 57 : Jika 5log 3 , maka 4log 15 = ⋅⋅⋅⋅⋅⋅ Solusi : Berdi) maka didapat 5log 3 ⋅ 3log 4 = 5log asarkan sifat (vii = ab erdasarkan sifat (iv) maka B 4log 15 = 4log 3 + 4log 5 Bb1 + ab1 Maka 4log 15 = ab a+1 Contoh 58 : Himpunan penyelesaian pertidaksama : an() xlog2 − x32log2− xlog (x + 6) + xxlog12+ = 1 adalah ⋅⋅⋅⋅⋅ SB olusi : sifat (i) dan (iii) maka persoalan di atas ekivalen dengan x x xlog erdasarkan x log (2x − 3) − log (x + 6) + log (x + 2) = 1 Berdasarkan sifat (iv) dan (v) maka ()() ( )623+ + xx = 1 2 x− kesamaan maka x − 3)(x + 2) = x(x + 6) 6 = x2 + 6x dua nilai sebut ke persamaan awal. basis pada xlog (x + 6) yaitu bahwa x > 0 dan x ≠ 1. ai x yang memenuhi adalah {6}. Berdasarkan (2 2x2 + x − x2 − 5x − 6 = 0 (x − 6)(x + 1) = 0 x1 = 6 dan x2 = −1 Uji kex ter x = −1 tidak memenuhi syarat hi syarat basis mapun numerus. Sedangkan x = 6 memenua nil Jadi, himpunan semu
  • 36. Pembinaan Olimpiade Matematika Eddy Hermanto, ST Aljabar 32 Contoh 59 : (OSK 2004) Jika log p + log q = log (p + q), maka p dinyatakan dalam q adalah p = ⋅⋅⋅⋅ olusi : g p + log q = log (p + q) g (pq) = log (p + q) q = p + q S l o lo p p(q − 1) = q J adi, p = 1−qq Contoh 60 : Jika 21 logab = dan kccb= logac , maka k = ⋅⋅⋅ olusi : erdasarkan sifat (i) akan didapat S B2loglogcc log 1 log g 1 = = = k+ c b c a Maka k + 1 = 2 adi, k = 1 Contoh 61 : (OSP 2003) Berapakah nilai x yang memenuhi 4log (2log x) + 2log (4log x) = 2 ? sifat (vi) maka 4log (2log x) = lologba J Solusi : 212 Berdasarkan log (2log x) sehingga 4log (2log x) = 2log (2log x)1/2 erdasarkan sifat (vi) maka 2log (4log x) = 2log ( 212log x) sehingga 2log (4log x) = 2log (2log x B ) x) = 2 42 log (log x) + 2log (4log x) = 2 sehingga 2log (2log x)1/2 + 2log (2log M aka log (xxloglog2212⋅⋅ 2 ) = 2 42loglog22212==⋅⋅xx ()8232=/logx 2log x = 4 2 x = 4 LATIHAN 5.C Jadi, x = 16 ( )() 1. 3315log 2 3 2 log 45 − log5 = ⋅⋅⋅⋅⋅⋅⋅ . (OSK 2003) Misalkan 3a = 4, 4b = 5, 5c = 6, 6d = 7, 7e = 8, dan 8f = 9. Berapakah hasil kali abcdef ? 3. (AIME 1984) Bilangan real x dan y me + 4log y2 = 5 dan 8log y + 4log x2 = 7. Tentukan xy. 3 2 menuhi 8log x
  • 37. Pembinaan Olimpiade Matematika Eddy Hermanto, ST Aljabar 33 (AHSME 1998) Tentukan nilai dari 4. !100log!100log!100log!100log100432 1 + 1 + 1 +L+ 1 i p i x a 3 log = 3 untuk x > y, maka x + y = ⋅⋅⋅⋅⋅ 7. , u2, u3, u4, u5 dan u6 serta u1 + u6 = 11 dan 10log u3 + 10log u4 = 1. ika untuk setiap n = 1, 2, 3, ⋅⋅⋅, 5 berlaku un+1 = p⋅un dan p > 1, maka 10log p = ⋅⋅⋅⋅ . (AIME 1983) Diketahui x, y dan z adalah bilangan real lebih dari 1 dan w adalah bilangan real positif. Jika xlog w = 24, ylog w = 40 dan xyzlog w = 12, tentukan zlog w. 9. (AIME 1988) Diberikan log (8log x) = 8log ( log x). Tentukan nilai dari ( log x)2. 10. dengan n! = 1 x 2 x 3 x ⋅⋅⋅ x n. 5. Batas-batas nilai p agar f(x) = 2log (px2 + px + 3) terdefinisada setiap niladalah ⋅⋅⋅⋅⋅ 6. Jika yxxloglog33+ = 27 dan xylog3+ y Diketahui barisan 6 bilangan u1 J 8 222 (AHSME 1997) Untuk bilangan asli n maka 8log n jika 8log n bilangan rasional f(n) = 0 untuk lainnya 1997 Nilai dari 3()Σ= rupakan faktor dari N dan memenuhi 2log (3log (5 )) 12. (ARML 2000 Individual ka b = deret tak hingga berikut : 1nnf 11. (AHSME 199 8) Ada berapa banyak bilangan prima yang me) = 11. log (7log N ) Ji 2000, hitunglah nilai ()()()()()()...5log2log5log2log5log2lo g 0 40 1 41 2 42 b b + b b + b b + 2 1 1 2 2 D. 1) Persamaan Lingkaran berpusat di (0,0) dan (a,b) ingkaran adalah kumpulan titik-titik yang memiliki jarak yang sama terhadap suatu titik aan : x2 + y2 = r2 yang merupakan persamaan lingkaran berpusat di (0,0) dan berjari-jari r. r2 yang merupakan persamaan lingkaran berpusat di (a,b) dan berjari-jari r. akan didapat persamaan umum lingkaran yang karan jika diketahui pusat lingkaran dan persamaan mus jarak titik ke n jari-jari 13. (AIME 2002) Penyelesaian dari sistem persamaan 225log x + 64log y = 4 dan xlog 225 − ylog 64 = 1 adalah (x, y) = (x1, y1) dan (x2, y). Nilai dari 30log (xyxy) adalah ⋅⋅⋅⋅⋅⋅ Persamaan Lingkaran L tertentu, yaitu pusat lingkaran. Jadi ada dua hal yang sangat berkaitan dengan lingkaran yaitu jari-jari lingkaran, R, dan pusat lingkaran. Dari pengertian lingkaran tersebut jika diturunkan akan didapat persam (x − a)2 + (y − b)2 = Jika persamaan (x − a)2 + (y − b)2 = r2 dijabarkan berbentuk : x2 + y2 + Ax + By + C = 0 Salah satu cara menentukan persamaan ling garis yang menyinggung lingkaran tersebut adalah dengan memanfaatkan ru suatu garis lurus sebab jarak titik pusat ke garis singgung tersebut adalah merupaka
  • 38. Pembinaan Olimpiade Matematika Eddy Hermanto, ST Aljabar 34 lingkaran. Misalkan suatu garis lurus memiliki persamaan Ax + By + C = 0. Maka rumus jarak titik (x1, y1) ke garis tersebut adalah 2211BACByAxd+ ++ =. Misalkan terdapat lingkaran den a 2) Hubungan antara titik dengan lingkaran gan persamaan (x − )2 + (y − b)2 = r2 dan titik (p, q). Maka hubungan titik (p, q) dengan (x − a)2 + (y − b)2 = r2 akan memiliki tiga kemungkinan hubungan : ) Jika (p − a)2 + (q − b)2 < r2 maka titik (p, q) terletak di dalam lingkaran (p, q) terletak pada lingkaran 3) (y − b)2 = r2. Bagaimana ubtitusikan persamaan y = mx + c ke persamaan lingkaran (x − a)2 + (y − b)2 = r2 sehingga h x, yaitu Ax2 + Bx + C = 0. ng lingkaran ntuk mencari hubungan antara s dengan irisan kerucut yang 4) sebut memiliki gradien m. Maka persamaan garis singgung dapat dinyatakan dengan a b) Jika (p − a)2 + (q − b)2 = r2 maka titik c) Jika (p − a)2 + (q − b)2 > r2 maka titik (p, q) terletak di luar lingkaran Hubungan antara garis lurus dengan lingkaran Misalkan diketahui suatu garis lurus y = mx + c dan lingkaran (x − a)2 + hubungan antara garis lurus dan lingkaran tersebut ? S didapat suatu persamaan kuadrat dalam peuba Dari persamaan tersebut dapat dihitung diskriminan = B2 − 4AC. (i) Jika B2 − 4AC < 0 maka garis lurus tidak memoto (ii) Jika B2 − 4AC = 0 maka garis lurus menyinggung lingkaran (iii) Jika B2 − 4AC > 0 maka garis lurus memotong lingkaran di dua titik Prinsip nilai diskriminan di atas tidak hanya dapat digunakan u garis lurus dengan lingkaran tetapi juga hubungan antara garis luru lain seperti parabola, elips maupun hiperbola. Persamaan Garis Singgung pada Lingkaran a) Garis singgung lingkaran dengan gradien tertentu Misalkan diketahui bahwa garis singgung ter (i) Untuk lingkaran x2 + y2 = r2 Persamaan Garis Singgung, 1+±=mrmxy 2 (ii) Untuk lingkaran (x − a)2 + (y −22 b) = r Persamaan Garis Singgung, ()12+±−=−mraxmby b) Mis lkan titik (x , y ) terletak pada lingkaran maka persamaan garis singgung yang melalui titik t ngan ) Untuk lingkaran x2 + y2 = r2 r2 (x − a) + (y1 − b)(y − b) = r2 c) Persa ngkaran ini dapat dilakukan dengan beberapa cara : (i) ertian hubungan antara aris lurus dengan lingkaran ri Garis Singgung melalui titik pada lingkaran a11ersebut dapat ditentukan de (i Persamaan Garis Singgung, x1x + y1y = (ii) Untuk lingkaran (x − a)2 + (y − b)2 = r2 Persamaan Garis Singgung, (x1 − a) maan Garis Singgung melalui titik di luar li Untuk menentukan persamaan garis singgung Dengan mencari rumus diskriminan lalu memanfaatkan peng g (ii) Dengan menggunakan persamaan garis polar (iii) Dengan memanfaatkan persamaan garis singgung dengan gradien m untuk menca nilai m
  • 39. Pembinaan Olimpiade Matematika Eddy Hermanto, ST Aljabar 35 Contoh 62 : SK 2005) Titik A(a, b) disebut titik letis jika a dan b keduanya adalah bilangan bulat. Banyaknya tik letis pada lingkaran yang berpusat di O dan berjari-jari 5 adalah olusi : ngkaran yang berpusat di O dan berjari-jari 5 adalah x2 + y2 = 25 dan (−4, −3). adi, banyaknya titik letis pada lingkaran yang berpusat di O dan berjari-jari 5 ada 12. dalah ⋅⋅⋅⋅⋅⋅ olusi : arak pusat (1, 4) ke garis 3x − 4y − 2 = 0 sama dengan jari-jari lingkaran tersebut. (O ti S Persamaan li Karena 02 + 52 = 32 + 42 = 25 maka pasangan (x, y) bulat yang memenuhi ada 12, yaitu (0, 5), (0, −5), (5, 0), (−5, 0), (3, 4), (3, −4), (−3, 4), (−3, −4), (4, 3), (4, −3), (−4, 3) J Contoh 63 : Persamaan lingkaran yang berpusat di (1, 4) dan menyinggung garis 3x − 4y − 2 = 0 a S J Jarak tersebut = d = ()() 224324413−− = 3. + Contoh 64 : rabola y = x2 + a tepat di satu olusi : 2 + a maka x2 − 6x + a = 0 agar y = 6x memotong parabola y = x2 + a di satu titik adalah Diskriminan = 0 6 − 4a = 0 9 aris singgung x2 + y2 − 6x + 4y − 12 = 0 di titik (7, −5) adalah ⋅⋅⋅⋅⋅ olusi : ubtitusi titik (7, −5) ke persamaan x2 + y2 − 6x + 4y − 12 = 0 didapat 6(7) + 4(−5) − 12 = 0 ersamaan garis singgungnya adalah (x − 3)(7 − 3) + (y + 2)(−5 + 2) = 25 samaan garis singgung lingkaran x2 + y2 − 6x + 4y − 12 = 0 di titik (7, −5) adalah olusi : arena 42 + 22 > 10 maka titik (4,2) terletak di luar lingkaran. Persamaan lingkaran berpusat di (1, 4) dan memiliki jari-jari 3 adalah (x − 1)2 + (y − 4)2 = 9 (OSK 2002) Untuk nilai a yang manakah garis lurus y = 6x memotong pa titik ? S Karena 6x = x Disk = 62 − 4(1)(a) = 36 − 4a Syarat 3 Jadi, a = Contoh 65 : Persamaan g S S (7)2 + (−5)2 − Artinya titik (7, −5) terletak pada lingkaran x2 + y2 − 6x + 4y − 12 = 0. P Jadi, per 4x − 3y = 43 Contoh 66 : Persamaan garis singgung yang ditarik dari titik (4,2) ke lingkaran x2 + y2 = 10 adalah ⋅⋅⋅⋅⋅⋅ S K
  • 40. Pembinaan Olimpiade Matematika Eddy Hermanto, ST Aljabar 36 Alternatif 1 : Persamaan garis melalui titik (4,2) dan gradien m adalah y − 2 = m(x − 4). Subtitusi garis tersebut maan lingkaran didapat 2 + 1)x2 + 2(−4m2 + 2m)x + 16m2 − 16m − 6 = 0 22(−4m2 + 2m)2 − 4(m2 + 1)(16m2 − 16m − 6) 16m − 6) = 0 4 + 16m3 m2 − m2 + 16m + 6 = 0 ke persa x2 + (mx − 4m + 2)2 = 10 (m Diskriminan = Agar y − 2 = m(x − 4) menyinggung lingkaran x2 + y2 = 10 maka diskriminan harus sama dengan 0. 22(−4m2 + 2m)2 − 4(m2 + 1)(16m2 − 16m4 − 16m3 + 4m2 − 16m + 6163m2 − 8m − 3 = 0 (3m + 1)(m − 3) = 0 Jika m − =3r1 maka gais singgung tersebut memiliki persamaan y − 2 = −31(x − 4). Jika m = 3 maka garis singgung tersebut memiliki persamaan y − 2 = 3(x − 4). aris singgung yang ditarik dari titik (4,2) adalah x + 3y = 10 dan 3x − y = 10. t yoy = 10 yaitu ubtitusikan persamaan garis polar tersebut ke lingkaran x + y = 10 didapat 10 maka y = 3 sehingga titik singgung dari garis singgung tersebut pada lingkaran adalah 2 = −1 sehingga titik singgung dari garis singgung tersebut pada lingkaran adalah persamaan garis singgungnya adalah 3x − y = 10. aris singgung yang ditarik dari titik (4,2) adalah x + 3y = 10 dan 3x − y = 10. an garis singgung tersebut adalah Jadi, persamaan g Alternatif 2 : Misalkan tiik (xo, yo) = (4, 2). Persamaan garis polar titik (xo, yo) terhadap lingkaran x2 + y2 = 10 adalah xox + 2x + y = 5 22 S x2 + (5 − 2x)2 = x2 − 4x + 3 = 0 x1 = 1 atau x2 = 3 Jika x1 = 1 1(1,3) sehingga persamaan garis singgungnya adalah x + 3y = 10. Jika x2 = 3 maka y (3,−1) sehingga Jadi, persamaan g Alternatif 3 : Misalkan gradien garis singgung tersebut adalah m. Maka persama 12+±=mrmxy K arena r = 10 2 maka 10102+±=mmxy Karena garis tersebut melalui titik (4,2) maka ()mm4210102−±=+ 22 10m + 10 = 4 − 16m + 16m (3m + 1)(m − 3) = 0 Jika m = −31 maka garis singgung tersebut memiliki persamaan y − 2 = −31 (x − 4). Jika m = 3 maka garis sing tersebut memiliki persamaan y − 2 = 3(x − 4). ng yang ditarik dari titik (4,2) adalah x + 3y = 10 dan 3x − y = 10. . Persamaan lingkaran dengan titik pusat (4,3) dan jari-jari = 4 adalah ⋅⋅⋅⋅⋅⋅ . Persamaan lingkaran yang titik pusatnya terletak pada garis y = x + 1 dan menyinggung sumbu X di titik (5,0) adalah ⋅⋅⋅⋅⋅ gung Jadi, persamaan garis singgu LATIHAN 5.D 1 2
  • 41. Pembinaan Olimpiade Matematika Eddy Hermanto, ST Aljabar 37 3. Suatu lingkaran berjari-jari 5, melalui titik (0,0) dan pusatnya pada garis y = x + 1 mempunyai persamaan ⋅⋅⋅⋅⋅⋅ 4. B(4,−3) dan P(x,y) terletak sedemikian sehingga (PA)2 + (PB)2 = (AB)2. Maka P merupakan titik-titik yang terletak pada busur lingkaran yang memotong sumbu X pada 5. Persamaan garis singgung di titik (7,−1) pada lingkaran x2 + y2 = 50 adalah ⋅⋅⋅⋅⋅ 6. 7. Jari-jari lingkaran yang menyinggung sumbu x di (6,0) dan menyinggung garis y = √3 x adalah ⋅⋅⋅⋅ 8. Persamaan garis singgung yang ditarik dari titik (7,−1) ke lingkaran x2 + y2 = 40 adalah ⋅⋅⋅⋅ 9. Persamaan garis singgung pada lingkaran x2 + y2 = 36 yang tegak lurus garis 4y = −3x + 80 adalah 10. Jarak terjauh dari titik (−12,5) ke lingkaran dengan persamaan x2 + y2 = 100 adalah ⋅⋅⋅⋅ 11. Bilangan real x dan y memenuhi (x + 5)2 + (y − 12)2 = 142, maka nilai minimum dari x2 + y2 ik potong jika k memenuhi a ≤ k ≤ b. Nilai dari b − a adalah ⋅⋅⋅⋅⋅ 13. 6) Diberikan persamaan x + y = 14x + 6y + 6. Nilai terkecil dari 3x + 4y yang memenuhi adalah ⋅⋅⋅⋅⋅ E. Nila dengan ⏐x⏐ dan memiliki pengertian ⏐x⏐ = x jika x ≥ 0 dan ⏐x⏐ = −x jika < 0. Jika hanya memuat satu tanda mutlak maka penyelesaian persamaan dapat dengan enggunakan pengertian nilai mutlak atau dapat juga dengan mengkuadratkan tanda mutlak. olusi : lternatif 1 : ian didapat jika x ≥ 2 maka x − 2 = 8 sehingga x = 10 yang memenuhi persamaan. hingga x = −6 yang juga memenuhi persamaan. adi, penyelesaian x yang memenuhi adalah x = −6 atau x = 10. 2 − 4x − 60 = 0 = 0 Diketahui titik A(−2,1) , koordinat ⋅⋅⋅⋅⋅⋅⋅⋅ Garis lurus 3x + 4y + k = 0 akan menyinggung lingkaran x2 + y2 + 6x + 8y = 0 jika k bernilai ⋅⋅⋅⋅⋅⋅⋅ adalah ⋅⋅⋅⋅⋅⋅ 12. (AHSME 1998) Kedua grafik x2 + y2 = 4 + 12x + 6y dan x2 + y2 = k + 4x + 12y memiliki tit (AHSME 19922 Persamaan Nilai mutlak i mutlak dari x ditulis x m Contoh 67 : Selesaikan persamaan ⏐x − 2⏐ = 8 S A Dari pengert Sedangkan jika x < 2 maka 2 − x = 8 se J Alternatif 2 : Karena ⏐x − 2⏐ bernilai tak negatif maka penyelesaiannya dapat dilakukan dengan mengkuadratkan kedua ruas. (x − 2)2 = 64 x (x − 10)(x + 6) x = 10 atau x = −6
  • 42. Pembinaan Olimpiade Matematika Eddy Hermanto, ST Aljabar 38 Persoalan menjadi lebih rumit apabila dalam persamaan tersebut memuat lebih dari satu tanda annya dapat dilakukan dengan membagi kasus. x + 2⏐ + ⏐3x⏐ = 14 Maka −x − 2 − 3x = 14 sehingga x = −4 (memenuhi bahwa x ≤ −2) Untuk −2 ≤ x ≤ 0 maka |x + 2| = x + 2 dan |3x| = −3x | + |3x| = 14. Maka x + 2 − 3x = 14 sehingga x = −6 (tidak memenuhi bahwa −2 ≤ x ≤ 0) d adalah = { −4, 3} . (OSK 2005) Tentukan semua solusi persamaan ⏐x − 1⏐ + ⏐x − 4⏐ = 2. 2. (AIME 1983) Tentukan nilai minimum ⎪x − 15⎪ + ⎪x − p − 15⎪ untuk suatu nilai x dalam batas p ≤ x ≤ 15 dimana 0 < p < 15. 3. Banyaknya bilangan bulat berbeda y yang memenuhi ⏐x − 7⏐ − ⏐x + 11⏐ = y adalah ⋅⋅⋅⋅ 4. − a⎪ − 3. Jika grafik f memotong sumbu-x tepat di tiga titik, maka a = ···· 5. (OSP 2006) Jika ⏐x⏐+ x + y = 10 dan x + ⏐y⏐ − y = 12, maka x + y = ⋅⋅⋅⋅⋅⋅ 6. banyak tripel bilangan bulat (a,b,c) yang memenuhi ⎪a + b⎪ + c = 19 dan ab + ⎪c⎪ = 97 7. (AHSME 1977) Untuk a, b, c bilangan real taknol, semua kemungkinan nilai dari mutlak. Penyelesai Contoh 68 : (OSP 2003) Apakah himpunan jawab dari persamaan ⏐ Solusi : • Untuk x ≤ −2, maka |x + 2| = −x − 2 dan |3x| = −3x |x + 2| + |3x| = 14. • |x + 2 • Untuk x ≥ 0 maka |x + 2| = x + 2 dan |3x| = 3x |x + 2| + |3x| = 14. Maka x + 2 + 3x = 14 sehingga x = 3 (memenuhi bahwa x ≥ 0) Jai, himpunan jawab dari persamaan |x + 2| + |3x| = 14 LATIHAN 5.E : 1 dari ⎪x − p⎪ + (OSP 2006) Diberikan fungsi f(x) = ⎪⎪x − 2⎪ (AHSME 1997) Ada berapa abccba+++ abccba adalah ⋅⋅⋅⋅⋅ 8. (AHSME 1999) Grafik y = −⎪x − a⎪ + b dan y = ⎪x − c⎪ + d berpotongan di titik (2,5) dan (8,3). Nilai a + c adalah ⋅⋅⋅⋅⋅⋅⋅⋅ 9. IME 1988) xi adalah bilangan real yang memenuhi −1 < xi < 1 dan ⎪x1⎪ + ⎪x2⎪ + ⋅⋅⋅ + ⎪xn⎪ = 19 + 0. (AIME 2001) Fungsi f(x) memenuhi persamaan f(3x) = 3f(x) untuk semua bilangan real x dan (A ⎪x1 + x2 + ⋅⋅⋅ + xn⎪. Tentukan nilai terkecil n yang memenuhi. 1 f(x) = 1 − ⎪x − 2⎪ untuk 1 ≤ x ≤ 3. Tentukan bilangan positif x terkecil yang memenuhi f(x) = f(2001).
  • 43. Pembinaan Olimpiade Matematika Eddy Hermanto, ST Aljabar 39 6. SIST Sistem A. Sist inier istem persamaan umum yang dikenal adalah sistem persamaan linier, yaitu sistem persamaan yang angkat variabelnya tidak lebih dari satu. Untuk menyelesaikan n buah variabel dibutuhkan n buah ersamaan. Penyelesaian sistem persamaan dapat dilakukan dengan menggunakan subtitusi, dengan memanfaatkan matriks. x + y = 3log 29 ⋅⋅⋅⋅⋅⋅⋅⋅⋅⋅⋅⋅⋅⋅⋅⋅⋅⋅⋅⋅ (1) ⋅⋅⋅⋅⋅⋅⋅⋅⋅⋅⋅⋅⋅⋅⋅⋅ (2) EM PERSAMAAN persamaan terdiri dari lebih dari satu persamaan dalam rangka mencari suatu penyelesaian. em Persamaan L S p p eliminasi maupun Contoh 69 : Harga x yang memenuhi : 3x + y = 29 dan x − y = 1 adalah ⋅⋅⋅ Solusi : 3x + y = 29 dan x − y = 1 x − y = 1 ⋅⋅⋅⋅ Dari persamaan (1) dan (2) didapat x = 2 3 log 29+1 dan y = 2129log3− Contoh 70 : (OSK 2003) Hari ini usiaku 31 kali usia ayahku. Lima tahun yang lalu, usiaku 41 kali usia ayahku pada h usiak g ? olusi : saat ini = X dan usia ayahku saat ini = Y, maka : waktu itu. Berapakau sekaran S Misal usiaku X = 3Y X − 5 = 141(Y − 5) X 5 = − 41 (3X − 5) , usiaku saat ini 15 tahun persamaan c + d = 0 + b + d + e = 1 + c + d + e = 2 = 4 persamaan di atas didapat ) = 12 3 hingga e = 3 + d = 3 sehingga d = −2 sehingga c = 2 m persamaan tersebut adalah (a, b, c, d, e) = (−1, 1, 2, −2, 3) 4X − 20 = 3X − 5 X = 15 Jadi Contoh 71 : Selesaikan sistem a + b + a + b + c + e = 5 a a b + c + d + e Solusi : Jumlahkan semua 4(a + b + c + d + e a + b + c + d + e = Maka 0 + e = 3 se 5 1 + c = 3 2 + b = 3 sehingga b = 1 4 + a = 3 sehingga a = −1 Jadi, penyelesaian siste
  • 44. Pembinaan Olimpiade Matematika Eddy Hermanto, ST Aljabar 40 Contoh 72 : Tentukan persamaan lingkaran yang melalui titik (0, 0), (6, 8) dan (7,7) ersamaan umum lingkaran adalah 2 + y2 + Ax + By + C = 0 diketahui dan ada 3 variabel yang dicari yaitu A, B dan C maka soal ini ubtitusikan titik (0,0) ke persamaan lingkaran didapat njadi x2 + y2 + Ax + By = 0 aka 6A + 8B = −100 ersamaan umum lingkaran tersebut adalah x2 + y2 − 6x − 8y = 0 B. bih dari satu atau merupakan perkalian ersamaan tak linier maka persoalan menjadi ontoh 73 : dengan x,y,z > 0 entukan nilai 3x + 2y + 3z. amaan didapat ))2 = 12 ⋅ 20 ⋅ 15 = (60)2 = 60 y − 2 = 4 idapat x = 4, y = 6 dan z = 8 + 2y + 3z = 48. ersamaan tak linier dapat dibawa ke dalam persamaan-telah dijelaskan pada rumus Vieta maka hubungan suku banyak dengan enyelesaikan persoalan sistem persamaan tak linier. ontoh 74 : bc = 24 an x3 − 9x2 + 26x − 24 = 0 ma horner didapat = (x − 2)(x − 3)(x − 4) = 0 b, c) yang memenuhi adalah (2, 3, 4) dan permutasinya. Solusi : P x Karena ada 3 titik yang merupakan sistem persamaan linier dengan 3 variabel (peubah). S C = 0 Maka persamaan umum lingkaran me Berdasarkan titik (6, 8) m 3A + 4B = −50 ⋅⋅⋅⋅⋅⋅⋅⋅⋅⋅⋅⋅⋅⋅⋅⋅⋅⋅⋅⋅⋅⋅⋅⋅⋅⋅⋅ (1) Berdasarkan titik (7, 7) maka 7A + 7B = −98 ⋅⋅⋅⋅⋅⋅⋅⋅⋅⋅⋅⋅⋅⋅⋅ (2) Dari persamaan (1) dan (2) didapat A = −6 dan B = −8 Maka p Sistem Persamaan Tak Linier Dalam sistem persamaan tak linier pangkat variabel bisa le di antara variabel-variabel yang ada. Dalam sistem p lebih sulit dan membutuhkan teknik penyelesaian yang lebih tinggi. C Ditentukan 3 buah persamaan (x − 1)(y − 2) = 12 (y − 2)(z − 3) = 20 (z − 3)(x − 1) = 15 T Solusi : Kalikan ketiga pers ((x − 1)(y − 2)(z − 3 (x − 1)(y − 2)(z − 3) Maka z − 3 = 5, x − 1 = 3 dan D Jadi, 3x Jika persamaan-persamaan dalam sistem p persamaan sebagaimana akar-akarnya dapat digunakan untuk m C Tentukan nilai a, b dan c yang memenuhi sistem persamaan berikut : a + b + c = 9 ab + ac + bc = 26 a Solusi : Sesuai dengan rumus Vieta maka a, b, dan c adalah akar-akar persama Dengan teore x3 − 9x2 + 26x − 24 Tripel (a,
  • 45. Pembinaan Olimpiade Matematika Eddy Hermanto, ST Aljabar 41 Contoh 75 : 0, n + m + n = 71 880 sehingga mn(m + n) = 880 (AIME 1991) m, n adalah bilangan asli yang memenuhi mn + m + n = 71 dan m2n + mn2 = 88 tentukan m2 + n2. Solusi : m m 2n + mn2 = mn + mn880 = 71 mn) ( 2 − 71(mn) + 880 = 0 )(mn − 55) = 0 = 55 , 2) dan (16, 1) tetapi tidak emenuhi m + n = 55. + n = 71 − 55 = 16 menuhi mn = 55 adalah (1, 55), (5, 11), (11, 5), (55, 1). + n = 16 adalah m = 5 dan n = 11 atau m = 11 dan n = 5 Jad Con Ten ab = de = 4 dan ea = 6 olusi : alikan kelima persamaan didapat ⋅ 32 b = 1 dan cd = 3 maka e = 4 sehingga d = 1 dan a = (mn − 16 mn = 16 atau mn • Jika mn = 16 maka m + n = 71 − 16 = 55 hi mn = 16 adalah (1, 16), (2, 8), (4, 4), (8Nilai (m, n) yang memenuada yang m • Jika mn = 55 maka m Nilai (m, n) yang me Yang memenuhi m m2 + n2 = 52 + 112 = 146 i, nilai dari m2 + n2 sama dengan 146. toh 76 : tukan penyelesaian dari sistem persamaan 1 ; bc = 2 ; cd = 3 ; S K (abcde)2 = 24 abcde = 12 atau abcde = −12 • Jika abcde = 12 Mengingat a23 . Maka b = 32 dan c = 3. Jadi, (a, b, c , d, e) = (2, 3 32 , 3, 1, 4) merupakan penyelesaian. = • Jika abcde = −12 23 Mengingat ab = 1 dan cd = 3 maka e = −4 sehingga d = −1 dan a − . 32 Maka b = − dan c = −3. Jadi, (a, b, c, d, e) = (−23 , −32 , −3, −1, −4) merupakan penyelesaian. d b, c, d, e) adalah (23 ,32 , 3, 1, 4) dan (−23 ,−32 Jai, penyelesaian (a, , −3, −1, −4). Con Sele x2 − yz = 3 2 − xy = 5 − xy − xz − yz = 12 toh 77 : saikan sistem persamaan : y2 − xz = 4 z Solusi : Alternatif 1 : x2 + y2 + z2
  • 46. Pembinaan Olimpiade Matematika Eddy Hermanto, ST Aljabar 42 2x2 + 2y2 + 2z2 − 2xy − 2xz − 2yz = 24 2) + (x2 − 2xz + z2) + (y2 − 2yz + z2) = 24 − y)2 + (x − z)2 + (y − z)2 = 24 ⋅⋅⋅⋅⋅⋅⋅⋅⋅⋅⋅ (1) (x2 − yz) = 1 z(y − x) = 1 ⋅⋅⋅⋅⋅⋅⋅⋅⋅⋅⋅ (2) (2), (3) dan (4) ke persamaan (1) (x2 − 2xy + y (x y2 − xz − (y + x)(y − x) + (y − x)(x + y + z) = 1 ⋅⋅⋅⋅⋅⋅⋅⋅⋅⋅⋅⋅⋅⋅⋅⋅ z2 − xy − (y2 − xz) = 1 (z + y)(z − y) + x(z − y) = 1 (z − y)(x + y + z) = 1 ⋅⋅⋅⋅⋅⋅⋅⋅⋅⋅⋅⋅⋅⋅⋅⋅⋅⋅⋅⋅⋅⋅⋅⋅⋅⋅⋅ (3) z2 − xy − (x2 − yz) = 2 (z + x)(z − x) + y(z − x) = 2 (z − x)(x + y + z) = 2 ⋅⋅⋅⋅⋅⋅⋅⋅⋅⋅⋅⋅⋅⋅⋅⋅⋅⋅⋅⋅⋅⋅⋅⋅⋅⋅⋅ (4) Subtitusikan persamaan ()21 zyx++ + ()21zyx++ + ()22zyx++ = 24 (x + y + z)2 = 41 • Jika x + y + z = 2 1 Dari persamaan (2), (3) dan (4) y − x = 2 z − y = 2 z − x = 4 x + (2 + x) + z = 21 2x + z = −23 2x + (4 + x) = −23 sehingga x = −611 y = 2 + (−611 ) = 61 z = 4 + (−611) = 613 • Jika x + y + z = −21 ) y − x = −2 z − y = −2 z − x = −4x + (−2 + x + z =−21 2x + z = 23 2x + (−4 + x) = 23 sehingga x = 611 y = −2 + (611 ) = 61 z = −4 + (611) = 613 −611,61,613 )( 611,−61,−613 Tripel (x, y, z) yang memenuhi adalah (( )) Alternat x2 − y 2 2xz (x − − y) + x(z − y) = 0 − Subtitusikan ke persamaan pada soal didapat y) = 3 ⋅ (5) if 2 : yz + z2 − x = 2y − y)(x + y) + (z − y)(z + y) + z(x (x + y + z)(x + z 2y) = 0 * Jika x + y + z = 0 x2 − y(−x − x2 + y2 + xy = 3 ⋅⋅⋅⋅⋅⋅⋅⋅⋅⋅⋅⋅⋅⋅⋅⋅ (−x − y)2 − xy = 5
  • 47. Pembinaan Olimpiade Matematika Eddy Hermanto, ST Aljabar 43 x2 + y2 + xy = 5 ⋅⋅⋅⋅⋅⋅⋅⋅⋅⋅⋅⋅⋅⋅⋅⋅⋅⋅⋅⋅⋅⋅ (6) pel (x, y, z) yang memenuhi. * ) e persamaan (7) maka ak memenuhi x2 − yz = 3 ersamaan (7) maka Maka tidak ada tri Jika x + z = 2y ⋅⋅⋅⋅⋅⋅⋅⋅⋅⋅⋅⋅⋅⋅⋅⋅⋅⋅⋅⋅⋅⋅⋅ (7) x2 − y(2y − x) = 3 x2 − 2y2 + xy = 3 ⋅⋅⋅⋅⋅⋅⋅⋅⋅⋅⋅⋅⋅⋅⋅⋅⋅⋅⋅ (8 y2 − x(2y − x) = 4 x2 + y2 − 2xy = 4 ⋅⋅⋅⋅⋅⋅⋅⋅⋅⋅⋅⋅⋅⋅⋅⋅⋅⋅⋅⋅ (9) 4(x2 − 2y2 + xy) = 3(x2 + y2 − 2xy) x2 − 11y2 + 10xy = 0 (x + 11y)(x − y) = 0 • Jika x = y Subtitusikan k x = y sehingga x = y = z yang tid • Jika x = −11y Subtitusikan ke p z = 13y y2 − xz = 4 y2 + 143y2 = 4 144y2 = 4 y = ± 6y = 1 * Jika 61 Maka x = −611 dan z = 613 * Jika y = −61 Mk a a x = 611 dan z = −613 Tripel (x, y, z) yang memenuhi adalah (( −611,61,613 )( 611,−61,−613 )) LATIHAN 6 : 1. − n ,− i-jari ⋅⋅⋅⋅⋅⋅ 2. entukan semua nilai x + y real yang memenuhi sistem persamaan : x + y + Lingkaran yang melalui titik-titik (2,2), (2,4)da (51) mempunyai jar Tyx = 232 dan () yyxx+ = 2007 4. (OSK 2005) Diberikan tiga bilang if ng s berbeda. Jika 3. Tentukan semua penyelesaian pasangan (x, y) real yang memenuhi x2 + y2 + x + y = 12 xy + x + y = 3 an posit x, y dan z yaemuanya zxy− = zyx+ = yx , maka nilai yx sama dengan 5. 1969) Tunjukkan bahwa jika ( Canadian MO332211baba ba == dan p1, p2 dan p3 semuanya tidak sama dengan nol, maka : nnnnnnn p b p b p b p a p a p a b a 1 1 2 2 3 3 1 1 2 2 3 3 1 1 + + + + = ⎟ ⎟⎠ ⎞ ⎜ ⎜⎝ ⎛
  • 48. Pembinaan Olimpiade Matematika Eddy Hermanto, ST Aljabar 44 6. (AIME 1989/OSN 2004) Diberikan x1 + 4x2 + 9x3 + 16x4 + 25x + 36x + 49x = 1 4x 5 6 7 x4 + 36x5 + 49x6 + 64x7 = 12 9x1 + 16x2 + 25x3 + 36x4 + 49x5 + 64x6 + 81x7 = 123. Tentukan nilai dari 16x1 + 25x2 + 36x3 + 49x4 + 64x5 + 81x6 + 100x7 7. (Irish MO 1999) Selesaikan sistem persamaan berikut : 8. ) yang memenuhi bahwa salah satu bilangan jika alah 2. menuhi sistem persamaan : 1 + 9x2 + 16x3 + 25 . y2 = (x + 8)(x2 + 2) y2 − (8 + 4x)y + (16 + 16x − 5x2) = 0 (Canadian MO 1970) Tentukan semua tripel (x, y, z ditambahkan dengan hasil kali kedua bilangan yang lain hasilnya ad 9. (COMC 1997) Tentukan bilangan real x, y dan z yang me 42=−yzx 6=−xzy 30−=−xyz 10.(Vietnamese MO 1996) Selesaikan sistem persamaan berikut : 2113=⎟⎟⎠ ⎞ ⎜⎜⎝ ⎛ + + yxx 24117=⎟⎟⎞ ⎠ ⎜ ⎜⎝ ⎛ + − x y y 7. KETAKSAMAAN A. Konsep Urutan dan Sifat-sifat dasar dari konsep urutan Sifat penting pada bilangan-bilangan real adalah adanya urutan sehingga dapat membandingkan dua ilangan sehingga didapat apakah kedua bilangan tersebut sama atau tidak sama. Sifat-sifat dari dari konsep urutan pada sistem bilangan real : (1) Setiap bilangan real a hanya memenuhi satu dan hanya satu dari kemungkinan (i) a = 0 0 (3) dan b > 0 maka a + b > 0 (4) dan b > 0 atau a < 0 dan b < 0 maka ab > 0 (5) dan b < c maka a < c (7) dan c < d maka a + c < b + d (8) dan c > 0 maka ac < bc (9) dan c < 0 maka ac > bc b (ii) a > (iii) a < 0 (2) Setiap bilangan real a dan b hanya memenuhi satu dan hanya satu dari kemungkinan (i) a = b (ii) a > b (iii) a < b Jika a > 0 Jika a > 0 Jika a < b (6) Jika a < b maka a ± c < b ± c untuk setiap bilangan real c Jika a < b Jika a < b Jika a < b (10)Jika a > 0 maka a1 > 0 (11)Jika a > 0 dan b > 0 maka ba > 0
  • 49. Pembinaan Olimpiade Matematika Eddy Hermanto, ST Aljabar 45 ab11<(12)Jika 0 < a < b atau a < b < 0 maka . aka a > b. nda < diganti dengan tanda ≤ kecuali untuk sifat (12) yang d keduanya tak nol. sitif yang memenuhi (13)Jika a > 0 dan b > 0 serta a > b ma 22 Sifat-sifat tersebut juga berlaku jika tmensyaratkan bahwa aan b Contoh 78 : db a < c maka ddbb a < a c < c Buktikan bahwa jika a, b, c dan d adalah bilangan po+ olusi : + . lternatif 1 : sitif serta S A Karena bd podcba< maka d + ab < bc + ab ⋅⋅⋅⋅⋅⋅⋅⋅⋅⋅⋅⋅⋅⋅⋅⋅⋅⋅⋅ (sifat 6) < b(a + c) ) positif maka ad < bc ⋅⋅⋅⋅⋅⋅⋅⋅⋅⋅⋅⋅⋅⋅⋅ (sifat 7) a a(b + d) Karena b(b + ddbcaba+< ⋅⋅⋅⋅⋅⋅⋅⋅⋅⋅⋅⋅⋅⋅⋅⋅⋅⋅⋅⋅⋅ 7) ⋅⋅⋅⋅⋅⋅ + (sifat ⋅⋅⋅⋅⋅⋅⋅⋅⋅⋅⋅⋅⋅⋅⋅⋅⋅⋅⋅⋅⋅⋅⋅⋅⋅⋅⋅⋅⋅⋅⋅⋅ (1) Karena bd positif serta dcba< maka ⋅⋅⋅⋅⋅⋅⋅⋅⋅⋅⋅⋅⋅⋅⋅⋅⋅⋅⋅ (sifat 6) Karena ad < bc ⋅⋅⋅⋅⋅⋅⋅⋅⋅⋅⋅⋅⋅⋅⋅ (sifat 7) ad + cd < bc + cd d(a + c) < c(b + d) d(b + d) positif maka ddb<+ ⋅⋅⋅⋅⋅⋅⋅⋅⋅⋅⋅⋅⋅⋅⋅⋅⋅⋅⋅⋅⋅ 7) ⋅⋅⋅⋅⋅⋅ a+c c (sifat ⋅⋅⋅⋅⋅⋅⋅⋅⋅⋅⋅⋅⋅⋅⋅⋅⋅⋅⋅⋅⋅⋅⋅⋅⋅⋅⋅⋅⋅⋅⋅⋅ (2) idapat Dari persamaan (1) dan (2) d ddbb<<+ (terbukti) ccaa + Alternatif 2 : Karena dcba< maka bc > ad badbca−+ + = ()dbadc+ b b − arena bc > ad sedangkan b dan (b + d) positif maka K bdb+ = ( +c − a ) adbb+ bc−ad > 0 Jadi, badba+ +c > ⋅⋅⋅⋅⋅⋅ (3) ⋅⋅⋅⋅⋅⋅⋅⋅⋅⋅⋅⋅⋅⋅⋅⋅⋅⋅⋅⋅⋅⋅⋅⋅⋅⋅ dbcadc+ +− = ()dbdadbc+ − Karena bc > ad sedangkan d dan (b + d) positif maka dbcadc+ +−= ( ) dbd > 0 adbc + − Jadi, dbcadc+ +> ⋅⋅⋅⋅⋅⋅⋅⋅⋅⋅⋅⋅⋅⋅⋅⋅⋅⋅⋅⋅⋅⋅⋅⋅⋅⋅⋅⋅⋅⋅ (4) Dari pers aan (3) dan (4) didapat am ddbb+ (terbukti) a < a+c c Jika a ≥ b da buktikan bahwa < Contoh 79 : n x ≤ y maka2byax+ ≤ 2ba+ ⋅ 2yx+
  • 50. Pembinaan Olimpiade Matematika Eddy Hermanto, ST Aljabar 46 Solusi : 2 ba+ ⋅ 2yx+ − 2byax+ = 422byaxbybxayax−−+++ = ()() 4xyba−− Karena a ≥ b dan x≤ y maka (a − b)(y − x) ≥ 0. adi, J2ba+ ⋅ 2yx+ − 2byax+ ≥ 0. Tanda kesamaan terjadi jika a = b atau x = y. erbukti bahwa T2byax+ ≤ 2ba+ ⋅ 2yx+ . Contoh 80 : Jika 21<+x , maka batas-batas nilai x yang memenuhi adal x 0 − + 2 ah ⋅⋅⋅ etaksamaan pada soal memiliki syarat bahwa bilangan dalam tanda akar tidak boleh negatif hingga ≥ −2 yang berakibat syarat pada soal adalah x ≥ −2 ⋅⋅⋅⋅⋅⋅⋅⋅⋅⋅⋅⋅⋅⋅⋅⋅⋅⋅⋅⋅⋅⋅⋅⋅⋅⋅⋅ (1) Solusi : K s e x ≥ −10 dan x10+x < 2 + 2+x Akar dari suatu bilangan selalu positif sehingga jika dikuadratkan tidak akan mempengaruhi tanda aan. ketaksam x + 10 < 2 + x + 2 + 42+x 3 < 22+x 9 < 4x + 8 x > 41 ⋅⋅⋅⋅⋅⋅⋅⋅⋅⋅⋅⋅⋅⋅⋅⋅⋅⋅⋅⋅⋅⋅⋅⋅⋅⋅⋅⋅ (2) Jadi, batas-batas nilai x yang memenuhi adalah x > 41 . B. at Seba ang Bilangan Real Selalu Tak Negatif ita ketahui bahwa kuadrat dari suatu bilangan real tidak mungkin negatif. Konsep ini uk menyelesaikan suatu persoalan. a sebarang bilangan real maka a2 ≥ 0. Tanda kesamaan terjadi hanya jika a = 0. 2 2 dan b. Kapan tanda kesamaan terjadi ? aan terjadi jika a = b = 2. Buktikan bahwa ab + bc tidak lebih dari 1. b) b + bc = 1 − (b − 1)2 gan kuadrat tidak mungkin negatif maka ab + bc ≤ 1 dengan tanda kesamaan terjadi KuadrrSebag k aimanapenting untJika : Contoh 81 Buktikan bahwa a + b ≥ 2ab untuk bilangan real a Solusi : Karena bilangan kuadrat tidak mungknegatif in maka (a − b)2 ≥ 0 T anda kesam22− a + b 2ab ≥ 0 a 2 + b2 ≥ 2ab (terbukti) Contoh 82 : Diketahui a, b, c > 0 serta a + b + c Solusi : ab + bc = b(a + c) = b(2 − a Karena bilan jika b = 1. adi, terbukti bahwa ab + bc tidak lebih dari 1. J
  • 51. Pembinaan Olimpiade Matematika Eddy Hermanto, ST Aljabar 47 Sifat bilangan kuadrat tidak mungkin negatif tidak hanya digunakan untuk menyelesaikan masalah i juga menyangkut persamaan. HSME 1997) Jika x, y dan z adalah bilangan real yang memenuhi (x − 3)2 + (y − 4)2 + (z − 5)2 = 0 aka nilai dari x + y + z adalah arena bilangan kuadrat tidak mungkin negatif maka penyelesaian (x − 3)2 + (y − 4)2 + (z − 5)2 = 0 anya didapat jika x − 3 = 0, y − 4 = 0 dan z − 5 = 0. esaian (x, y, z) yang memenuhi adalah x = 3, y = 4 dan z = 5. C. aan Rataan Kuadrat (QM), Rataan Aritmatik (AM), Rataaan Geometri (GM) dan Rataan asing rataan. ataan Kuadrat (QM) = pertidaksamaan tetap Contoh 83 : (A m Solusi : K h Maka, penyel Jadi, nilai dari x + y + z = 3 + 4 + 5 = 12. Ketaksam Harmonik (HM) Perlu dijelaskan terlebih dahulu pengertian masing-m Misalkan x1, x2, x3, ⋅⋅⋅, xn adalah bilangan real positif. R nx xxx222+++ n 2 1 2 3 L+ (AM) = Rataan Aritmatik nxxxxn++++L321 Rataan Geometri (GM) = nnxxxxL321 Rataan Harmonik (HM) = n x x x x n 1 1 1 1 1 2 3 + + +L+ n HM dari bilangan-bilan Solusi : M = Contoh 84 : Hitunglah QM, AM, GM dagan 2, 3 dan 7. Q 3623732222=++ AM = 3 2+3+7 = 4 GM = 3342732=⋅⋅ H M = 71 3121+ 3 + = 41126 Hubungan antara QM, AM, GM dan HM adalah QM ≥ AM ≥ GM ≥ HM kesamaan terja ka x1 = x2 = x3 = ⋅⋅⋅ = xn Tandadi ji
  • 52. Pembinaan Olimpiade Matematika Eddy Hermanto, ST Aljabar 48 Contoh 85 : positif a dan b. olusi : engan memanfaatkan ketaksamaan AM-GM didapat Buktikan bahwa a2 + b2 ≥ 2ab untuk bilangan real S D222ba+ ≥ 22ba = ab aan terjadi jika a2 = b2 sehingga a = b. Tentukan nilai minimal dari Tanda kesam a2 + b2 ≥ 2ab (terbukti) Contoh 86 : xzzyyx842++ dan kapan nilai terkecil itu terjadi ? e s an AM-GM maka Solusi : rdaarkan ketaksama B xzy842 x + y + z ≥ 3 ⋅ 3842xzzyyx⋅⋅ = 43 Jadi, nilai minimal dari xzzyyx2+lah 4 8 + ada 43 . Tanda kesamaan terjadi jika xzzyyx842== . Misalkan xzzyyx842== = k maka k3 = xz8 = z y y x 2 4 ⋅ ⋅ 641 k = 41 Jadi, y2 = x 4 sehingga y = 2x 1 x8 = z 4 sehingga zJadi, tanda k 1 = 2x esamaan terjadi jika y = z = 2x nilai min l ri Makaimada xzzyyx842++ adalah 43 yang terjadi jika y = z = 2x. Untuk a, b dan c bilangan positif, buktikan ketaksamaan (a + b)(b + c)(c + a) ≥ 8abc. n AM ≥ GM Contoh 87 : Solusi : Berdasarkan ketaksamaamaka 2ba+ ≥ ab a + b ≥ 2ab Dengan cara yang sama maka a + c ≥ 2ac b + c ≥ 2bc Tanda kesam an terjadi jika a= b = c. (b + c) ≥ 2 a(a + b)(a + c)ab ⋅ 2 ac ⋅ 2 bc (a + b)(b + c)(c + a) ≥ 8abc (terbukti)
  • 53. Pembinaan Olimpiade Matematika Eddy Hermanto, ST Aljabar 49 Contoh 88 : Buktikan bahwa untuk x, y, z positif maka x2 + y2 + z2 ≥ xy + xz + yz dan kapan tanda kesamaan olusi : erdasarkan ketaksamaan AM-GM maka 2 + y2 ≥ 2xy 2 + z2 ≥ 2xz n ketiga persamaan didapat x2 + 2y2 + 2z2 ≥ 2xy + 2xz + 2yz z2 ≥ xy + xz + yz (terbukti) inimal dari terjadi. S B x x y2 + z2 ≥ 2yz Tanda kesamaan terjadi jika x = y = z Jumlahka 2 x2 + y2 + Contoh 89 : (OSP 2009/AIME 1983) Tentukan nilai m xxxxsin4sin922+ untuk 0 < x < π. Solusi : xxxxsin4sin922+ = 9x sin x + xxsin4 Berdasarkan ketaksamaan AM-GM maka xxxxsin4sin922+ = 9x sin x + xxsin4 ≥ xxxxsin4sin92⋅ = 12 Maka nilai minimum dari xxxxsin4sin922+ sama dengan 12. LATIHAN 7 : 1. (OSK 2010) Bilangan bulat yang memenuhi pertidaksamaan x4 ≤ 8x2 − 16 sebanyak ⋅⋅⋅⋅⋅ 2. SP 2004) Tentukan semua bilangan real x yang memenuhi x2< ⎪2x − 8⎪ 3. IME 1987) Tentukan bilangan bulat terbesar n sehingga terdapat bilangan unik k yang memenuhi (O (A137 < < n+k n . 158 4. (India RMO 1995) Tunjukkan bahwa untuk sembarang bilangan real x maka 0cossin2122>+++xxxxx . (OSP 2009) Banyaknya bilangan real x yang memenuhi persamaan x − 2x3 + 5x2 − 176x + 2009 = 0 6. Selesaikan persamaan berikut dalam bilangan real 4 5adalah ⋅⋅⋅⋅⋅⋅ v uzyxvuzyx++++=++−++22 7. altic Way 2000) Tentukan semua bilangan real positif (x, y) yang memenuhi persamaan : (B x + y − x1 − y1 + 4 = 2()1212−+−yx